ΣΗΜΕΙΩΣΕΙΣ ΦΙΛΟΣΟΦΙΑΣ i

91
ΣΗΜΕΙΩΣΕΙΣ ΦΙΛΟΣΟΦΙΑΣ I ΠΛΑΤΩΝΑΣ Τι είναι ιδέες, ποιο το επιχείρημα για την ύπαρξη των ιδεών; Οι ιδέες είναι οντότητες, με τη δική τους, ανεξάρτητη οντολογική υπόσταση. Οι ιδέες δηλαδή υπάρχουν ανεξάρτητα από τη νόηση. Κατά τον Πλάτωνα οι ιδέες είναι οι αιτίες των αισθητών. Δεν υπάρχει δηλαδή τίποτα στον αισθητό κόσμο που να μην χρωστά την οντολογική του υπόσταση απόλυτα στις ιδέες. Αυτό το δικαιολογεί με το επιχείρημα ότι όλοι έχουμε γνώση π.χ. για την ισότητα και μπορούμε να αναγνωρίζουμε την παρουσία της στα πράγματα. Όσα ξέρουμε όμως δεν τα έχουμε μάθει μόνο μέσω της παρατήρησης. Συνεπώς, θα πρέπει να έχουμε αποκτήσει αυτή τη γνώση από κάπου αλλού. Υπάρχει λοιπόν μια οντότητα με την οποία έχουν έρθει σε γνωσιολογική επαφή και έχουμε αποκτήσει ένα είδος "πρώην γνώσης". Αυτή η οντότητα για τον Πλάτωνα είναι η ιδέα της ισότητας. Επιπλέον, τα αισθητά πράγματα είναι οντολογικά ανεπαρκή. Ενώ οι ιδέες είναι αμετάβλητες και οντολογικά πλήρεις, γι' αυτό και αποτελούν τα πραγματικά αντικείμενα της γνώσης. Τι είναι η πλατωνική Πολιτεία; Η Πλατωνική Πολιτεία θέτει το ερώτημα τι είναι δικαιοσύνη. Ο Πλάτωνας μιλάει για μια ιδεατή πολιτεία που αποτελείται από 3 τάξεις (παραγωγοί, φύλακες, κυβερνήτες). Η τοποθέτηση των πολιτών γίνεται σύμφωνα με ποιο μέρος της ψυχής τους είναι κυρίαρχο και όχι με κριτήρια όπως φύλο, ηλικία, καταγωγή (ακόμα και δούλοι γίνονται κυβερνήτες). Το βασικό κριτήριο είναι η απόδοση των πολιτών στα μαθηματικά και την φιλοσοφία. Αυτό δεν σημαίνει ότι όσοι

Upload: mihalis-siafarikas

Post on 06-Aug-2015

141 views

Category:

Documents


0 download

TRANSCRIPT

Page 1: ΣΗΜΕΙΩΣΕΙΣ ΦΙΛΟΣΟΦΙΑΣ I

ΣΗΜΕΙΩΣΕΙΣ ΦΙΛΟΣΟΦΙΑΣ I

ΠΛΑΤΩΝΑΣ

Τι είναι ιδέες, ποιο το επιχείρημα για την ύπαρξη των ιδεών; Οι ιδέες είναι οντότητες, με τη

δική

τους, ανεξάρτητη οντολογική υπόσταση. Οι ιδέες δηλαδή υπάρχουν ανεξάρτητα από τη

νόηση.

Κατά τον Πλάτωνα οι ιδέες είναι οι αιτίες των αισθητών. Δεν υπάρχει δηλαδή τίποτα στον

αισθητό κόσμο που να μην χρωστά την οντολογική του υπόσταση απόλυτα στις ιδέες. Αυτό

το

δικαιολογεί με το επιχείρημα ότι όλοι έχουμε γνώση π.χ. για την ισότητα και μπορούμε να

αναγνωρίζουμε την παρουσία της στα πράγματα. Όσα ξέρουμε όμως δεν τα έχουμε μάθει

μόνο

μέσω της παρατήρησης. Συνεπώς, θα πρέπει να έχουμε αποκτήσει αυτή τη γνώση από

κάπου

αλλού. Υπάρχει λοιπόν μια οντότητα με την οποία έχουν έρθει σε γνωσιολογική επαφή και

έχουμε αποκτήσει ένα είδος "πρώην γνώσης". Αυτή η οντότητα για τον Πλάτωνα είναι η

ιδέα

της ισότητας. Επιπλέον, τα αισθητά πράγματα είναι οντολογικά ανεπαρκή. Ενώ οι ιδέες

είναι

αμετάβλητες και οντολογικά πλήρεις, γι' αυτό και αποτελούν τα πραγματικά αντικείμενα

της

γνώσης.

Τι είναι η πλατωνική Πολιτεία;

Η Πλατωνική Πολιτεία θέτει το ερώτημα τι είναι δικαιοσύνη. Ο Πλάτωνας μιλάει για μια

ιδεατή

πολιτεία που αποτελείται από 3 τάξεις (παραγωγοί, φύλακες, κυβερνήτες). Η τοποθέτηση

των

πολιτών γίνεται σύμφωνα με ποιο μέρος της ψυχής τους είναι κυρίαρχο και όχι με κριτήρια

όπως φύλο, ηλικία, καταγωγή (ακόμα και δούλοι γίνονται κυβερνήτες). Το βασικό κριτήριο

είναι

η απόδοση των πολιτών στα μαθηματικά και την φιλοσοφία. Αυτό δεν σημαίνει ότι όσοι

Page 2: ΣΗΜΕΙΩΣΕΙΣ ΦΙΛΟΣΟΦΙΑΣ I

βρίσκονται στις άλλες δύο κατηγορίες είναι χαζοί, απλώς εφαρμόζουν τα ενδιαφέροντα

τους σε

άλλους τομείς. Οι φιλόσοφοι-κυβερνήτες κατέχουν τις 3 εξουσίες και το έργο

διακυβέρνησης. Οι

φύλακες βοηθούν σ' αυτό, οι παραγωγοί παράγουν αγαθά για όλους. Κριτική Αριστοτέλη:

δεν

τη δέχεται, γιατί η Πλατωνική Πολιτεία καταργεί τη διαφορετικότητα του πολίτη και τον

εντάσσει σε ένα σύνολο όμοιων, καταργεί θεσμούς όπως η οικογένεια και επιβάλλει

άλλους

όπως η κοινοκτημοσύνη των παιδιών, προκειμένου να υπάρξει απόλυτη ενότητα στο

εσωτερικό

της.

ΑΚΙΝΑTΗΣ (μέσος - όχι ακραίος - πραγματοκράτης /ρεαλιστής)

Σχέση φιλοσοφίας-θεολογίας: δε δέχεται το δόγμα της διπλής αλήθειας (Αβερρόης),

υποστηρίζει

ότι η τέλεια αλήθεια για τον Θεό βρίσκεται μόνο στη θεολογία. Κάνει διάκριση ως προς τη

μέθοδο (θεολογία-αποκάλυψη δόγματος, φιλοσοφία-ανθρώπινη λογική) και ως προς το

αντικείμενο (θεολογία-Θεός, φιλοσοφία-άνθρωπος και κόσμος). Δύο είδη θεολογίας:

φυσική

θεολογία (χρήση λογικής) και δογματική - θεολογία (χρήση δόγματος). ΔΕΝ πιστεύει ότι οι

φιλοσοφικές θεωρίες είναι καταδικασμένες σε πλάνη, αλλά ότι είναι ατελείς.

Διάκριση μεταξύ ουσίας και ύπαρξης στον Ακινάτη

Ο ακινάτειος υλομορφισμός έχει εφαρμογή μόνο στον ορατό κόσμο. Ύλη και μορφή έχουν

τα

δημιουργήματα του αισθητού κόσμου (όχι άγγελοι και ψυχή), ενώ η μορφή πρέπει να είναι

εξατομικευμένη. Τι γίνεται λοιπόν με τον αόρατο κόσμο; Ο Ακινάτης διαχωρίζει τα κτιστά

όντα

από τον Άκτιστο Θεό και λέει ότι η ουσία είναι η δυνατότητα για ύπαρξη και η ύπαρξη η

πραγματωμένη αυτή δυνατότητα. Άρα, στα κτιστά όντα (υλικά και μη) η ύπαρξη

διαχωρίζεται

από την ουσία. Μόνο στον Θεό η ύπαρξη ταυτίζεται με την ουσία του.

Page 3: ΣΗΜΕΙΩΣΕΙΣ ΦΙΛΟΣΟΦΙΑΣ I

Η θεωρία της ουσιοκρατίας: ΑΚΙΝΑΤΗΣ: τη δέχεται, γιατί η ουσία έχει μεγαλύτερη σημασία

από

την ύπαρξη. Το ανθρώπινο πνεύμα που ερευνά τον κόσμο ενδιαφέρεται άμεσα για την

ουσία

των υλικών πραγμάτων. ΟΚΚΑΜ: δεν τη δέχεται. Η παρανόηση των ρεαλιστών (Ακινάτης)

τους

οδήγησε σε λάθος συμπεράσματα για τον κόσμο και το θεό. Η παντοδυναμία και η απόλυτη

ελευθερία του Θεού, βασικά στοιχεία του χριστιανικού δόγματος δεν συμβιβάζονται με την

ουσιοκρατία. Για τον Όκκαμ το πιo σημαντικό αντικείμενο της μεταφυσικής είναι η ύπαρξη.

Σχέση εκκλησιαστικής-πολιτικής ουσίας: ΑΚΙΝΑΤΗΣ: Δόγμα δύο σπαθιών: συμμαχία δύο

εξουσιών. Ενοποιεί τις 2 θεωρώντας ανώτερη την εκκλησιαστική που καθοδηγεί την

κρατική.

Για το καλό των πολιτών, ο άρχοντας πρέπει να υπακούει στο θεϊκό και φυσικό νόμο

(στόχος η

τέλεια ευδαιμονία), αλλιώς ο υπήκοος έχει το δικαίωμα να μην υπακούσει. Η θεωρία έθεσε

ηθικούς φραγμούς στους άρχοντες και τους υποχρέωνε να σεβαστούν τις ελευθερίες του

πολίτη. ΟΚΚΑΜ (ακραίος ονοματοκράτης): επαναστατική θεωρία πλήρης διαχωρισμός

κοσμικής

- εκκλησιαστικής εξουσίας. Αμφισβήτησε πολιτική εξουσία και ιδιοκτησία υλικών αγαθών

σε

πάπα. Δεν πιστεύει ότι η πολιτική εξουσία των ευρωπαίων ηγεμόνων πρέπει να

υποτάσσεται

στον πάπα.

ΝΤΕΚΑΡΤ=ΚΑΡΤΕΣΙΟΣ (νοησιαρχικός, ορθολογιστής, απόρριψη εμπειρίας, πίστη σε

έμφυτες ιδέες και ορθό λόγο)

1. Cogito ergo sum: πρόκειται για την ενορατική Βεβαιότητα, την ύπαρξή, μας. Εδώ βασίζει

τη

θεμελίωση της γνώσης, χρησιμοποιώντας μάλιστα το ίδιο το όπλο των σκεπτικιστών,

δηλαδή τη

συστηματική αμφιβολία. Η ικανότητά μου να σκέφτομαι και να αμφιβάλλω για τα πάντα,

βεβαιώνει αυτόματα και την ίδια μου την ύπαρξη. Σκέφτομαι άρα υπάρχω. Η ίδια η

αμφιβολία

Page 4: ΣΗΜΕΙΩΣΕΙΣ ΦΙΛΟΣΟΦΙΑΣ I

μου δηλ. με βεβαιώνει ότι είμαι μια οντότητα, ότι υπάρχω. Η γνώση που πηγάζει από εδώ

είναι

άμεση και ενορατική. Κάθε άνθρωπος δηλαδή έχει άμεση συνείδηση για ό,τι συμβαίνει

μέσα του.

Ο Καρτέσιος δεν πιστεύει στις εμπειρικές γνώσεις (κατώτερες γνωστικές δυνάμεις), αλλά

στις

έμφυτες ιδέες που προϋπάρχουν κάθε εμπειρίας, ως αιώνιες μεταφυσικές αρχές. Είναι

ορθολογιστής και νοησιαρχικός, γιατί ρίχνει το βάρος στη λογική των φυσικών επιστημών

και

μαθηματικών για την αναζήτηση της γνώσης και λεει ότι ο λόγος καθορίζει τη βούληση.

Επίσης,

δίνει έμφαση στη μέθοδο προσέγγισης της αλήθειας κι όχι στην ουσία των πραγμάτων.

Βλέπει

το σύμπαν μηχανιστικά, ως ένα αιτιακά καθορισμένο σύστημα μαθηματικών αρχών, όπου

τίποτα

δεν είναι τυχαίο.

2. Πώς σχετίζεται το Cogito με το δυϊσμό του Καρτεσίου: έκανε διάκριση μεταξύ

πνευματικού

και υλικού κόσμου. Σώμα και ψυχή=δύο εντελώς διαφορετικές μεταξύ τους οντότητες. Η

ιδιότητα του υλικού σώματος είναι η έκταση (σώμα=εκτατή υλική οντότητα), της ψυχής η

συνείδηση (ψυχή=μη εκτατή, άυλη οντότητα - όταν ένα ψυχικό γεγονός εκδηλώνεται μέσα

μου

π.χ. πόνος, έχω συνείδηση γι' αυτό). Πιστεύει στη μηχανική αντίληψη για το σύμπαν (ζώα,

άψυχα πράγματα, ανθρώπινο σώμα είναι μηχανές). Η ψυχή όμως δεν υπόκειται στους

φυσικούς

νόμους όπως το σώμα. Συνδέει την ύπαρξη της ψυχής με το Cogito, γιατί λέει ότι

καταλαβαίνω

την παρουσία της ψυχής, επειδή υπάρχει κάτι μέσα μου που σκέφτεται. Ταυτίζει δηλαδή:

την

ύπαρξη του, τον εαυτό του. με την ψυχή. Υπάρχει κανείς όσο σκέπτεται, αμφιβάλλει, όσο

εκδηλώνονται μέσα του ψυχικά γεγονότα. (κωνάριο: υπόφυση εγκεφάλου που εξασφαλίζει

τη

σχέση σώματος-ψυχής).

Page 5: ΣΗΜΕΙΩΣΕΙΣ ΦΙΛΟΣΟΦΙΑΣ I

3. Πού βάζει το Θεό ο Καρτέσιος στο σύστημά του και γιατί; Στο σύστημα του Καρτέσιου, ο

Θεός είναι η ασφαλιστική δικλείδα που μας εγγυάται ότι ο τρόπος που το πνεύμα μας

αντιλαμβάνεται τον εξωτερικό κόσμο είναι σωστός κι όχι ένα σύνολο παραισθήσεων. Ο

Θεός, ως

τέλειο και καλό ον, δεν θα επέτρεπε ποτέ κάτι τέτοιο. Χρησιμοποιεί δηλαδή τη βεβαιότητα

της

ύπαρξης του Θεού για να θεμελιώσει τη γνώση μας για τον κόσμο. Ο Θεός είναι η ιδέα ενός

τέλειου όντος και ως τέτοια δεν μπορεί να πηγάζει από τις αισθήσεις μας. Άρα είναι μια

έμφυτη

ιδέα που προϋπάρχει στον ανθρώπινο νου. Ο Θεός λοιπόν υπάρχει κατά λογική

αναγκαιότητα.

ΛΟΚ (εμπειρικός ιδεαλιστής, απορρίπτει τις έμφυτες ιδέες)

1. Τα 2 είδη ιδεών (ποιοτήτων) των υλικών όντων: Είναι οι πρωτεύουσες και οι

δευτερεύουσες.

Οι ποιότητες αποτελούν τις ιδιότητες πραγμάτων του εξωτερικού κόσμου. Κάθε πράγμα

έχει μια

υλική υπόσταση, όπου και στηρίζονται οι ποιότητές του, δηλαδή τα χαρακτηριστικά, οι

ιδιότητές

του. Οι πρωτεύουσες συνδέονται άρρηκτα με τα πράγματα (π.χ. η ποιότητα της έκτασης,

γιατί

δεν νοείται υλικό αντικείμενο χωρίς έκταση). Οι δευτερεύουσες δεν συνδέονται άρρηκτα με

τα

πράγματα (π.χ. κόκκινο χρώμα ενός τραπεζιού, το τραπέζι μπορεί να νοηθεί και χωρίς

αυτό).

2. Τι λέει ο Λοκ για την άγραφη πλάκα (tabula rasa); Η άγραφη πλάκα είναι η ψυχή, που

κατά

τη διάρκεια της ζωής μας γεμίζει με ιδέες (παραστάσεις του κόσμου) τις οποίες

σχηματίσαμε

μέσω της εμπειρίας μας (αισθήσεις και λογισμός). Έτσι, η γνώση είναι επίκτητη, ενώ ΔΕΝ

υπάρχουν έμφυτες ιδέες, γιατί τότε θα τις είχαν και τα μικρά παιδιά ή τα καθυστερημένα

άτομα.

Οι ιδέες είναι αντικείμενα της νόησης και αντιπροσωπεύουν τα πράγματα του εξωτερικού

κόσμου.

Page 6: ΣΗΜΕΙΩΣΕΙΣ ΦΙΛΟΣΟΦΙΑΣ I

ΜΠΑΡΚΛΕΫ (απόλυτος ιδεαλιστής, οι ιδέες είναι τα δεδομένα της εμπειρίας)

1. Το είναι ταυτίζεται με το αντιλαμβάνεσθαι: η πραγματικότητα εξισώνεται με ό,τι

αντιλαμβανόμαστε μέσω των αισθήσεων. Μπορούμε να είμαστε σίγουροι μόνο για τις

ιδέες με

τις οποίες μας τροφοδοτεί η εμπειρία μας. Για τον Μπάρκλεϋ δεν υπάρχει τίποτα πέρα ,από

τις

ιδέες. Αρνείται την ύλη και υποστασιοποιεί το πνεύμα. Ο κόσμος υπάρχει μόνο ως

αντικείμενο

του πνεύματος. Ο Μπάρκλεϋ υιοθετεί μόνο την ύπαρξη της νοητικής υπόστασης, ενώ

αρνείται

την υπόσταση της ύλης, γιατί τη συνδέει με την αθεΐα.

ΧΙΟΥΜ (εμπειριστής, βέβαιος μόνο για εμπειρίες-αισθήσεις, απορρίπτει ορθό λόγο)

1. Η θεωρία της αιτιότητας: Για το Χιουμ, η γνώση που έχουμε για τα πράγματα του κόσμου

στηρίζεται σε μια σχέση αιτίας-αποτελέσματος. Η σχέση αυτή δεν είναι αντικειμενική, αλλά

ψυχολογική, επειδή υπάρχει μια τάση στον άνθρωπο να δέχεται την κανονικότητα των

φυσικών

νόμων βάσει της συνήθειας και της παρατήρησης που έχει κάνει πολλές φορές στο

παρελθόν.

Ως εμπειριστής, ο Χιουμ δέχεται ότι αποδεκτές είναι μόνο οι ιδέες που προκύπτουν απ' την

εμπειρία και αποδεικνύονται μέσα απ' το πείραμα. Επομένως, κατ' ανάγκη αληθείς είναι οι

μαθηματικές προτάσεις και οι ταυτολογίες, ΟΧΙ οι φυσικοί νόμοι. Μόνη πηγή γνώσης είναι

ό,τι

πηγάζει από τις αισθήσεις μας. Άσκησε κριτική στον ιδεαλισμό τύπου Μπάρκλεϋ, ο οποίος

αναζήτησε στον Θεό το κριτήριο που εξασφαλίζει την αντικειμενικότητα των ιδεών

επικοινωνώντας με τον Θεό, είπε ο Μπάρκλεϋ, μπορούμε να συγκρίνουμε τις ιδέες μας με

τις

δικές Του, για να δούμε ποιες είναι αληθινές και ποιες όχι. Αυτή η θεωρία είναι άστοχη,

γιατί δεν

είναι δυνατόν οι ιδέες που διαμορφώνονται στο μυαλό μας να έχουν σχέση με τις ιδέες του

Θεού, γιατί Αυτός δεν συλλαμβάνει τίποτα με τις αισθήσεις. Για τον Χιουμ όμως, που

κατέληξε

Page 7: ΣΗΜΕΙΩΣΕΙΣ ΦΙΛΟΣΟΦΙΑΣ I

στην αποθέωση της εμπειρίας, τα πάντα πηγάζουν από την εμπειρία μας. Έτσι, απορρίπτει

τον

κόσμο των ιδεών του Θεού. Με τη θεωρία της αιτιότητας, υπονόμευσε το κύρος της

επιστήμης,

γιατί υποστήριξε ότι οι θεμελιώδεις έννοιες της επιστήμης έχουν ψυχολογική ισχύ, αφού

περιγράφουν αυτό που πιστεύουμε εμείς για την πραγματικότητα κι όχι πως είναι η

πραγματικότητα καθαυτή, ανεξάρτητα από τις δικές μας γνώμες.

ΚΑΝΤ (κριτικός ιδεαλιστής - δέχεται και ιδέες και εμπειρία)

1. Τι είναι η Κοπερνίκεια επανάσταση του Καντ; Στη γνωσιοθεωρία του, ο Καντ καινοτομεί

γιατί

δέχεται μεν τη συμβολή της εμπειρίας, αλλά εισάγει και τη συμμετοχή του εαυτού (ατομική

συμμετοχή υποκειμένου) ως παρατηρητή και συνθέτη των ετερόκλητων παραστάσεων της

εμπειρικής πραγματικότητας. Ο Παρατηρητής, με τη βοήθεια του a priori εξοπλισμού του

(μορφές χώρου, χρόνου και κατηγορίες όπως η αιτιότητα), ταξινομεί τις εμπειρίες μας και

φτάνει αντικειμενικά στη γνώση.

2. Συνθετικές a priori προτάσεις του Καντ σε σύγκριση με προτάσεις Χιουμ. Αν βασιστούμε

στις

προτάσεις του Χιουμ, δηλαδή στις αναλυτικές (αληθείς, χωρίς γνωστική αξία), και τις

συνθετικές

(με γνωστική αξία, όχι όμως κατ' ανάγκην αληθείς

3. Ποια η σχέση φαινόμενου-νοούμενου σε Καντ. Επιστημολογική ταπεινοφροσύνη. Ο Καντ

έκανε αυτή τη διάκριση για να διασφαλίσει την εγκυρότητα των παραστάσεων της

πραγματικότητας. Υποστήριξε ότι γνωρίζουμε τα φυσικά αντικείμενα με τις αισθήσεις μας.

Αν

υπήρχε όμως μόνο ό,τι αντιλαμβανόμαστε με τις αισθήσεις, τότε οι υποκειμενικές αυτές

παραστάσεις αυτές θα ήταν απατηλές. Για να το αποφύγει αυτό, εισάγει ένα κριτήριο που

διασφαλίζει την αντικειμενικότητα της υλικής Πραγματικότητας. Αυτό είναι το πράγμα

καθαυτό

(Ding-an-sih), με το οποίο όμως δεν μπορούμε να έρθουμε σε γνωσιολογική επαφή γιατί

υπάρχει πέρα από την εμπειρία μας. Και για τον Καντ, τα όρια της γνώσης μας είναι τα όρια

της

Page 8: ΣΗΜΕΙΩΣΕΙΣ ΦΙΛΟΣΟΦΙΑΣ I

εμπειρίας μας. Για τον Καντ μόνο ο Θεός έχει γνώση για το καθαυτό, επειδή το έχει

δημιουργήσει, αντίθετα με τον άνθρωπο, που δεν έχει δικαιοδοσία σε θέματα που

υπερβαίνουν

την αισθητηριακή του αντίληψη. Ο άνθρωπος δεν μπορεί επομένως να δίνει λογικές

αποδείξεις

για τη φύση της ψυχής και του Θεού (ταπεινοφροσύνη).

ΧΟΜΠΣ (ακραίος εμπειριστής στην ηθική όχι στην πολιτική, ντετερμινιστής, υλιστής)

1. Γιατί είναι ντετερμινιστής ο Χομπς; Έχει μηχανιστική αντίληψη για το σύμπαν, που τη

θεμελιώνει πάνω στην κίνηση, ενώ βλέπει τον άνθρωπο σαν αυτόματο, μια μηχανή δηλ.

που

κινείται, με τα όργανα του σώματος να παίζουν το ρόλο μηχανικών εξαρτημάτων. Σ' αυτή τη

Μηχανιστική Αντίληψη, προσθέτει μια ντετερμινιστική αντίληψη για την ανθρώπινη

συμπεριφορά. Λέει ότι η βούληση καθορίζεται αιτιακά από τα κίνητρα γι' αυτό και δεν είναι

ελεύθερη. Η ελευθερία, απ' την άλλη, σχετίζεται με τον εξωτερικό κι όχι τον εσωτερικό

κόσμο

του ανθρώπου, και περιγράφει εκείνη την κατάσταση κατά την οποία ο άνθρωπος δεν

εμποδίζεται να κάνει αυτό που θέλει από κάποιο εξωτερικό αίτιο. Ελευθερία υπάρχει στον

άνθρωπο όταν βρίσκεται στη φυσική του κατάσταση.

ΠΟΛΙΤΙΚΕΣ ΙΔΕΕΣ ΣΤΟΝ ΔIΑΦΩΤΙΣΜΟ

1. Η θεωρία του κοινωνικού συμβολαίου στο Χιουμ, Λοκ, Χομπς και Ρουσσώ.

Το κοινωνικό συμβόλαιο στον ΧΙΟΥΜ: Κατά του συμβολαίου, γιατί δεν πιστεύει ότι

μπορούμε να οργανώσουμε μια ορθολογική κοινωνία. Ως ωφελιμιστής, πιστεύει στην

αναγκαία ύπαρξη των κανόνων. Οι άνθρωποι δεν συνάπτουν συμβόλαιο για τη δημιουργία

της κοινωνίας, αλλά συναινούν σιωπηρά στην ίδρυσή της, επειδή εξυπηρετεί τους στόχους

τους. Για τον Χιουμ, για να υπάρξει οποιοδήποτε είδος κοινωνικού συμβολαίου, θα πρέπει

να εξυπηρετεί την αρχή της ωφελιμότητας. Γενικά ο Χιούμ θεωρεί παράγοντες, όπως η

ανάπτυξη του εμπορίου και της οικονομίας ως θετικούς για μια κοινωνία, γιατί

εξυπηρετούν τα συμφέροντα των πολιτών.

Ο πολιτικός αυταρχισμός του ΧΟΜΠΣ: Η φυσική κατάσταση του ανθρώπου χαρακτηρίζεται

από την επιθυμία απόκτησης δύναμης και αυτοσυντήρησης, που διασφαλίζει την επιβίωση.

Πρόκειται για μια συνεχή, εγωιστική σύγκρουση "όλων εναντίον όλων", όπου κανείς δεν

εμπιστεύεται κανέναν. Για να καταργήσουν τη δυστυχία αυτή, οι άνθρωποι φτιάχνουν από

κοινού μια πολιτική κοινωνία και εκχωρούν τα φυσικά τους δικαιώματα σε έναν τρίτο, τον

κυρίαρχο. Ο κυρίαρχος ασκεί την εξουσία του χωρίς περιορισμούς, με μόνο καθήκον την

Page 9: ΣΗΜΕΙΩΣΕΙΣ ΦΙΛΟΣΟΦΙΑΣ I

προστασία των πολιτών από τον πόλεμο και τη διασφάλιση της ειρήνης. Αν εκλείψει η

συγκεντρωτική εξουσία, η κοινωνία καταστρέφεται και οι άνθρωποι επανέρχονται στη

φυσική τους κατάσταση, δηλαδή την αναρχία. Για τον Χομπς, δεν υπάρχει ενδιάμεσο

στάδιο. Η πολιτική αυτή θεωρία του είναι αυταρχική και απολυταρχική, γιατί παραχωρεί

στον κυρίαρχο μια εξουσία χωρίς όρια και δεσμεύσεις, ενώ ο άνθρωπος είναι ένα αυτόματο

που δεν συμμετέχει στα κοινά.

Το συμβόλαιο στο ΡΟΥΣΣΩ: Βασικός εκφραστής του κοινωνικού συμβολαίου. Στη φυσική

τους κατάσταση, οι άνθρωποι είναι αθώοι, ικανοποιούν τις φυσικές τους ορμές και

βρίσκονται σε αρμονία με τη φύση. Οι άνθρωποι σταδιακά απομακρύνθηκαν από τα

φυσικά τους ένστικτα και ίδρυσαν μια κοινωνία που, δρώντας επί ίσοις όροις για όλους

τους πολίτες, θα διασφάλιζε την ισότητα και την ελευθερία τους. Η ελευθερία για τον

Ρουσσώ είναι η συμμετοχή του ατόμου στις συλλογικές αποφάσεις της κοινωνίας και η

ιδεώδης κοινωνία είναι η δημοκρατία και όχι η φιλελεύθερη, συνταγματική διακυβέρνηση

(Λοκ).

ΧΕΓΚΕΛ=ΕΓΕΛΟΣ Θεωρία της ολότητας:

O Χέγκελ θεωρεί ότι το μόνο πράγμα που έχει υπόσταση είναι το συνολικό σύμπαν. Η

ύπαρξη

κάθε πράγματος εξαρτάται από όλα τα άλλα και μπορούμε να το δούμε αποκομμένο απ'

αυτά.

Επομένως, η αυτονομία των πραγμάτων είναι πλασματική. Μόνο το σύνολο (συμπαντική

ολότητα) είναι πραγματικό και καμιά γνώση δεν μπορεί να ισχύσει έξω απ' αυτή την

ολότητα.

Έτσι, για να γνωρίσουμε ένα επιμέρους πράγμα (π.χ. έναν κόκκο άμμου), πρέπει πρώτα να

γνωρίσουμε ολόκληρο το σύμπαν! (παράλογη αντίληψη, αναιρεί την επιστημονική έρευνα).

1 . Τι είναι η ιστορική ζωή και η ιστορική σχολή του δικαίου στον Χέγκελ;

Οι αρχαίοι Έλληνες πίστευαν ότι η ιστορία είναι η κυκλική και αέναη επανάληψη μιας

σειράς

γεγονότων. Αντίθετα, ο χριστιανισμός πιστεύει στην ευθύγραμμη πορεία της ιστορίας. Την

άποψη αυτή ενστερνίστηκαν οι Διαφωτιστές και ο Χέγκελ προσθέτοντας ότι η πορεία της

ιστορίας, που δεν είναι καθόλου τυχαία αλλά δεδομένη εκ των προτέρων, έχει ένα στοιχείο

σταθερής προόδου με ένα τέλος αυτό θα είναι η κορύφωση των ανθρώπινων επιτευγμάτων

σε

όλους τους τομείς του πνεύματος (επιστήμη, τέχνη κ.λπ.). Για τον Χέγκελ, η ιστορικότητα

δεν

Page 10: ΣΗΜΕΙΩΣΕΙΣ ΦΙΛΟΣΟΦΙΑΣ I

είναι ένα οντολογικό σύμβολο, απογυμνωμένο από το κοινωνικό γίγνεσθαι. Σκοπός του

ιστορισμού του είναι να μελετήσει τα αιτήματα του λόγου, όπως είναι η ισότητα και η

ελευθερία,

εντάσσοντας τα μέσα στο ίδιο το ιστορικό γίγνεσθαι. Η ιστορική σχολή του δικαίου, τονίζει

ότι οι

νόμοι εξαρτώνται από τις τοπικές και ιστορικές ιδιαιτερότητες κάθε κοινότητας και δεν

αποτελούν καθολικές έννοιες για όλες τις κοινωνίες. Αυτό ερχόταν να ασκήσει κριτική στις

οικουμενικές πολιτικές ιδέες του 1789. Ο Χέγκελ ήθελε να αναδείξει έναν κοινό πολιτισμό,

στον

οποίο θα συμβάλλει κάθε τοπική κοινωνία τις ιδιαίτερες εμπειρίες της. Έτσι, οι έννοιες

παγκοσμιότητα και εντοπιότητα αλληλοσυμπληρώνονται για ένα κοινό πολιτισμικό

αποτέλεσμα,

που δεν αναιρεί όμως την πολιτισμική πολυμορφία.

2. Η ιστορία και ελευθερία στον Χέγκελ.

Η ιστορία για τον Χέγκελ δεν είναι η ανεξέλεγκτη βούληση του ανθρώπου που νοιάζεται για

το

συμφέρον του, αλλά η ένταξη του Εγώ του σε μια πολιτισμική ολότητα, που εξασφαλίζει

την

αναγνώριση των διαφορών του συνανθρώπου και την αποδοχή ενός συλλογικού οράματος

ζωής. Αυτό που προτάσσει η ελευθερία είναι η δημιουργία μιας λειτουργικής και δίκαιης

κοινωνίας (αν και η έννοια του δικαίου αλλάζει, ανάλογα με το πολιτισμικό και πολιτικό

υπόβαθρο κάθε κοινωνίας). Εν τέλει, ιστορία είναι μια πορεία προς την ελευθερία, καθώς

τελικός σκοπός της είναι η ελευθερία του πνεύματος.

3. Το πνεύμα του λαού και το παγκόσμιο πνεύμα στον Χέγκελ. Υπάρχουν μερικά ξεχωριστά

"έθνη" (για τον Χέγκελ, όρος με πολιτιστικό περιεχόμενο) με πολύτιμη συνεισφορά στην

ιστορία. Τα έθνη αυτά ήταν πρωτοπόρα, γιατί τα διανοητικά τους επιτεύγματα προώθησαν

προς

τα εμπρός το παγκόσμιο πνεύμα, τη συνολική δηλαδή παιδεία και πολιτισμό της

ανθρωπότητας.

Λαοί, όπως οι αρχαίοι Έλληνες, οι Γάλλοι Διαφωτιστές κ.ά. δημιούργησαν πολιτισμό για

λογαριασμό ολόκληρης της ανθρωπότητας, γι' αυτό και η δική τους κληρονομιά λαμβάνεται

Page 11: ΣΗΜΕΙΩΣΕΙΣ ΦΙΛΟΣΟΦΙΑΣ I

ιδιαίτερα υπόψη από τη φιλοσοφία και την ιστορία.

4. Σχέση κυρίου και δούλου στον Χέγκελ. Η εξέλιξη της ιστορίας γίνεται μέσα από συνεχείς

αντιπαραθέσεις θεσμών και ιδεών. Υπάρχει η ελίτ των "κυρίων", που ιδιοποιείται το μόχθο

των

"δούλων" και αντλεί την εξουσία της από την εκμετάλλευσή τους. Οι δούλοι είναι η

παραγωγική

τάξη που προμηθεύει τους κυρίους με τα υλικά και πολιτιστικά αγαθά, ενώ οι ίδιοι δεν

έχουν

κανένα δικαίωμα. Σε κάποια ιστορική στιγμή, η δύναμη των κυρίων παρακμάζει και οι ίδιοι

γίνονται ένα παρασιτικό σώμα, ενώ οι δούλοι αφυπνίζονται κοινωνικά, τους αφαιρούν την

εξουσία και γίνονται αυτοί η νέα κυρίαρχη τάξη, για να εκμεταλλευτούν με τη σειρά τους τα

κατώτερα στρώματα και να παρακμάσουν στη συνέχεια. Ενώ η μοίρα των κυρίων είναι η

ανατροπή τους, από το σώμα των δούλων προέρχεται κάθε πρόοδος της ανθρωπότητας,

γιατί,

μέσα από τη σκληρή και επίπονη εργασία τους, έχουν μάθει να αποκτούν συναίσθηση των

ικανοτήτων τους που εκμεταλλεύονται οι κύριοι. Επειδή δεν έχουν κανένα δικαίωμα, δεν

έχουν

και τίποτα να χάσουν. Όταν έρθει λοιπόν η στιγμή να παλέψουν, μπαίνουν σ' αυτό τον

αγώνα

ολόψυχα, έχοντας ανταλλάξει την καταπιεσμένη τους συνείδηση με μια άλλη, αυθύπαρκτη

και

ισχυρή. Όπως βλέπουμε, η ιστορία για τον Χέγκελ είναι μια συνεχής ανατροπή κάθε

πολιτικού

και διανοητικού κατεστημένου. Οι ιδέες αυτές επηρέασαν στη συνέχεια τον Μαρξ για τη

δομή

της κοινωνίας του.

ΦΡΕΓΚΕ

1. Διάκριση ανάμεσα στο νόημα και την έκταση.

Ο Φρέγκε αναγάγει τα μαθηματικά στη λογική και βλέπει τη γλώσσα ως ένα σύστημα

νοηματικών συναρτήσεων, ακριβώς όπως και οι μαθηματικές συναρτήσεις. Το νόημα για

τον

Page 12: ΣΗΜΕΙΩΣΕΙΣ ΦΙΛΟΣΟΦΙΑΣ I

Φρέγκε δεν έχει υποκειμενική διάσταση, αλλά ανήκει στην κοινή γλώσσα. Η διάκριση

ανάμεσα

στο νόημα μιας έννοιας και στην έκταση της εφαρμογής της κατέχει κεντρική θέση στη

φιλοσοφία των μαθηματικών του Φρέγκε. Το νόημα είναι ο τρόπος με τον οποίο ο νους

συλλαμβάνει τη φυσική πραγματικότητα. Η έκταση είναι εκείνα τα αντικείμενα που

ξεχωρίζουν

χάρις στα συγκεκριμένα χαρακτηριστικά που ορίζει το νόημα. Για παράδειγμα, ο Αυγερινός

και ο

Αποσπερίτης είναι δύο διαφορετικά νοήματα αλλά έχουν την ίδια έκταση, γιατί

αναφέρονται

στον ίδιο πλανήτη (Αφροδίτη).

ΡΑΣΕΛ

1. Ο επιστημολογικός εμπειρισμός του Ράσελ.

Ο Ράσελ τονίζει την υποκειμενικότητα της πρότασης. Στην αναλυτική θεωρία του λογικού

ατομισμού του, υποστηρίζει ότι μια πρόταση έχει νόημα όταν είναι δυνατόν να την

αναλύσουμε

σε επιμέρους προτάσεις που να τις συνδέουν λογικοί σύνδεσμοι. Επιδίωξη του Ράσελ είναι

μια

ιδεώδης γλώσσα με νοηματική καθαρότητα και αυστηρή λογική δομή, για τη δημιουργία

της

οποίας είναι απαραίτητη η γλώσσα των μαθηματικών. Η επιστημολογία του Ράσελ είναι

εμπειρική, γιατί δεν δέχεται ότι για να ορίσουμε θεωρητικά ένα αντικείμενο, θα πρέπει να

έχουμε πρώτα έρθει σε άμεση αισθητηριακή επαφή μαζί του (knowledge by acquaintance).

Ο

Ράσελ θεωρεί απαραίτητη τη λογική τακτοποίηση των προτάσεων, αλλά υποστηρίζει ότι

γίνεται

για να μπορούν στην συνέχεια να επαληθευθούν εμπειρικά.

ΒΙΤΓΚΕΝΣΤΑΙΝ (η φιλοσοφία περιορίζεται στο πλαίσιο μιας γλωσσικής άσκησης και

μόνο - η μονομερής ενασχόληση με το νόημα της γλώσσας μπορεί να οδηγήσει σε

ανούσιες φλυαρίες - δεν μπορούμε να περιορίσουμε τη φιλοσοφία σε

γλωσσαναλυτικές κατευθύνσεις μόνο).

Page 13: ΣΗΜΕΙΩΣΕΙΣ ΦΙΛΟΣΟΦΙΑΣ I

1. Διαφορά αριστοτελικής και σύγχρονης λογικής στον Βιτγκενστάιν: Η παλιά (αριστοτελική)

λογική υποστήριζε την ύπαρξη σταθερών και αμετάβλητων οντολογικών κατηγοριών όπου

εντάσσονται όλα τα είδη του φυσικού κόσμου. Η αναλυτική φιλοσοφία διαμόρφωσε τη νέα

λογική, η οποία απέρριψε τις ολιστικές και τελολογικές θεωρήσεις του παρελθόντος και

σταμάτησε να ασχολείται με το ΟΛΟΝ της αριστοτελικής μεταφυσικής, υποστηρίζοντας τη

θέση

ότι η πραγματικότητα αποτελείται από μεμονωμένα γεγονότα. Τα ατομικά αυτά συμβάντα

περιγράφονται από προτάσεις, τα συστατικά των οποίων ενώνουν λογικοί σύνδεσμοι.

Στόχος της νέας λογικής των αναλυτικών φιλοσόφων είναι η επίλυση των φιλοσοφικών

προβλημάτων μέσα από την ανάλυση της γλώσσας. Οι αναλυτικοί απεχθάνονταν τη

μεταφυσική

και την ταύτιζαν με την πλάνη και το ψεύδος. Δεν ενδιαφέρονται για το πώς

δημιουργήθηκε ο

κόσμος γιατί πιστεύουν ότι η φιλοσοφία δεν πρέπει ν' ασχολείται με τη δομή του κόσμου.

2. Τι είναι α-νοησία; Η σιωπή στον Βιτγκενστάιν; Ποια είναι η απεικονιστική θεωρία του

νοήματος του Βιτγκενστάιν;

Στην απεικονιστική θεωρία του νοήματος, ο φιλόσοφος επιχείρησε να εξηγήσει τους

κινδύνους

που συνεπάγεται η κακή χρήση της γλώσσας, όταν δηλαδή δεν διατυπώνουμε σωστά ό,τι

πρέπει

να λεχθεί ή διατυπώνουμε αυτό που δεν πρέπει να λεχθεί με αποτέλεσμα να λέμε

ανοησίες.

3. Επιχείρημα Βιτγκενστάιν κατά της ιδιωτικής γλώσσας. Η γλώσσα για τον Βιτγκενστάιν έχει

δημόσιο κι όχι ιδιωτικό χαρακτήρα. Είναι εξ ορισμού ένα δημόσιο σύστημα, μια συλλογική

πραγματικότητα, την οποία δεν μπορεί να ιδιοποιηθεί η εσωτερική συνείδηση. Οι

άνθρωποι

υποστηρίζουν το αντίθετο επειδή συγχέουν τα ψυχολογικά τους συναισθήματα με τις λέξεις

που

χρησιμοποιούν για να τα δηλώσουν. Ιδιωτικά είναι τα συναισθήματα και οι σκέψεις μας

(ψυχολογική γλώσσα), όχι όμως οι λέξεις που χρησιμοποιούμε για να τα εκφράσουμε

(φυσική

Page 14: ΣΗΜΕΙΩΣΕΙΣ ΦΙΛΟΣΟΦΙΑΣ I

γλώσσα). Για τον Βιτγκενστάιν, φυσική και ψυχολογική γλώσσα είναι δύο διαφορετικά

γλωσσικά

παίγνια. Η σύγχυση προέρχεται από τον λανθασμένο τρόπο χρήσης των λέξεων. Εξάλλου, η

αποκατάσταση της σωστής χρήσης της γλώσσας είναι και ο ορισμός της φιλοσοφίας του

Βιτγκενστάιν.

4. Το νόημα ως χρήση κατά της ουσιοκρατίας στον Βιτγκενστάιν. Με τη θεωρία του

νοήματος

κατά τη χρήση του, η γλώσσα εξελίσσεται συνεχώς, σύμφωνα με τις αλλαγές του

πολιτιστικού

γίγνεσθαι. Δέχεται δηλαδή νέα νοήματα και όχι μόνο ένα και αιώνιο νόημα (ουσία). Ο

Βιτγκενστάιν αναιρεί έτσι την ουσιοκρατία, έναν τρόπο σκέψης που αναζητούσε ένα μόνο

νόημα, που να ισχύει πάντα. Για τον Βιτγκενστάιν τα νοήματα παράγονται μέσα από μια

πολύμορφη επικοινωνία και πρακτική, έτσι όπως την ορίζουν οι στόχοι κάθε κοινωνίας. Δεν

υπάρχουν κανόνες που να είναι δεδομένοι εκ των προτέρων. Οι κανόνες κατανοούνται

πάντα εκ

των υστέρων, αφότου έχουμε παίξει το γλωσσικό παιχνίδι.

ΧΟΥΣΣΕΡΛ (δεν μπορούμε να τα αναγάγουμε όλα στην πρωτογενή αυθορμησία του

συλλογικού υποκειμένου)

1. Τι είναι φαινομενολογία; Ο όρος ανήκει στον Χέγκελ και δηλώνει τα στάδια από τα οποία

περνάει η συλλογική συνείδηση για να κατατάσσει τη γνώση του κόσμου. Φαινόμενα είναι

οι

διάφοροι κοινωνικοί και ηθικοί θεσμοί που αντανακλούν την ωριμότητα κάθε συλλογικής

ανθρώπινης συνείδησης. Στη φαινομενολογία του Χούσσερλ, η συνείδηση καθορίζεται

ανεξάρτητα από την εμπειρία (ή πραγματικότητα). Μ' άλλα λόγια, η συνείδηση δεν είναι

μέρος

του φυσικού ή κοινωνικού συστήματος, αλλιώς οι λειτουργίες της (π.χ. η λογική) χάνουν την

αυτονομία τους και υπακούουν αναγκαστικά στις φυσικές ή άλλες επιταγές.

2. Η έννοια της εποχής και η καθαρή συνείδηση του Χούσσερλ. Στη φαινομενολογία του

Χούσσερλ, η ρήξη μεταξύ του εμπειρικού κόσμου και του Εγώ είναι πλέον σαφής. Η

Page 15: ΣΗΜΕΙΩΣΕΙΣ ΦΙΛΟΣΟΦΙΑΣ I

υπερβατολογική συνείδηση "ξεχνάει" τον εμπειρικό κόσμο (τον οποίο βάζει μέσα σε

αγκύλες)

και αναδιπλώνεται στον εαυτό της. Έτσι, οι απόψεις του Εγώ για τον εξωτερικό κόσμο

βασίζονται σε εσωτερικούς στοχασμούς και όχι στην εμπειρική παρατήρηση. Η

φαινομενολογική

εποχή του Χούσσερλ δεν αποφαίνεται λοιπόν για την ύπαρξη των εμπειρικών πραγμάτων.

Αυτό

όμως δεν σημαίνει ότι αρνείται τον φυσικό κόσμο, αλλά δεν τον μελετάει με τις μεθόδους

των

λογικών και εμπειρικών επιστημών.

3. Η φαινομενολογία ως ειδητική επιστήμη στον Χούσσερλ. Η φαινομενολογία είναι

ειδητική

επιστήμη γιατί ασχολείται με το νόημα των λογικών ειδών.

ΣΑΡΤΡ επινόησε τον όρο υπαρξισμός - οι ρίζες της σκέψης του στη φαινομενολογία.

1. Τι είναι ναυτία, γιατί υπάρχει; Διαχωρισμός του "είναι" για τον εαυτό του και στον

εαυτόν για

τον Σαρτρ. Ναυτία είναι η κατάσταση κατά την οποία το υπαρκτικό εγώ βρίσκεται σε έντονη

αντίθεση με το συμπαντικό Είναι. Η σχέση ανάμεσα σ' αυτούς τους δύο πόλους είναι μια

απόλυτα αντιθετική σχέση. Το συμπαντικό Είναι προκαλεί στο υπαρκτικό εγώ μια

απώθηση, μια

ναυτία, γιατί το εμποδίζει να υπάρξει "για τον εαυτό του". Για τον Σαρτρ, η ατομική

συνείδηση

υπάρχει ανεξάρτητα από κάθε πολιτιστικό και κοινωνικό σύστημα και μάλιστα έρχεται σε

σύγκρουση μ' αυτά. "Η ύπαρξη έχει προτεραιότητα από την ουσία". Αυτό δίνει στο Εγώ μια

χωρίς όρια ελευθερία να πράξει, να "λερώσει τα χέρια του", προκειμένου να δομήσει

αυτόβουλα

τον εαυτό του, ανεξάρτητα από τους μηχανισμούς της εξουσίας. Η ανθρώπινη ύπαρξη είναι

η

μόνη υπεύθυνη για τις πράξεις της και πορεύεται στον κόσμο "εν τη απουσία του Θεού",

όπως

υποστήριξε και ο Χάιντεγκερ. Για τον Σαρτρ, δεν υπάρχει νομοτέλεια κάποιου είδους (φύση,

Page 16: ΣΗΜΕΙΩΣΕΙΣ ΦΙΛΟΣΟΦΙΑΣ I

κράτος) και το υπαρξιακό υποκείμενο κάνει μόνο του τις επιλογές του. Το μόνο κριτήριο

που

δίνει είναι η συναισθηματική ειλικρίνεια. "Εμπιστεύσου το συναίσθημά σου", λέει

χαρακτηριστικά.

2. Τι είναι κακή πίστη στον Σαρτρ; Στην κακή πίστη, κρύβω την αλήθεια από τον εαυτό μου,

αλλά έχω πλήρη επίγνωση ότι λέω ψέματα. Μ' αυτό τον τρόπο, αρνούμαι την ελευθερία

που

μου δίνει η πλήρης ευθύνη για τις πράξεις μου. Δεν δέχομαι δηλαδή την ευθύνη γι' αυτές,

αλλά

τις αποδίδω σε άλλες αιτίες, όπως τον Θεό, το κοινωνικό και ηθικό κατεστημένο κ.λπ. Μ'

αυτό

τον τρόπο όμως δεν μπορώ να προοδεύσω ως ύπαρξη.

3. Η σχέση του υπαρξισμού του Σαρτρ με τον κομμουνισμό. Κατά τη μεταπολεμική περίοδο,

ο

Σαρτρ ενσωμάτωσε στις θεωρίες του την κοινωνική πραγματικότητα, όπως την είχε

συλλάβει ο

Μαρξ. Δικαιολόγησε την ένταξη του υπαρξιακού υποκειμένου στον αγώνα για μια αταξική

κοινωνία και την υιοθέτηση του προλεταριακού ιδανικού, λέγοντας ότι μόνο έτσι θα

μπορούσε

να κατακτήσει την ελευθερία του. Η πολιτική δέσμευση του υποκειμένου όμως, γίνεται

μόνο

επειδή το θέλει ΤΟ ίδιο και έτσι μπορεί να πάψει ανά πάσα στιγμή. Πρωταρχικό του

καθήκον

παραμένει η αποδέσμευση από τους μηχανισμούς της εξουσίας. Επειδή η πολιτική αυτή

δέσμευση του Σαρτρ στο μαρξιστικό ιδεώδες δεν ήταν πειθαρχημένη και συνειδητή,

οδηγήθηκε

στο τέλος στη ρήξη.

ΕΠΟ 22 ΙΣΤΟΡΙΑ ΤΗΣ ΦΙΛΟΣΟΦΙΑΣ ΣΤΗΝ ΕΥΡΩΠΗ

ΣΗΜΕΙΩΣΕΙΣ ΦΙΛΟΣΟΦΙΑΣ II

Ο ΚΑΝT ΚΑΙ Η ΣΥΝΘΕΤΙΚΗ ΕΚ ΤΩΝ ΠΡΟΤΕΡΩΝ ΓΝΩΣΗ

Η επιστημολογία του μέσω της "Κριτικής του καθαρού λόγου" αποπειράται να διασώσει το

Page 17: ΣΗΜΕΙΩΣΕΙΣ ΦΙΛΟΣΟΦΙΑΣ I

νευτώνειο σύστημα από την αμφιβολία που το μάστιζε θεμελιακά. Ήταν μια αντιστροφή

γνωσιολογικής οπτικής, η οποία θα διέσωζε την αιτιότητα μα με τις έννοιες του χώρου και

του

χρόνου που είχαν όπως διαπιστώθηκε σχετικοποιηθεί από τον Λάϊμπνιτζ από τη "Χιουμιανή

αμφισβήτηση". Η αφετηρία της είναι ο απόλυτος διαχωρισμός ανάμεσα στην ψυχολογία

και στην

επιστημολογία. Η αιτιακή σχέση δεν είναι μια επαγωγική γενίκευση, δηλαδή ένα εμπειρικό

συμπέρασμα βασισμένο στην εξακολουθητική παρατήρηση της διαδοχής των φαινομένων.

Πρέπει να θεωρηθεί μια νοητική μορφή η κατηγορία, ένας καθαρός τύπος του σκέπτεσθαι,

ο

οποίος βρίσκεται μέσα στο λογικό a - priori. Οι αιτιακές εξηγήσεις στη φυσική συνιστούν

γνώση

a - priori συνθετική. Πρόκειται για θεωρητικά πρότυπα που θέτει το λογικό, με την

βεβαιότητα

όμως ότι κάθε δυνατή εμπειρία θα τα επαληθεύει. Ο επιστημονικός νους γνωρίζει εκ των

πρότερων ότι κάθε εμπειρία που θα αποκομίσει από την επαφή του με το φυσικό

περιβάλλον θα

έχει αιτιακή δομή. Η γνώση βέβαια από μόνη της είναι κενή. Θα πρέπει να έλθει στη

συνέχεια η

παρατήρηση για να διδάξει το πραγματικό σχήμα αυτών των διασυνδέσεων αιτιακού

χαρακτήρα.

Η "Καντιανή θεωρία της επιστήμης" ήταν μια προσπάθεια να εξουδετερωθεί ο

σκεπτικισμός

χωρίς να αμφισβητηθεί το γνωσιακό υπόβαθρο. Είναι ένα πάντρεμα ορθολογισμού και

εμπειρισμού. Βασιζόταν σε δυο ουσιώδεις παραδοχές που αργότερα αποδείχτηκαν

λανθασμένες.

Πρώτο ότι η νευτώνεια κοσμολογία αποτελούσε το τελικό στάδιο της φυσικής σκέψης και

δεύτερο ότι η τυπική "Ευκλειδιακή λογική" ήταν επίσης ο μοναδικός και αλάνθαστος

τρόπος

λογικού στοχασμού που αν καταλυθεί αποβάλλει ο νους την ίδια τη λογικότητα του.

ΜΙΛΛ

Page 18: ΣΗΜΕΙΩΣΕΙΣ ΦΙΛΟΣΟΦΙΑΣ I

Η θεωρία της επιστήμης κατά τον 19ο αιώνα εξελίσσεται θετικιστικά. Συνεχίζει την

προσπάθεια

της αγγλοσαξονικής επιστημολογίας να ορίσει την επιστημονική γνώση ως παράγωγο της

συστηματικής εμπειρικής έρευνας και να αντιμετωπίσει τα παράδοξα. Ο Μιλλ χτίζει πάνω

στην

επαγωγική γνωσιολογία του Χιουμ προχωρώντας όμως στο αδύνατο κατά αυτόν στην

επαγωγική δηλαδή απόδειξη της ίδιας της αρχής της επαγωγής. Το ότι κάθε αποτέλεσμα

(εμπειρικό φαινόμενο) έχει το αίτιο του, καθώς και ότι αυτή η αιτία παράγει πάντα το αυτό

αποτέλεσμα είναι το συμπέρασμα μιας άπειρης σειράς παρατηρήσεων που έχουν αποτεθεί

στη

συλλογική ανθρώπινη μνήμη. Η ανθρώπινη σκέψη βαίνει πάντοτε από το μερικό προς το

γενικό.

Καμία έννοια δεν έχει ενορατικό υπόβαθρο. Επειδή γνωρίζουμε από την εμπειρία του

παρελθόντος ότι η επαγωγική γενίκευση αποφέρει θεωρητικές έννοιες (ακόμα και των

μαθηματικών) που επιτρέπουν τον έλεγχο της πραγματικότητας και την προδιαγραφή του

μέλλοντος, εξακολουθούμε να στηριζόμαστε μεθοδολογικά σ' αυτήν. Μέσω τεσσάρων

βασικών

συγκριτικών μεθόδων ο Μιλλ αντιπαραβάλλει τα πειραματικά ευρήματα.

ΕΠΑΓΩΓΙΚΗ ΜΕΘΟΔΟΣ ΤΗΣ ΔΙΑΦΟΡΑΣ

Η πιο σημαντική επαγωγή μέθοδος είναι της "διαφοράς". Συλλέγουμε ικανό αριθμό από

εμπειρικά παραδείγματα παρουσίας ενός φαινομένου και παραλλήλως παραδείγματα

απουσίας

του. Εάν οι περιβάλλουσες φυσικές συνθήκες είναι κοινές και στις δυο περιπτώσεις εκτός

από

μια τότε συμπεραίνουμε ότι η συνθήκη που λείπει αποτελεί την αιτία η μέρος της που

προκαλεί

το υπό εξέταση φαινόμενο.

Ο ΚΥΚΛΟΣ ΤΗΣ ΒΙΕΝΝΗΣ =ΛΟΓΙΚΟΣ ΘΕΤΙΚΙΣΜΟΣ=ΕΜΠΕΙΡΙΚΟΣ ΘΕΤΙΚΙΣΜΟΣ

Η Αφετηριακή παραδοχή του λογικού θετικισμού (εμπειρισμός) είναι, κατά τη διατύπωση

του R.Carnap "ότι η εκ των προτέρων συνθετική γνώση είναι αδύνατη" με δυο στόχους έναν

φιλοσοφικό και έναν πολιτικό. Έρχεται κατ' αρχήν σε αντίθεση με την επιστημολογία του

Page 19: ΣΗΜΕΙΩΣΕΙΣ ΦΙΛΟΣΟΦΙΑΣ I

Νεοκαντιανισμού. Αφού αντιπαρατέθηκε στις μεγάλες ολοκληρωτικές ιδεολογίες είναι

ταυτόχρονα εξοπλισμένος με ένα περίπλοκο τεχνικό σύστημα λογικής ανάλυσης των

επιστημονικών όρων και θεωριών, που φωτίζει τη διαπλοκή τους με τα δεδομένα της

εμπειρίας.

Πρόκειται για μια γνωσιοθεωρία που μετατοπίζει την προβληματική από την έννοια της

αλήθειας

σε εκείνη του νοήματος. Το βασικό επιχείρημα είναι ότι πριν γίνει δυνατόν να αποδειχτεί

πειραματικά εάν μια θεωρία είναι αληθής η ψευδής πρέπει να βγάζει νόημα.

ΑΡΧΗ ΤΗΣ ΕΠΑΛΗΘΕΥΣΙΜΟΤΗΤΑΣ

Η λογική προϋπόθεση γι' αυτό είναι να μας υποδεικνύει εκείνο το συστατικό της εμπειρίας

που

θα ήταν δυνατό να την επαληθεύσει η να τη διαψεύσει (αρχή της επαληθευσιμότητας).

Α-ΝΟΗΣΙΑ

Μια άνευ νοήματος α-νόητη πρόταση είναι εκείνη που δεν μας καθοδηγεί σχετικά με το τι

εμπειρικές δραστηριότητες έπονται από τους ισχυρισμούς που διατυπώνει. Μια α-νοησία

είναι

επομένως εκ τω πρότερων συμβατή με οποιαδήποτε εμπειρική κατάσταση και την αντίθετη

της.

(Μη δυνατός ο πειραματικός έλεγχος). Η θεωρία που δεν θίγεται είναι κενή νοήματος και

χωρίς

επιστημονική αξία. Από αυτή τη θέση η ιδεολογία είναι το κοσμικό υποκατάστατο της

θεολογίας.

Το κύριο σώμα της παραδοσιακής φιλοσοφίας είναι μια παρατεταμένη μεταφυσική

ανοησία.

ETHICAL EMOTIVISM

Οι προτάσεις και οι θεωρίες της ηθικής φιλοσοφίας αποτελούν μια ειδική κατηγορία. Δεν

είναι

ούτε αληθείς ούτε ψευδείς επειδή δεν περιγράφουν εμπειρικά η αντικειμενικά γεγονότα. Ο

Χιουμ είχε ήδη επιχειρηματολογήσει ότι το δέον δεν εξάγεται από το είναι. Μια ηθική

πρόταση

εκφράζει απλώς την συναισθηματική η ψυχολογική αντίδραση του παρατηρητή, την

αρέσκεια η

Page 20: ΣΗΜΕΙΩΣΕΙΣ ΦΙΛΟΣΟΦΙΑΣ I

απαρέσκεια του απέναντι σε ένα αντικειμενικό πράγμα η γεγονός χωρίς καμία ηθική χροιά.

ΧΑΡΑΚΤΗΡΙΣΤΙΚΑ

Ένα άλλο χαρακτηριστικό του λογικού θετικισμού είναι η θέση του για την μεθοδολογική

ενότητα της επιστήμης. Αυτό σημαίνει ότι το πρότυπο για την κατασκευή θεωρίας καθώς

και

επιμέρους εξηγήσεων που εξάγεται από τις φυσικές επιστήμες ισχύει και για την ιστορία

και για

τις κοινωνικές επιστήμες γενικά. Η κοινωνιολογία η ψυχολογία και η ιστορία διαφέρουν

από τις

άλλες θετικές επίσημες μόνο ως προς το αντικείμενο τους όχι ως προς τη μέθοδο.

ΓΛΩΣΣΙΚΗ ΣΤΡΟΦΗ ΣΤΗ ΦΙΛΟΣΟΦΙΑ

Η λογική ανάλυση της επιστημονικής γλώσσας ήταν η κύρια συνεισφορά του λογικού

θετικισμού. Η γλώσσα της επιστήμης αποτελείται από δυο ειδών όρους: τους

παρατηρησιακούς

και τους θεωρητικούς. Οι παρατηρησιακοί όροι είναι εκείνοι που παράγονται ευθέως μέσα

από

τη εμπειρική παρατήρηση και συνδέονται με το εμπειρικό υπόβαθρο της θεωρίας μέσω

"κανόνων αντιστοιχίας". Οι τελευταίοι αποτελούν και την άγκυρα που δένει μια θεωρία με

τον

φυσικό περίγυρο. Οι θεωρητικοί όροι, αντίθετα είναι αφηρημένες έννοιες , οι οποίες δεν

είναι

δυνατόν να υποβληθούν σε άμεση πειραματική επιβεβαίωση, απαιτούνται όμως για να

ολοκληρωθεί η λογική δομή μιας θεωρίας. Ο Λογικός θετικισμός έχει συναίσθηση του

γεγονότος

ότι η θεωρία υπερβαίνει το εκάστοτε διαθέσιμο σώμα παρατηρήσεων (επιστημολογία του

Μιλλ).

Η απαγωγή και η λογική διερεύνηση της θεωρίας αφορά τη λειτουργία των υποθέσεων

στην

πειραματική διαδικασία.

ΠΛΑΙΣΙΟ ΤΗΣ ΑΝΑΚΑΛΥΨΗΣ

Για μια συγκεκριμένη ερευνητική προσπάθεια χρειάζεται εκ των πρότερων μια

καθοδηγητική

Page 21: ΣΗΜΕΙΩΣΕΙΣ ΦΙΛΟΣΟΦΙΑΣ I

ιδέα σχετικά με τις δυνητικές εξηγήσεις του φαινομένου που μελετάται. Με πρωτεύοντα

ρόλο τη

φαντασία και την ενορατική δυνατότητα του ερευνητή η καθοδηγητική υπόθεση εργασίας

πρέπει να είναι διατυπωμένη με τέτοιο τρόπο ώστε να έχει πειραματικά ελεγχόμενες

συνέπειες.

Η αναγνώριση κάθε φορά ερευνητικών υποθέσεων αποτελεί το λεγόμενο πλαίσιο της

ανακάλυψης της επιστημονικής διαδικασίας χωρίς λογικούς κανόνες και τεχνικές για την

απομόνωση τους.

ΠΛΑΙΣΙΟ ΤΗΣ ΕΠΙΚΥΡΩΣΗΣ

Η πειραματική διαδικασία έρχεται στη συνέχεια να στηρίξει η να υπονομεύσει τις αρχικές

μας

υποθέσεις . Οι εμπειρικές τους συνέπειες "βασανίζονται" πειραματικά στο εργαστήριο, και

μόνο

μέσα από αυτή τη διαδικασία προκύπτει επιστημονική γνώση. Πρόκειται για το πλαίσιο της

επικύρωσης όπου τίθενται σε ισχύ οι τεχνικές και οι κανόνες του έλεγχου που είναι κοινά

αποδεκτοί και δεσμεύουν όλη την επιστημονική κοινότητα οπότε παύει πλέον και η

αυτόνομη

λειτουργία της επιστημονικής φαντασίας.

ΤΕΛΙΚΟ ΚΡΙΤΗΡΙΟ ΤΗΣ ΘΕΩΡΙΑΣ

Αυτό εξακολουθεί να είναι για το κύρος της η σχέση της σε τελική ανάλυση με την

αντικειμενική

πραγματικότητα η οποία υφίσταται αφεαυτής έξω από το λογικό μας. Για αν καταστεί

δυνατό

αυτό η θεωρία πρέπει να έχει μια εσωτερική λογική δομή που να επιτρέπει μια σειρά από

συμπεράσματα από τις βασικές της παραδοχές. Η απαγωγική δομή των θεωριών είναι

κρίσιμο

προαπαιτούμενο για την πειραματική επαλήθευση τους. Την παραπάνω προβληματική

αποτυπώνει το υποθετικό-επαγωγικό μοντέλο των επιστημονικών εξηγήσεων που

επεξεργάστηκε ο Hempen που εκφράζει την ωριμότητα του θετικισμού ως φιλοσοφικού

κινήματος..

ΠΑΡΑΔΟΞΑ ΤΗΣ ΕΠΙΒΕΒΑΙΩΣΗΣ

Page 22: ΣΗΜΕΙΩΣΕΙΣ ΦΙΛΟΣΟΦΙΑΣ I

Ωστόσο αναδείχτηκαν βασικά εσωτερικά προβλήματα στη σύλληψη του λογικού

εμπειρισμού Το

πρώτο παράδοξο ήταν η λογική πιθανότητα ενός καθολικού νόμου (όχι της βαρύτητας) να

είναι

πολύ κοντά στο μηδέν .Ο λόγος είναι ότι ένας τέτοιος νόμος αναφέρεται σε κάθε δυνατή

εμπειρία ενώ τα πειραματικά ευρήματα που τον επιβεβαιώνουν αντλούν από ένα

απειροελάχιστο

σώμα πειραματικών δεδομένων σε σχέση με το άπειρο σύμπαν. Το άλλο παράδοξο είναι

ότι κάθε

πρόταση καθολικού κύρους (π.χ. όλα τα κοράκια είναι μαύρα) επιβεβαιώνεται από

εμπειρικές

παρατηρήσεις που είναι άσχετες εντελώς με αυτήν όπως π.χ. από ένα κόκκινο αυτοκίνητο.

Και

αυτό γιατί η πρόταση "όλα τα κοράκια είναι μαύρα" είναι λογικά ισοδύναμη με τη πρόταση

"όλα

τα μη κοράκια είναι μη μαύρα".

ΑΣΥΜΜΕΤΡΙΑ ΛΟΓΙΚΩΝ Κ ΓΝΩΣΤΙΚΩΝ ΔΥΝΑΜΕΩΝ

Ο Λογικός θετικισμός δεν είχε κατορθώσει τελικά να επιλύσει το βασικό επιστημολογικό

πρόβλημα που είχε αναγνωρίσει η θεωρία της επιστήμης τόσο του Χιουμ όσο και του Καντ

(χωρίς να επιλύεται). Και το πρόβλημα αυτό είναι ακριβώς η ασυμμετρία ανάμεσα στις

λογικές

και γνωστικές δυνάμεις του ανθρώπινου υποκείμενου και στο αντικείμενο με το οποίο

ασχολείται. Στην προκειμένη περίπτωση η συνολική φύση.

ΥΠΕΡΒΑΣΗ ΤΟΥ ΘΕΤΙΚΙΣΜΟΥ. ΕΠΙΣΤΡΟΦΗ ΣΤΗΝ ΙΣΤΟΡΙΑ ΤΗΣ ΕΠΙΣΤΗΜΗΣ

Στο θεωρητικό οικοδόμημα παρατηρούνται σημαντικές ρωγμές. Μια ογκούμενη

αμφισβήτηση

καθιστά σαφή τα όρια του και προοιωνίζεται τις νέες αναζητήσεις που θα σημαδέψουν τη

μεταπολεμική φιλοσοφία της επιστήμης. Η καίρια μεθοδολογική πρόταση του θετικισμού

είναι το

αίτημα για την αμφιμονοσήμαντη αντιστοίχιση της θεωρίας με τα εμπειρικά δεδομένα. Η

θεωρία

συνοψίζει τα αποτελέσματα της παρατήρησης η ανάγεται καθ' οιονδήποτε τρόπο στα

Page 23: ΣΗΜΕΙΩΣΕΙΣ ΦΙΛΟΣΟΦΙΑΣ I

παρατηρησιακά δεδομένα ο δε βαθμός της επιστημονικής της αξιοπιστίας (ο βαθμός

επαλήθευσης η επιβεβαίωσης) είναι συνάρτηση του πλήθους των πειραματικών

ευρημάτων τα

οποία συμφωνούν με τις προβλέψεις που πηγάζουν από αυτήν. Κάθε θεωρητική υπόθεση

είναι

είτε μια επαγωγική γενίκευση της οποίας η πιθανότητα εξαρτάται από τη ποσότητα των

εμπειρικών μαρτυριών που μας παρέχουν την (μεθοδολογική, ορθολογική) δικαιολογία να

τη

πιστέψουμε είτε, εναλλακτικά μια πρόταση η οποία αν και δεν εξάγεται επαγωγικά

εντούτοις

δύναται τελικά να συσχετιστεί με τα αποτελέσματα της πειραματικής έρευνας. Τόσο το

νόημα

όσο και η αλήθεια των θεωριών προσδίδονται σε τελική ανάλυση από τα κάτω προς τα

πάνω.

ΣΤΟΙΧΕΙΑ ΜΗ ΙΣΤΟΡΙΚΗΣ ΑΠΟΔΟΣΗΣ

Είχε γίνει προφανές ότι η επιστημονική μεθοδολογία όπως την είχε αναπαραστήσει ο

λογικός

θετικισμός ελάχιστη σχέση είχε με το πώς οι ίδιοι οι επιστήμονες αντιλαμβάνονται τη

λειτουργία

της θεωρίας στην εμπειρική ερευνά.

Ούτε απέδιδε την ιστορική πραγματικότητα, τον τρόπο δηλαδή με τον οποίο η

επιστημονική

γνώση διαμορφώθηκε και εξελίχθηκε από την πρώιμη νεωτερικότητα. ΤΡΙΑ είναι τα βασικά

στοιχεία στη διαπίστωση αυτή. Το πρώτο έχει να κάνει με την αδυναμία του άτεγκτου

επαγωγισμού των θετικιστών να θεμελιώσουν την ορθολογικότητα των καθολικών φυσικών

νόμων, που αποτελούν το θεμέλιο κάθε λειτουργικής θεωρίας σχετικά με τη δομή του

κόσμου.

Η δεύτερη κρίσιμη ανεπάρκεια της θετικιστικής μεθοδολογίας σχετίζεται με τη δραστική

υποβάθμιση του καθαρά θεωρητικού τμήματος κάθε ολοκληρωμένης φυσικής θεωρίας,

των

όρων δηλαδή και των παραδοχών εκείνων που είναι ακριβώς αδύνατον να συσχετισθούν

με τα

Page 24: ΣΗΜΕΙΩΣΕΙΣ ΦΙΛΟΣΟΦΙΑΣ I

διαθέσιμα αισθητηριακά δεδομένα. Το τρίτο μειονέκτημα του κλασικού θετικισμού - και

πιο

κρίσιμο ίσως με την έννοια ότι αφορά όχι μόνο τις φυσικές αλλά με ιδιαίτερη ένταση και τις

κοινωνικές επιστήμες - σχετίζεται με την κατανόηση της αντικειμενικότητας και της

απροκαταληψίας στη πειραματική ερευνά. Η βασική παραδοχή εδώ (από το Μπέικον

ακόμα)

είναι ότι η φύση μιλάει από μόνη της και πρέπει να την ακούσουμε. Να πει την ιστορία της

τη

φυσική της ιστορία.

Χωρίς κάποια υπόθεση εργασίας που προσανατολίζει εκ των πρότερων τον ερευνητή προς

το

πεδίο εκείνο της φυσικής περιοχής όπου αναμένει να πορισθεί κρίσιμα εμπειρικά

ευρήματα αλλά

και τον προϊδεάζει σχετικά με την πιθανή λύση του προβλήματος που ερευνά δεν είναι

δυνατόν

να οργανωθεί η πειραματική διαδικασία. Για να κατανοηθεί και η πιο στοιχειώδης

εμπειρική

πρόταση προαπαιτείται μια περίπλοκη θεωρητική κατανόηση των λογικών κατηγοριών

διάμεσου

των οποίων γίνεται αντικείμενο της γνωστικής συνείδησης. Η παρατήρηση είναι

διαποτισμένη

από θεωρία η ότι η θεωρία υποκαθορίζεται από τα εμπειρικά δεδομένα.

Ο ΜΠΕΙΚΟΝ ΚΑΙ ΤΟ ΙΔΕΩΔΕΣ ΤΗΣ ΕΠΑΓΩΓΗΣ

ΕΜΠΕΙΡΙΚΗ ΕΠΑΓΩΓΗ

Ο στοχαστής που πρώτος σαλπίζει την έναρξη της νέας επιστημονικής εποχής είναι ο

Άγγλος Φράνσις Μπέικον (Francis Bacon, 1561-1626). Ο Μπέικον υπήρξε όχι τόσο

επιστήμονας ο ίδιος, αλλά ένας ποιητής της επιστημονικότητας, ο καλλιεπής κήρυκας της

νέας

γνωστικής μεθόδου, την οποία ταυτίζει με τη συστηματική παρατήρηση. Η συλλογή μιας

επαρκούς βάσης εμπειρικών δεδομένων καθιστά δυνατή τη θεωρητική γενίκευση, που μας

δίνει

την αλήθεια για τη φυσική πραγματικότητα. Πρόκειται για τη μέθοδο της εμπειρικής

Page 25: ΣΗΜΕΙΩΣΕΙΣ ΦΙΛΟΣΟΦΙΑΣ I

επαγωγής (induction), η οποία χτίζει το γενικό (την αιτιακή δηλαδή εξήγηση μιας ολόκληρης

κατηγορίας φαινομένων) πάνω σε ένα υπόβαθρο συγκεκριμένων και επιμέρους

παρατηρήσεων.

Η εφαρμογή της μεθόδου αυτής προϋποθέτει την κατηγορηματική απόρριψη των

μεθοδολογικών παραδοχών που κληρονομήθηκαν από τον αριστοτελισμό. Ο Μπέικον

υπήρξε

αδιάλλακτος πολέμιος των κλασικών αντιλήψεων για την επιστήμη, οι οποίες δίνουν το

προβάδισμα στην τεχνική του συλλογισμού, στη διαπλοκή δηλαδή λεκτικών τύπων κενών

εμπειρικού περιεχομένου. Η αριστοτελική λογική είναι κατ' αυτόν ένα περίτεχνο φραστικό

οικοδόμημα χωρίς στήριγμα στην απτή εμπειρία, ένα πάθος για αφαίρεση που παράγει τη

φύση

από τις ιδέες, από τις "προλήψεις του λογικού" (anticipations of the mind), και όχι

αντιστρόφως.

ΣΤΟΧΟΣ

Ο πραγματικός στόχος της διανοητικής δραστηριότητας του ανθρώπου δεν είναι η

περίτεχνη διαπλοκή λέξεων, αλλά ο έλεγχος πάνω στη φύση. Η γνώση καθαυτή είναι

άχρηστη εάν δεν εξαργυρώνεται με ισχύ (power) επάνω στα πράγματα προς όφελος της

κοινωνίας. Το πιο περίφημο ίσως απόφθεγμα του Μπέικον είναι αυτή η ταύτιση της

επιστήμης με

την εξουσία πάνω στον φυσικό περίγυρο της ζωής μας: "Knowledge is power". Για να

επιτευχθεί όμως αυτός ο γνωστικός αναπροσανατολισμός είναι απαραίτητος ο καθαρμός

του

ανθρώπινου μυαλού από τις παραδεδομένες προκαταλήψεις που έχουν καθιερωθεί από

τη λογική μέθοδο, συσκοτίζοντας τα αισθητήρια και εμποδίζοντας την κατάδυση στην

πρωτογενή φυσική εμπειρία.

ΕΙΔΩΛΑ

Η θεωρία των "ειδώλων του νου" (idols of the mind) είναι ίσως το πιο γνωστό τμήμα της

γνωσιολογίας του Μπέικον. Πρόκειται για μιαν απόπειρα να ταξινομηθούν οι βασικοί τύποι

της

Page 26: ΣΗΜΕΙΩΣΕΙΣ ΦΙΛΟΣΟΦΙΑΣ I

πλάνης που έχουν παγιωθεί μέσα στην κοινή συνείδηση καθιστώντας την υποτελή στη

στείρα

λογοκρατία. Μία κατηγορία τέτοιων σφαλμάτων έχουν να κάνουν με τη φυσική τάση τόσο

ατόμων όσο και ομάδων να εγκλωβίζονται στις ανεξέταστες πίστεις και οπτικές τους, και

να πιστεύουν ότι ο κόσμος είναι όπως τον παρουσιάζει η υποκειμενική τους αντίληψη ή

επιθυμία. Οι πλάνες αυτές επιγράφονται είδωλα του "σπηλαίου" και της "φυλής" (idols of

the cave, idols of the tribe). Η πιο χαρακτηριστική από τις στάσεις αυτές είναι η πεποίθηση

ότι,

επειδή το ανθρώπινο λογικό διαθέτει μιαν εσωτερική τάξη και κανονικότητα, ο εξωτερικός

κόσμος οφείλει να αντιστοιχεί στη δομή των ιδεών μας. Άλλες απατηλές παραδοχές

πηγάζουν

από τις αυθαίρετες Σημασίες που προσδίδει στις λέξεις το εθιμικό της καθημερινής

επικοινωνίας (είδωλα της αγοράς - idols of the marketplace). Τo περιεχόμενο των εννοιών

υποτάσσεται στα συμφέροντα και στην ψυχολογία αυτών που τις χρησιμοποιούν, με

αποτέλεσμα

να περιβάλλεται η πραγματικότητα με ένα αδιαπέραστο προπέτασμα ψευδαισθήσεων. Το

αίτημα

του Μπέικον για τον ακριβή καθορισμό του νοήματος των όρων της γλώσσας με κριτήριο το

τμήμα της εμπειρίας στο οποίο αναφέρονται είναι η απαρχή της συστηματικής

ενασχόλησης με

τη δομή και τη λειτουργία της γλώσσας που χαρακτηρίζει την αγγλοσαξονική φιλοσοφική

παράδοση μέχρι και τις ημέρες μας. Μια άλλη, τέλος, πηγή σφαλμάτων είναι ο υπερβολικός

σεβασμός που τρέφει ο κοινός άνθρωπος για τις φιλοσοφικές αυθεντίες του

επιστημονικώς αναλφάβητου παρελθόντος (είδωλα του θεάτρου - idols of the theater).

Η μηχανική επανάληψη των ιδεών των σοφών της Αρχαιότητας (με πρώτο τον Αριστοτέλη)

αποφέρει τη νέκρωση του λογικού. Η αποτίναξη των ολέθριων δοξασιών τους θα καθαρίσει

το

έδαφος για τη μεθοδική εφαρμογή της "γνήσιας επαγωγής" (genuine induction) και τον

φωτισμό της κοινωνίας.

ΑΡΧΗ

Page 27: ΣΗΜΕΙΩΣΕΙΣ ΦΙΛΟΣΟΦΙΑΣ I

Στη φιλοσοφία είναι αναγκαία μια νέα αρχή από μηδενικό σημείο. Περιεχόμενο τώρα του

στοχασμού πρέπει να είναι η "φυσική ιστορία" (natural history),δηλαδή η απροκατάληπτη

εμπειρική έρευνα που δεν λαμβάνει τίποτε ως δεδομένο ως προς την εξήγηση των

φαινομένων

που μελετάει, Στόχος δε του εγχειρήματος αυτού είναι η διακρίβωση των πραγματικών

"φυσικών μορφών" (natural forms), η καταγραφή δηλαδή των βασικών ιδιοτήτων των

υλικών

σωμάτων και των αιτιακών διασυνδέσεων ανάμεσα τους. Ως παράδειγμα μιας τέτοιας

εμπειρικά

θεμελιωμένης αιτιακής εξήγησης ο Μπέικον προτείνει την κινητική θεωρία της θερμότητας:

η θερμότητα προκαλείται από τη βίαιη και ακανόνιστη κίνηση των στοιχειακών σωματιδίων

από

τα οποία σύγκειται ένα αισθητό πράγμα. Η υπόθεση αυτή έμελλε να βρει την πειραματική

της

επιβεβαίωση αιώνες αργότερα (βλέπε κεφάλαιο 4). Στην αντίληψη του Μπέικον, κατά

συνέπεια,

η επιστημονική θεωρία χτίζεται από τα κάτω προς τα πάνω. Ο νους παρακολουθεί την

εμπειρία

και συμμορφώνεται προς αυτήν. Όλες οι εξηγήσεις και οι νομοτελειακές διαπλοκές των

πραγμάτων αναφαίνονται εκ των υστέρων (a posteriori), ως αποτέλεσμα δηλαδή της

συναγωγής των εμπειρικών παρατηρήσεων. Η φύση μας διδάσκει ως προς την υφή της,

αρκεί

να αποδεχθούμε τον ρόλο μας ως μαθητών. Ταυτόχρονα όμως η παρατήρηση δεν είναι μια

τυχαία και σπασμωδική διαδικασία Αντίθετα, οργανώνεται από τον επιστημονικό νου με

τέτοιο τρόπο, ώστε να μεγιστοποιούνται οι πιθανότητες να εξαχθούν από αυτήν θεωρητικά

συμπεράσματα, να "σκοντάψει" - ούτως ειπείν - ο επιστήμονας στην κρίσιμη αιτιακή

διασύνδεση

που αναζητεί. Η επαγωγή είναι μια συγκροτημένη μέθοδος, και ο Μπέικον μας περιγράφει

τους

βασικούς τύπους για την ορθολογική οργάνωση της εμπειρικής έρευνας.

ΚΡΙΣΙΜΗ ΠΡΟΫΠΟΘΕΣΗ

Page 28: ΣΗΜΕΙΩΣΕΙΣ ΦΙΛΟΣΟΦΙΑΣ I

Η κρίσιμη προϋπόθεση είναι η δυνατότητα για την αντιπαραβολή των παρατηρησιακών

δεδομένων. Για τον σκοπό αυτό ο Μπέικον προτείνει να καταστρώνονται "συγκριτικοί

πίνακες" (comparative tables) που θα βάζουν τάξη στα ευρήματα μας. Πρόκειται καταρχήν

για πίνακες ταυτόχρονης παρουσίας ή "συμφωνίας" (tables of agreement): συλλέγουμε

πληθώρα εμπειρικών παραδειγμάτων του υπό εξέτασιν φαινομένου (π.χ. της θερμότητας)

και

παρατηρούμε ποιες άλλες εμπειρικές συνθήκες είναι ταυτόχρονα παρούσες κάθε φορά.

Μια άλλη

κατηγορία είναι οι πίνακες "απουσίας" (tables of absence): παρατηρούμε καταστάσεις από

τις

οποίες απουσιάζει το υπό μελέτη φαινόμενο και σημειώνουμε ποιες συνθήκες επίσης

απουσιάζουν. Τέλος, έχουμε τους πίνακες "βαθμιαίας μεταβολής" (tables of degrees):

παρατηρούμε το φαινόμενο σε διάφορα επίπεδα έντασης και καταγράφουμε ποιες

εμπειρικές

μεταβλητές εντείνονται ή εξασθενούν εκ παραλλήλου.

ΦΑΣΗ ΕΞΗΓΗΣΗΣ

Ο Μπέικον πιστεύει ότι κατά την κατασκευή των πινάκων αυτών θα φθάσει κάποια στιγμή

που ο

ερευνητής (έχοντας πλέον στη διάθεση του πληθώρα εμπειρικών στοιχείων μεθοδικά

συγκεντρωμένων) θα μπορέσει να "δει" την αιτιακή σχέση που ζητεί να αναδύεται μέσα

από το

σώμα της εμπειρίας που έχει ενώπιον του. Το κρίσιμο αυτό σημείο (όπου τερματίζεται η

διαδρομή μέσα στην εμπειρία και αρχίζει η θεωρητική φάση της έρευνας) θα καθορισθεί

από την

ύπαρξη μιας "παραδειγματικής περίπτωσης" (prerogative instance). Αυτή είναι μια

παρατήρηση-κλειδί η οποία θα επιτρέψει τον τερματισμό της συλλογής δεδομένων και

τη μεταπήδηση στην εξήγηση. Η "παραδειγματική περίπτωση" δεν ορίζεται εκ των

προτέρων.

Ο επιστήμονας θα κρίνει με βάση την πρότερα θεωρητική του κατάρτιση η πείρα αν έχει

φτάσει

η στιγμή να κλείσει η έρευνα με αφορμή ένα κρίσιμο δεδομένο. Η επιστημονικότητα

Page 29: ΣΗΜΕΙΩΣΕΙΣ ΦΙΛΟΣΟΦΙΑΣ I

παραμένει τελικά ένα φιλοσοφικό ιδεώδες, μια πολιτισμική στάση δια της οποίας

αγγέλλεται το

τέλος της μεσαιωνικής δουλείας του λογικού και η αρχή της νεωτερικότητας.

Η ΑΥΤΟΤΕΛΕΙΑ ΤΟΥ ΝΟΥ ΚΑΙ Η ΜΗΧΑΝΙΚΗ ΣΥΓΚΡΟΤΗΣΗ ΤΟΥ ΦΥΣΙΚΟΥ ΝΟΜΟΥ: Ο

ΟΡΘΟΛΟΓΙΚΟΣ ΕΝΟΡΑΤΙΣΜΟΣ ΤΟΥ ΚΑΡΤΕΣΙΟΥ.

ΑΦΕΤΗΡΙΑΚΗ ΠΑΡΑΔΟΧΗ

Σε αντιδιαστολή προς τον άκρατο επαγωγισμό του Μπέικον αναπτύχθηκε ένα εναλλακτικό

μεθοδολογικό πρόγραμμα με άξονα τις ιδέες του Γάλλου μαθηματικού και φιλοσόφου

Καρτέσιου (Rene Descartes, 1596-1650). Αφετηριακή παραδοχή του καρτεσιανισμού είναι

ότι

τα περιεχόμενα της εμπειρίας δεν μπορούν να αποτελέσουν το θεμέλιο για τη θεωρητική

κατανόηση των φυσικών νομοτελειών. Τα αισθητήρια όργανα είναι αναξιόπιστοι μάρτυρες

όσον αφορά την υποκείμενη δομή της πραγματικότητας. Δια των αισθήσεων παράγεται μια

καθαρά υποκειμενική αντίληψη των πραγμάτων, που δεν αντιστοιχεί κατ' ανάγκη με την

πραγματικότητα καθαυτή. Η εμπειρική γνώση βρίθει συγχύσεων και πλανών. Μια πρόταση

βασισμένη στην αισθητηριακή αντίληψη ενδέχεται να αποδίδει την πραγματικότητα, αλλά

δεν

είμαστε βέβαιοι για κάτι τέτοιο, εφόσον τίποτε δεν αποκλείει να πηγάζει από μια

παράκρουση ή

φαντασίωση.

ΚΡΙΤΗΡΙΟ ΓΝΩΣΗΣ

Με αναφορά στην εμπειρία και μόνο δεν επιτυγχάνουμε τίποτα περισσότερο από μια

πιθανολογική περιγραφή των όντων και των φαινομένων. Το κριτήριο όμως για γνώση με

την αυστηρή έννοια είναι κατά τον Καρτέσιο η απόλυτη βεβαιότητα για την αλήθεια των

λεγομένων μας. Βασιζόμενοι στις αισθήσεις είναι αδύνατον να υπερβούμε την αμφιβολία

για το

πώς είναι πράγματι ο κόσμος (Descartes, Meditations on First Philosophy, Meditation I).

Η καρτεσιανή γνωσιολογία επαναφέρει στην επικαιρότητα την επιχειρηματολογία των

αρχαίων σκεπτικών σχετικά με το αφερέγγυο της αισθητηριακής εποπτείας και την

Page 30: ΣΗΜΕΙΩΣΕΙΣ ΦΙΛΟΣΟΦΙΑΣ I

αδυναμία των αισθήσεων να παραγάγουν γνώση στέρεα "αγκυροβολημένη" σε αυτή την

ίδια τη

φύση των πραγμάτων. Η αναζωπύρωση του σκεπτικισμού είναι η αθέατη πλευρά της

επιστημολογίας της νεωτερικότητας. Η αποκοπή του ανθρώπινου υποκειμένου από τις

υπερβατικές αυθεντίες που εγγυούνταν παλιότερα την απόλυτη αλήθεια των

κοσμοθεωριών

γέννησε μια σκοτεινή αμφιβολία ως προς τη δυνατότητα των γνωστικών μας δυνάμεων να

παραγάγουν την αδιάσειστη και τελεσίδικη εξήγηση των φυσικών φαινομένων που

υποσχόταν η

καινούργια επιστήμη. Η "αποχώρηση του Θεού" (deus absconditus) από τη φυσική και την

κοινωνική πραγματικότητα έθεσε το εύλογο ζήτημα αν ένα πεπερασμένο όν -όπως ο

άνθρωπος-

είναι όντως σε θέση να συλλάβει την ουσία ενός άπειρου σύμπαντος με τα περιορισμένα

γνωστικά μέσα που διαθέτει. Την απορία αυτή διατύπωσε με αξιοσημείωτη υπαρξιακή

ένταση

η φιλοσοφία του Michel de Montaigne (1533-1592). Η γνωσιολογία του Καρτέσιου απηχεί

την ανησυχία αυτή, επεξεργάζεται όμως, τα μέσα για να την κατανικήσει.

ΔΙΑΚΡΙΣΗ ΑΠΟΚΛΕΙΣΜΟΥ ΤΗΣ ΕΜΠΕΙΡΙΑΣ

Για να αιτιολογήσει τον αποκλεισμό της εμπειρίας από τη διαδικασία παραγωγής της

αλήθειας ο

Καρτέσιος χρησιμοποιεί μια διάκριση. Πρόκειται για τον διαχωρισμό ανάμεσα στις

πρωταρχικές ή πρωτεύουσες και στις δευτερογενείς ιδιότητες των φυσικών σωμάτων

(primary and secondary qualities). Ένα αντικείμενο π.χ. ένα κομμάτι κερί) είναι δυνατόν να

περιγραφεί από δύο εντελώς διαφορετικές οπτικές γωνίες. Πρώτον, ως φαινόμενο, με

αναφορά

δηλαδή στον τρόπο με τον οποίο εμφανίζεται σε εμάς, τους εμπειρικούς παρατηρητές. Από

τη

σκοπιά αυτή καταγράφουμε το πώς επιδρά στα αισθητήρια μας: το χρώμα, τη γεύση, την

οσμή,

τον ήχο του, την αφή του, τη σκληρότητα του κτλ. Οι ιδιότητες αυτές είναι αντιδράσεις των

αισθητηρίων οργάνων μας και ενυπάρχουν μέσα σε αυτά εξαρτώμενες πλήρως από τη

Page 31: ΣΗΜΕΙΩΣΕΙΣ ΦΙΛΟΣΟΦΙΑΣ I

φυσική τους κατάσταση (όποιος έχει κρυολόγημα δεν αντιλαμβάνεται τις γεύσεις των

τροφών). Για τον λόγο αυτό ονομάζονται και δευτερογενείς, επειδή κατά τον Καρτέσιο δεν

αφορούν την πραγματική υπόσταση του υπό εξέτασιν πράγματος. Μπορούν κάλλιστα να

αλλοιωθούν όλες, χωρίς το πράγμα να απολέσει την αντικειμενική του ουσία. Όταν, επί

παραδείγματι, φέρουμε το κερί κοντά στη φλόγα και λιώσει, τότε όλες οι εμπειρικές του

ιδιότητες αλλάζουν άρδην, χωρίς να πάψει να υφίσταται το εν λόγω φυσικό σώμα.

Η ουσιώδης κατασκευή των σωμάτων βρίσκεται πίσω και πέρα από τις εντυπώσεις που

σχηματίζουμε γι' αυτά. Μας φανερώνεται μέσα από μια πράξη όχι των αισθήσεων, αλλά

του

νου, ο οποίος διεισδύει στη μη εμπειρική, καθαρά μαθηματική διάσταση της

πραγματικότητας.

Αυτή η πραγματική δομή του κόσμου μας αποκαλύπτεται κατά τον Καρτέσιο μέσω των

εννοιών της γεωμετρίας· για άλλους - όπως ο Γαλιλαίος και ο Λοκ - μέσω των

χαρακτηριστικών ενός σώματος τα οποία υπόκεινται σε ποσοτικές μετρήσεις (βάρος,

σχήμα, μέγεθος, ταχύτητα): αυτές είναι και οι πρωτογενείς του ιδιότητες. Αυτές έχουν

τη ρίζα τους μέσα στην ίδια την ουσία των πραγμάτων και δεν εξαρτώνται από τη

λειτουργία

των αισθητηρίων μας. Ο "πραγματικός κόσμος" στον οποίον αναφέρεται η επιστήμη είναι

άχρωμος, άοσμος, άγευστος, άηχος κτλ., ένα σύστημα με άλλα λόγια υλικών σωμάτων εν

κινήσει. Τα ζώα, στην πραγματικότητα αποτελούν μηχανικά αυτόματα (όπως άλλωστε και

το ανθρώπινο σώμα), ένα σύστημα μοχλών, εμβόλων και άλλων τέτοιων εξαρτημάτων,

που απλώς εκτελούν κάποιες προδιαγεγραμμένες κινήσεις χωρίς αυτοσυνειδησία. Η

ανθρωπιά

του ανθρώπου έγκειται ακριβώς στη δυνατότητα του να λαμβάνει τον εαυτό του ως

αντικείμενο

(να αυτοσκοπείται), να έχει δηλαδή συνείδηση του Εγώ του ως πηγή και αιτίας των

πράξεων

του.

COGITO ERGO SUM

Page 32: ΣΗΜΕΙΩΣΕΙΣ ΦΙΛΟΣΟΦΙΑΣ I

Ο άνθρωπος σκέπτεται τις σκέψεις του, και καταλαβαίνει ότι τις σκέπτεται. Νοεί τον εαυτό

του

ως πράγμα νοούν (res cogitans), και η άμεση αυτή επίγνωση ορίζει ακριβώς την ύπαρξη

του. Ο

άνθρωπος υπάρχει ως τέτοιος όχι μέσα από το σώμα του, δια του οποίου εντάσσεται στη

ζωική τάξη, αλλά μέσα από την καθαρή διάνοια του. Αυτό είναι το νόημα της περίφημης

ρήση "cogito ergo sum" που συγκεφαλαιώνει την καρτεσιανή ανθρωπολογία (Descartes,

Meditations, Meditation II). Η επιστήμη συνεπώς προχωράει πέρα από τα εμπειρικά

φαινόμενα

και ψηλαφεί την ουσία των όντων όπως υπάρχει καθαυτή, ανεξάρτητα από το τι

αισθανόμαστε ή

πιστεύουμε ή νομίζουμε περί αυτών. Η φύση είναι μια άπειρη μηχανή όσο και αν δεν

είμαστε σε

θέση να την αντιληφθούμε ως τέτοια.

ΧΩΡΙΣΜΟΣ - ΕΠΙΣΤΗΜΟΛΟΓΙΚΟ ΠΛΑΙΣΙΟ

Ο χωρισμός ανά μέσα στη φαινομενικότητα και την πραγματικότητα που υποθέτει ο

Καρτέσιο έθεσε το επιστημολογικό πλαίσιο για όλη τη νεότερη επιστήμη. Παρά τις

αποκλίσεις ως

προς τη συγκεκριμένη μέθοδο για την επιτυχία του στόχου, κοινή επιδίωξη για κάθε

επιστήμονα

στο εξής θα ήταν να σαρωθεί το πέπλο των υποκειμενικών κρίσεων και προτιμήσεων για

να αναδυθεί ο αντικειμενικός πυρήνας τω' πραγμάτων.

ΕΠΙΣΤΗΜΗ - ΕΠΙΣΤΗΜΟΛΟΓΙΚΟΣ ΡΕΑΛΙΣΜΟΣ

Επιστήμη σημαίνει περιγραφή των φυσικών όντων όπως είναι καθαυτά και την επιτυχία

μιας ευρείας συναίνεσης όλων των διαπιστευμένων ερευνητών γύρω από τις κεφαλαιώδεις

θεωρητικές αλήθειες περί του φυσικού σύμπαντος. Αυτός ο επιστημολογικός ρεαλισμός,

όσο και

αν αμφισβητήθηκε, απετέλεσε εντούτοις μια γενική παραδοχή εργασίας πάνω στην οποία

εδράσθηκε η αυτό πεποίθηση της νεωτερικής επιστήμης. Η δομή λοιπόν των όντων

καθαυτών

μας αποκαλύπτεται κατά τον Καρτέσιο μόνο μέσα από τη μεθοδική χρήση του λογικού,

Page 33: ΣΗΜΕΙΩΣΕΙΣ ΦΙΛΟΣΟΦΙΑΣ I

ανεπηρέαστου από τις παρεμβολές της εμπειρίας.

ΘΕΜΕΛΙΑΚΗ ΠΡΟΫΠΟΘΕΣΗ

Η αυτοσυγκέντρωση του νου είναι η θεμελιακή προϋπόθεση για την επιστήμη. Η

επιστημονική αποτυχία των αρχαίων δεν οφείλεται στην έμφαση που έδιναν στον λόγο,

αλλά

στο γεγονός ότι δεν είχαν αναπτύξει την ορθή μέθοδο για τον "διακανονισμό"

(reglement) και τον έλεγχο της διανοητικής διαδικασίας. Ο Καρτέσιος θα προχωρήσει στη

διατύπωση αυτής της μεθόδου, δια της οποίας ζητάει να θεμελιώσει εξ υπ' αρχής την

επιστήμη

σε όλα της τα πεδία.

ΛΟΓΙΚΗ ΠΑΡΑΓΩΓΗ - deduction - ΕΡΕΥΝΑ - ΑΞΙΩΜΑ

Πρόκειται για την αξιωματική μέθοδο ή τη μέθοδο της λογικής παραγωγής

(deduction), η οποία λαμβάνει ως πρότυπο τη λειτουργία του νου στην περιοχή των

μαθηματικών. Η επιστημονική έρευνα άρχεται με την αναζήτηση των πρωταρχικών

αληθειών ή αξιωμάτων που θεμελιώνουν μιαν επιστήμη, από τα οποία παράγονται με τη

διαδικασία της έγκυρης λογικής συνεπαγωγής όλες οι υπόλοιπες προτάσεις (θεώ ρήματα)

που τη

συγκροτούν ως πλήρες αληθειακό σύστημα. Αξίωμα είναι μια αληθής ιδέα την οποία ο

νους, όταν συγκεντρώσει την προσοχή του στον εαυτό τον συλλαμβάνει αμέσως,

επειδή είναι απόλυτα καθαρή και διακριτή από όλες τι υπόλοιπες (clear and distinct).

Η παρουσία της ιδέας αυτής του επιβάλλεται απ' τη λογική της αναγκαιότητα ή προφάνεια.

Το

αξίωμα "δείχνει" αμέσως την αλήθεια του χωρίς να χρειάζεται να αποδειχτεί η αλήθεια

αυτή με

τη μεσολάβηση άλλων αληθειών. Οι αξιωματικές προφάνειες είναι έμφυτες στον νου

(innate)

στον νου: sunt in nobis semina scientiae (βρίσκονται μέσα μας τα σπέρματα της επιστήμης)

το πρώτο δε βήμα στην επιστημονική έρευνα είναι να τις αναγνωρίσουμε.

ΒΗΜΑΤΑ ΑΝΑΛΥΤΙΚΗΣ ΜΕΘΟΔΟΥ:

Δεν υπάρχει πιο κατηγορηματική κατάφαση στη δυνατότητα του ανθρώπινου λόγου να

Page 34: ΣΗΜΕΙΩΣΕΙΣ ΦΙΛΟΣΟΦΙΑΣ I

κατακτήσει τον οντολογικό πυρήνα της πραγματικότητας μέχρι τα έσχατα βάθη του. Ο

άνθρωπος μπορεί να έχει αποκοπεί από τον Θεό, δύναται όμως να αντλήσει από μέσα του

τη

δύναμη για να κατακυριεύσει τον φυσικό κόσμο. Αυτή η ίδια η ύπαρξη του Θεού γίνεται

τώρα

ένα λογικό πρόβλημα, το οποίο επιλύεται κατά τον Καρτέσιο δια της εφαρμογής της ως άνω

αξιωματικής μεθόδου (πρόκειται για το λεγόμενο "οντολογικό επιχείρημα"). Από τη στιγμή

που

αποδεικνύεται η ύπαρξη του Θεού είμαστε βέβαιοι ότι οι προφανείς ιδέες που

ανακαλύπτουμε

μέσα μας αντανακλούν αυτή την ίδια την οντολογική τάξη του σύμπαντος, και δεν

πρόκειται για

μετέωρες φαντασιώσεις. Η αισιόδοξη αυτή επιστημολογία σφράγισε τον αιώνα του

Διαφωτισμού.

Καίριο ρόλο στην ευρετική αυτή διαδικασία παίζει η αναλυτική μέθοδος (όπως την είχε

ήδη ορίσει η σχολή της Πάδοβας). Τα βήματα της, όπως συνοψίζονται από τον Καρτέσιο,

είναι

τα ακόλουθα:

Το πρώτο ήταν να μην παραδέχομαι τίποτε ως αληθές που δεν το γνώριζα αδιαμφισβήτητα

ως

τέτοιο. Με άλλα λόγια να αποφεύγω προσεκτικά κάθε βιασύνη και να μην περιλαμβάνω

στις

κρίσεις μου τίποτε περισσότερο από ό,τι φανερωνόταν στο μυαλό μου ευκρινώς και

διακριτώς

(clearly and distinctly), έτσι που να αποκλείει κάθε λόγο για αμφιβολία. Το δεύτερο ήταν να

διαιρώ καθεμία από τις δυσκολίες που εξέταζα σε όσα τμήματα ήταν δυνατόν και αναγκαίο

για

την ορθή τους επίλυση. Το τρίτο ήταν να διευθύνω τις σκέψεις μου με τέτοια τάξη, που να

αρχίζει με τα αντικείμενα που είναι τα απλούστερα και ευκολότερα για να τα γνωρίσω. Και

μετά

να ανέρχομαι βαθμιαία στη γνώση των πιο σύνθετων...Και τελευταίο, σε κάθε περίπτωση

να

Page 35: ΣΗΜΕΙΩΣΕΙΣ ΦΙΛΟΣΟΦΙΑΣ I

κάνω εξαντλητικές απαριθμήσεις και γενικές θεωρήσεις που θα με βεβαίωναν ότι τίποτε

δεν είχε

παραλειφθεί (Descartes, Discours de Ια methode, Μέρος Β).

Η μέθοδος αυτή δεν είναι εντέλει παρά εκείνη του γεωμέτρη, που εστιάζει την προσοχή του

στο

προ οφθαλμών πρόβλημα τέμνοντας το στα επιμέρους συστατικά του μέχρι να "λάμψει"

εμπρός

του μια κρίσιμη διασύνδεση δια της οποίας ανάγεται σε σχέσεις οι οποίες έχουν ήδη

αποδειχθεί.

Η βασική παραδοχή εδώ είναι ότι όσο πιο απλή είναι μια πρόταση, τόσο περισσότερο

γνωστή

καθίσταται. Ένα σύνθετο ζήτημα είναι ένα πλέγμα απλών εννοιών, οπότε η θεωρητική

απόδειξη

αρχίζει με την ανάλυση του στις πρωταρχικές ιδέες που το αποτελούν.

ΚΑΡΤΕΣΙΟΣ - ΕΦΑΡΜΟΓΗ ΑΝΑΛΥΤΙΚΗΣ ΜΕΘΟΔΟΥ - ΘΕΜΕΛΙΩΔΕΣ ΑΞΙΩΜΑ ΤΗΣ

ΦΥΣΙΚΗΣ

Η εφαρμογή της μεθόδου αυτής στη φυσική επιστήμη είχε δύο σημαντικά

αποτελέσματα. Κατακύρωσε, κατά πρώτον, τελεσίδικα το μηχανιστικό κοσμοείδωλο,

που κυριάρχησε στη νεότερη επιστήμη, αλλά και ηγεμόνευσε στον πολιτισμό της

νεωτερικότητας. Παραλλήλως, όμως, διαμόρφωσε ένα ιδιότυπο σύστημα φυσικής σκέψης,

που στις αρχές του 18ου αιώνα ανταγωνιζόταν τη νευτώνεια μηχανική (η οποία τελικώς

και επεκράτησε). Το θεμελιώδες "αξίωμα" της φυσικής είναι κατά τον Καρτέσιο η ταύτιση

της

ύλης με την έκταση: "παν σώμα εκτατόν":

Αρχή W: Η φύση των υλικών σωμάτων δεν συνίσταται στο βάρος τους ούτε στη σκληρότητα

τους ούτε στο χρώμα τους κτλ., αλλά στην έκταση τους και μόνο (Descartes, Principles of

Philosophy, Μέρος Π). Ο ορισμός αυτός επιτρέπει μια μαθηματική ανάλυση της φυσικής

τάξης,

σύμφωνα με τις έννοιες της γεωμετρίας.

ΚΙΝΗΣΗ ΤΗΣ ΥΛΗΣ

Από την παραδοχή αυτή συνάγεται ότι δεν υπάρχει στη φύση κενό: το υλικό σύμπαν είναι

Page 36: ΣΗΜΕΙΩΣΕΙΣ ΦΙΛΟΣΟΦΙΑΣ I

plenum, μια πλησμονή φυσικής ουσίας. Το κενό είναι μια "πρόληψη" που βασίζεται στην

ανεπεξέργαστη αίσθηση. Η κίνηση των σωμάτων επιτελείται μέσα στο σχήμα αυτό μόνο

μέσα από την άμεση πρόσκρουση ή επαφή του ενός με το άλλο. Από αυτό έπεται ότι η

μετατόπιση ενός σώματος συνεπιφέρει την αλλαγή θέσης όλων ταυτοχρόνως. Η κίνηση της

ύλης του σύμπαντος είναι υπό την έποψη αυτή μια γιγαντιαία δίνη (vortex), κατά την οποία

τα

βαρύτερα σώματα φέρονται προς το κέντρο, τα δε ελαφρύτερα προς την περιφέρεια. Η

καρτεσιανή φυσική απορρίπτει συνεπώς (ως λογικώς αστήρικτη) την αφετηριακή υπόθεση

της

νευτώνειας μηχανικής ότι η βαρύτητα δρα εξ αποστάσεως διαμέσου του κενού. Απορρίπτει

επίσης και το σχήμα του σύμπαντος που προκύπτει από αυτήν (βαρύτητα).

ΧΑΡΑΚΤΗΡΙΣΤΙΚΑ

Συν τω χρόνω τα πειραματικά ευρήματα βάρυναν αποφασιστικά υπέρ της νευτώνειας

θεωρίας.

Ο καρτεσιανισμός απέτυχε τελικά ως φυσική θεωρία. Η μη εμπειρική θεμελίωση των

εννοιών ισχύει στην περιοχή των μαθηματικών. Εκεί σημειώθηκε και η πρωτοπόρος

επιστημονική συνεισφορά του Καρτέσιου, που ήταν η θεμελίωση του καινούργιου κλάδου

της

αναλυτικής γεωμετρίας. Στην περιοχή της φυσικής σκέψης όμως η αχίλλειος πτέρνα του

ήταν η

αδυναμία να κατανοήσει την αναγκαία διαπλοκή ανάμεσα στη θεωρία και στην υλική

πραγματικότητα, να αναγνωρίσει το πείραμα δηλαδή ως έσχατο κριτή της εγκυρότητας των

θεωρητικών υποθέσεων.

ΧΡΗΣΙΜΟΤΗΤΑ ΤΟΥ ΠΕΙΡΑΜΑΤΟΣ

Ο Καρτέσιος πιστεύει ότι το πείραμα χρησιμεύει είτε α) για να προσδώσει εκ / των

υστέρων εμπειρικό περιεχόμενο στη θεωρία που έχει διατυπωθεί ανεξάρτητα από την

παρατήρηση (όπως, λ.χ., οι μαθηματικές έννοιες που χρησιμοποιούνται για να

μορφοποιήσουν

τον φυσικό χώρο χωρίς να έχουν παραχθεί από αυτόν), είτε β) για να επιβεβαιωθεί η μία ή

η

Page 37: ΣΗΜΕΙΩΣΕΙΣ ΦΙΛΟΣΟΦΙΑΣ I

άλλη από δύο εναλλακτικές λογικές συνεπαγωγές, που είναι και οι δύο δυνατές από μία

και την αυτή αρχική παραδοχή. Και στις δύο όμως περιπτώσεις δεν θίγεται η αυθυπαρξία

της

θεωρίας έναντι των εμπειρικών δεδομένων και τελικά του πειράματος. Είναι αδιανόητο η

παρατήρηση να διαψεύδει τη θεωρία που έχει παραχθεί κατ' αυστηρή εφαρμογή της

"ορθής μεθόδου" για τη χρήση του λογικού. Το πείραμα δικαιώνεται μόνο στον βαθμό που

κυρώνει εκ των υστέρων τις λογικές ενοράσεις του επιστημονικού νου. Εάν δεν το κάνει

αυτό,

και είμαστε βέβαιοι ότι δεν έχουμε παραβιάσει τους κανόνες της λογικής απαγωγής, τότε

κάποιο

πρόβλημα υπάρχει με το πείραμα. Αυτός ο δογματισμός του καθαρού (μαθηματικού)

λόγου "χρωμάτισε" ολόκληρη την παράδοση του φιλοσοφικού ορθολογισμού που πήγασε

από

τη σκέψη του Καρτέσιου.

Η ΕΜΠΕΙΡΙΚΗ ΚΑΤΑΓΩΓΗ ΤΩΝ ΕΝΝΟΙΩΝ ΚΑΙ Ο ΕΜΜΕΣΟΣ Η ΜΕΤΡΙΟΠΑΘΗΣ

ΡΕΑΛΙΣΜΟΣ ΤΗΣ ΓΝΩΣΙΟΛΟΓΙΑΣ ΤΟΥ ΛΟΚ.

ΚΕΝΤΡΙΚΗ ΠΑΡΑΔΟΧΗ

Το ζήτημα των ορίων της ανθρώπινης γνώσης (το ζήτημα δηλαδή του γνωσιολογικού

ρεαλισμού), η αντικειμενικότητα των εννοιών μας, καθώς και η έννοια της αιτιότητας

εξακολούθησαν να ερευνώνται στο πλαίσιο του άλλου μεγάλου επιστημολογικού ρεύματος

που διαμορφώθηκε τον 18ο αιώνα, του εμπειρισμού. Ο εμπειρισμός είναι μία

γνωσιολογική θεώρηση που επεξεργάζεται συστηματικά τις μεθόδους δια των οποίων

εξάγεται η

γνώση από την αισθητηριακή αντίληψη, προσπάθεια δηλαδή που είχε αρχίσει ο Μπέικον.

Στην

πορεία όμως τροποποιεί πολλές από τις παραδοχές που διαπερνούσαν την πρώιμη και εν

πολλοίς αφελή εκείνη εκδοχή της θεωρίας.

Ο εμπειρισμός διαμορφώνεται σε συστηματική επιστημολογία από τον Τζών Λόκ (John

Locke, 1632-1704). Η κεντρική του παραδοχή είναι ότι τα όρια της γνώσης συμπίπτουν

Page 38: ΣΗΜΕΙΩΣΕΙΣ ΦΙΛΟΣΟΦΙΑΣ I

με τα όρια της εμπειρίας. Ό,τι ξεπερνάει την εμπειρία ή δεν μπορεί να συσχετισθεί με

αυτήν

είναι εξ ορισμού μη γνωστό. Χωρίς την εισροή πληροφοριών από τον δίαυλο των

αισθητηρίων

οργάνων, ο ανθρώπινος νους θα παρέμενε απολύτως κενός, μια άγραφη δέλτος (tabula

rasa). Από αυτό έπεται ότι δεν υπάρχουν ιδέες έμφυτες στο λογικό, όπως ισχυριζόταν η

καρτεσιανή επιστημολογία. Ο νους όχι μόνο δεν είναι αποκομμένος από το σώμα, αλλά η

λειτουργία του είναι ακριβώς η επεξεργασία των υλικών που του παρέχουν οι αισθήσεις. Οι

αισθήσεις ενσωματώνουν τον άνθρωπο στον φυσικό του περίγυρο και τον συντονίζουν

προς

τους ρυθμούς του (Locke, 1959, Βιβλί

Έχει επεξεργασθεί από τον/την marlene στις Τρι Ιούν 19, 2007 10:30 pm, 1 φορά

Επιστροφή στην κορυφή

Επισκόπηση του προφίλ των χρηστών Αποστολή προσωπικού μηνύματος

marlene

Χρήστης

Χρήστης

Ένταξη: Sep 22, 2005

Δημοσιεύσεις: 2463

ΔημοσίευσηΔημοσιεύθηκε: Τρι Ιούν 19, 2007 10:25 pm Θέμα δημοσίευσης: Απάντηση

με παράθεση αυτού του μηνύματος

Η κεντρική του παραδοχή είναι ότι τα όρια της γνώσης συμπίπτουν

με τα όρια της εμπειρίας. Ό,τι ξεπερνάει την εμπειρία ή δεν μπορεί να συσχετισθεί με

αυτήν

Page 39: ΣΗΜΕΙΩΣΕΙΣ ΦΙΛΟΣΟΦΙΑΣ I

είναι εξ ορισμού μη γνωστό. Χωρίς την εισροή πληροφοριών από τον δίαυλο των

αισθητηρίων

οργάνων, ο ανθρώπινος νους θα παρέμενε απολύτως κενός, μια άγραφη δέλτος (tabula

rasa). Από αυτό έπεται ότι δεν υπάρχουν ιδέες έμφυτες στο λογικό, όπως ισχυριζόταν η

καρτεσιανή επιστημολογία. Ο νους όχι μόνο δεν είναι αποκομμένος από το σώμα, αλλά η

λειτουργία του είναι ακριβώς η επεξεργασία των υλικών που του παρέχουν οι αισθήσεις. Οι

αισθήσεις ενσωματώνουν τον άνθρωπο στον φυσικό του περίγυρο και τον συντονίζουν

προς

τους ρυθμούς του (Locke, 1959, Βιβλίο Ι, κεφ. I, "No Innate Speculative Principles").

Οι αντιλήψεις αυτές θέτουν τέλος στο πανσοφικό ιδεώδες που ενέπνεε το κίνημα του

ορθολογισμού. Η γνώση μας είναι περιορισμένη, ελλιπής και αποσπασματική. Δεν είναι

όμως

λόγος αυτός για να περιφρονήσουμε τις εμπειρικές της ρίζες, αναζητώντας φανταστικά

υποκατάστατα που αναβιώνουν τον μεσαιωνικό δυϊσμό ανάμεσα στην ψυχή και στο σώμα.

ΑΝΘΡΩΠΟΣ

Ο άνθρωπος είναι ένα φυσικό όν προικισμένο με διανοητικές δυνάμεις οι οποίες, όσο

ατελείς και αν είναι, παραμένουν εντούτοις ικανές να αντεπεξέλθουν στα πρακτικά

καθήκοντα της καθημερινής ζωής, εξασφαλίζοντας και την επιβίωση και την πρόοδο μας. Η

επιστήμη καταξιώνεται στον βαθμό ακριβώς που συνεισφέρει στην αποδοτική οργάνωση

της

ζωής, με στόχο τη βιοτική άνεση και την πολιτισμική ανέλιξη. Δεν έχει νόημα να της

επιβάλλεται

ένα εξωπραγματικό κριτήριο εγκυρότητας (όπως εκείνο της απόλυτης βεβαιότητας που είχε

προκρίνει ο Καρτέσιος), γιατί στην περίπτωση αυτή θα πρέπει να πούμε ότι δεν υπάρχει

γνώση.

ΑΡΧΗ ΤΗΣ ΣΥΝΔΥΑΣΤΙΚΟΤΗΤΑΣ

Η ανθρώπινη διάνοια (understanding) παράγει λοιπόν αλήθεια προσδεχόμενη τα

πρωτογενή

δεδομένα που της παρέχει η αισθητηριακή αντίληψη, και δουλεύοντας πάνω σε αυτά. Ο

τρόπος

που τα επεξεργάζεται βασίζεται στην "αρχή της συνδυαστικότητας" (principle of

Page 40: ΣΗΜΕΙΩΣΕΙΣ ΦΙΛΟΣΟΦΙΑΣ I

association), που είναι η κεντρική μεθοδολογική υπόθεση του κλασικού εμπειρισμού. Το

λογικό παράγει προτάσεις που περιγράφουν τον κόσμο συνδυάζοντας με κατάλληλους

τρόπους

τις πληροφορίες που μεταφέρουν οι αισθήσεις.

ΣΥΣΤΑΤΙΚΑ ΕΜΠΕΙΡΙΚΗΣ ΓΝΩΣΗΣ

Τα πρωταρχικά συστατικά της εμπειρικής γνώσης είναι οι λεγόμενες "απλές ιδέες" της

αισθητηριακής αντίληψης: του φυσικού κόσμου από τη μια μεριά και του εαυτού μας από

την

άλλη (simple ideas of sensation and reflection). Γνωρίζω, επί παραδείγματι, ότι υπάρχω

παρατηρώντας τον εαυτό μου να σκέπτεται και να επιθυμεί (Locke, 1959, Βιβλίο Ι, κεφ.

VII).Τα

πράγματα της καθημερινότητας είναι σύνθετες ιδέες. Η ιδέα δεν αποτελεί μιαν υπέρτερη

πραγματικότητα. Είναι αντίθετα ένα λίγο ή πολύ χλομό απείκασμα των πραγμάτων του

φυσικού μας περιβάλλοντος και των ιδιοτήτων τους. Από την τοποθέτηση όμως αυτή

προκύπτει το οξύτατο ζήτημα της σχέσης ανάμεσα στις ιδέες (που υπάρχουν στο εσωτερικό

του

γνωστικού υποκειμένου) και στα υλικά όντα που υπάρχουν καθαυτά (έξω και ανεξάρτητα

από

τη διάνοια). Εφόσον ο νους ασχολείται με αντικείμενα που βρίσκει εντός του (τις ιδέες),

πώς θα

βεβαιωθούμε ότι η γνώση που κατασκευάζει από αυτά δεν είναι μια (συλλογική έστω)

φαντασίωση ξεκομμένη από την αντικειμενική υπόσταση των πραγμάτων;

Για να απαντήσει στην απορία αυτή (που είναι ακριβώς η καρτεσιανή ένσταση κατά της

εγκυρότητας της εμπειρικής γνώσης) ο Λοκ κινητοποιεί πάλι το γνωστό μας σχήμα των

πρωτογενών και των δευτερογενών ιδιοτήτων των πραγμάτων (primary and secondary

qualities). Ενώ όμως ο Καρτέσιος από τη διάκριση αυτή εξάγει την πλήρη αφερεγγυότητα

της

εμπειρικής γνώσης, ο Λόκ τη χρησιμοποιεί για να στηρίξει τώρα ένα επιχείρημα

μετριοπαθούς ή κριτικού ρεαλισμού με βάση την αισθητηριακή αντίληψη. Ο ισχυρισμός

είναι κατ' ουσία ότι η εμπειρία διορθώνει την εμπειρία. Δεν είναι ανάγκη να απορρίψει

κανείς

Page 41: ΣΗΜΕΙΩΣΕΙΣ ΦΙΛΟΣΟΦΙΑΣ I

ολοσχερώς τις πληροφορίες που μεταδίδουν τα αισθητήρια για να αποκαθάρει το

εμπειρικό

περιεχόμενο του νου από τις συγχύσεις και τις πλάνες που ομολογουμένως το

χαρακτηρίζουν

(Locke, 1959, Βιβλίο Ι, κεφ. VII, VIII).

ΠΟΣΟΤΙΚΑ ΧΑΡΑΚΤΗΡΙΣΤΙΚΑ ΤΩΝ ΣΩΜΑΤΩΝ

Τα δεδομένα των αισθήσεων αποτελούν θεμέλιο για επιστημονική γνώση του εξωτερικού

κόσμου, επειδή οι πρωτογενείς ιδιότητες των σωμάτων περιγράφουν την αντικειμενική

τους

κατασκευή: αντιστοιχούν ή προσομοιάζουν στη δομή ενός φυσικού όντος όπως είναι

καθαυτό. Η

συστηματική μέτρηση των ποσοτικών χαρακτηριστικών των σωμάτων (μάζα, σχήμα,

ταχύτητα κ.λπ.) επιτρέπει στην επιστήμη να σχηματίσει μια θεωρητική εικόνα του φυσικού

συστήματος που είναι η εγγύτερη δυνατή (λογικώς και πρακτικώς) στην αντικειμενική

πραγματικότητα.

ΑΙΤΙΟΤΗΤΑ

Μέσα στο θεωρητικό σχήμα που χτίζει η επιστήμη περιέχεται και η αιτιότητα. Ο κόσμος

είναι μια

αιτιακή διαπλοκή υλικών πραγμάτων (ουσιών, substances) που υπάρχουν καθαυτά (έξω

από τη

συνείδηση) και καθορίζουν το περιεχόμενο του νου. Η αιτιότητα είναι μέσα στη φύση. Τα

υλικά

σώματα επενεργούν το ένα πάνω στο άλλο κατά καθορισμένους τρόπους, αλλά και πάνω

στην

ανθρώπινη συνείδηση που τα παρατηρεί, αφήνοντας πάνω της τον "τύπο" της μορφής

τους.

ΣΥΝΘΗΚΕΣ

Ο Λόκ φροντίζει ιδιαίτερα να υπογραμμίσει δύο θεμελιώδεις συνθήκες. Πρώτον: η

επιστήμη

είναι αυστηρώς περιγραφική. Δεν κάνει "βαθιά οντολογία". Είναι εύλογο να υποθέσουμε

ότι

Page 42: ΣΗΜΕΙΩΣΕΙΣ ΦΙΛΟΣΟΦΙΑΣ I

τα φυσικά πράγματα έχουν μέσα τους "δυνάμεις" (powers), οι οποίες είναι και το

πρωταρχικό

αίτιο των ιδεών που ανακαλύπτουμε μέσα στον νου. Τις δυνάμεις όμως αυτές τις

γνωρίζουμε

μόνο μέσα από τις επιδράσεις τους πάνω στα αισθητήρια μας και όχι καθαυτές. Δεν έχουμε

δηλαδή τρόπο να περάσουμε "πίσω" - ούτως ειπείν - από το επίπεδο της παρατήρησης (το

"πέπλο των φαινομένων"), για να "αδράξουμε" ένα πράγμα στον καθαρό οντολογικό του

πυρήνα. Η επιστήμη μας διδάσκει πώς λειτουργούν τα πράγματα της εμπειρίας. Δεν μας

διδάσκει

γιατί είναι αναγκαίο να λειτουργούν με αυτό τον τρόπο και όχι αλλιώς. Ένα υλικό πράγμα

το

γνωρίζουμε μέσα από τη συναλλαγή μας μαζί του, και η συναλλαγή αυτή διαμεσολαβείται

πάντα

από την υποκειμενική αίσθηση.

Η "πραγματική ουσία" των πραγμάτων (real essence) είναι αυτό που κρύβεται πίσω

από τις εμφανίσεις τους σε εμάς, και "συγκρατεί" αυτό και μόνο αυτό το πλέγμα των

ιδιοτήτων που παρατηρούμε, ούτως ώστε να μένει στο διηνεκές σταθερό. Αυτό το κάτι δεν

μπορούμε να το γνωρίσουμε (Locke, 1959, Βιβλίο Π, κεφ. XXIII).Με τον περιορισμό αυτό

απορρίπτεται συνεπώς ο στόχος της επιστήμης όπως τον είχε ορίσει ο Μπέικον: η διάγνωση

δηλαδή των θεμελιωδών "φυσικών μορφών" στο έσχατο βάθος τους. Η επιστήμη, παρά την

κεντρική της θέση στην ανθρώπινη κατανόηση του φυσικού Είναι, εμφανίζεται πλέον ως

μία

δραστηριότητα ατελής και περιορισμένη από τις ίδιες τις εγγενείς αδυναμίες του

ανθρώπινου νου. Η γνώση που παράγει είναι η καλύτερη δυνατή και εξόχως επωφελής. Δεν

παύει όμως να είναι ενδεχομενική και πιθανολογική. Της λείπει η βεβαιότητα που

χαρακτηρίζει

τις μαθηματικές αλήθειες. (Οι τελευταίες όμως δεν αναφέρονται στη φυσική

πραγματικότητα

αλλά στον τρόπο με τον οποίο παράγεται η μία έννοια από την άλλη μέσα στο μυαλό μας.

Αυτή

η επιστημολογική, ταπεινοφροσύνη είναι τελικά η πολυτιμότερη κληρονομιά του

Page 43: ΣΗΜΕΙΩΣΕΙΣ ΦΙΛΟΣΟΦΙΑΣ I

εμπειρισμού τόσο στη φιλοσοφία όσο και στον πολιτισμό γενικότερα.

ΣΚΕΠΤΙΚΙΣΜΟΣ

Βλέπουμε λοιπόν ότι η γνωσιολογία του Λόκ λίγο πολύ εγκαθιστά έναν υγιή και

αναπόφευκτο

σκεπτικισμό (την αμφιβολία δηλαδή σχετικά με την εμβέλεια του επιστημονικού λόγου)

στην

καρδιά της διανοητικής μας ζωής την ίδια στιγμή βέβαια επιμένει στην αναγκαιότητα της

πειραματικής επιστήμης ως μοναδικής πηγής θεωρητικών αληθειών περί του φυσικού

κόσμου.

Και ο συμβιβασμός αυτός είναι χαρακτηριστικός της μετριοπάθειας και της

αυτοσυγκράτησης του κοινού νου που χαρακτηρίζει τον θεμελιωτή του νεότερου

εμπειρισμού.

ΕΜΠΕΙΡΙΚΟΣ ΙΔΕΑΛΙΣΜΟΣ ΤΟΥ ΜΠΕΡΚΛΕΫ ΓΝΩΣΙΟΛΟΓΙΚΟΣ ΣΚΕΠΤΙΚΙΣΜΟΣ ΚΑΙ

ΨΥΧΟΛΟΓΙΚΗ ΕΡΜΗΝΕΙΑ ΤΗΣ ΑΙΤΙΟΤΗΤΑΣ ΣΤΗ ΣΚΕΨΗ ΤΟΥ ΧΙΟΥΜ

Ο Μπέρκλεϋ θεωρεί ανυπόστατο το διαχωρισμό ανάμεσα σε πρωτογενείς και

δευτερογενείς

ιδιότητες των σωμάτων. Η "πραγματικότητα" είναι η αντίληψη μας για την πραγματικότητα.

Και

η αντίληψη αυτή αποτελείται πάντοτε από παρατηρήσεις του επιμέρους του χωροχρονικά

προσδιορισμένου και του μερικού. Ο όρος υλικό σώμα είναι μια νοητική συντομογραφία

που δεν

αντιστοιχεί ούτε αντανακλά τίποτε το αυθύπαρκτο πέρα από τα αισθητήρια μας. Ένα

εμπειρικό

πράγμα είναι με τη πλήρη και αυστηρή έννοια ότι έχει ποτέ παρατηρηθεί σχετικά με αυτό.

Τα

πράγματα είναι συμπλέγματα ιδεών.

ΧΙΟΥΜ

Δεν επικροτεί τον Μπέρκλεϋ και υιοθετεί μια ενδιάμεση θέση ανάμεσα στο διστακτικό και

μετριοπαθή ρεαλισμό του Λοκ και τον άκρατο ιδεαλισμό του πρώτου. Αυτή η θέση είναι ο

σκεπτικισμός. Είναι δυνατόν αλλά αναπόδεικτο ότι υπάρχει ένας υλικός κόσμος καθαυτό

Page 44: ΣΗΜΕΙΩΣΕΙΣ ΦΙΛΟΣΟΦΙΑΣ I

ανεξάρτητος από τα αισθητήρια μας. Από κει και πέρα σκοπός του επιστημονικού νου δεν

είναι

να εισχωρήσει σε αυτή την υπερβατική πραγματικότητα - πράγμα αδύνατον με τα

εμπειρικά

μέσα που διαθέτουμε. Η επιστημολογία του Χιουμ αναφέρει την επιστήμη αποκλειστικά

στο

υποκείμενο. Η επιστήμη μας διδάσκει τους σταθερούς τρόπους και τύπους υπό τους

οποίους έχει

εμφανιστεί στο ανθρώπινο είδος ο κόσμος κατά την διάρκεια της συλλογικής του εμπειρίας.

Δεν

αναζητεί τίποτε έξω από αυτή.

ΑΝΑΛΥΣΗ ΤΗΣ ΕΝΝΟΙΑΣ ΤΗΣ ΑΙΤΙΟΤΗΤΑΣ

Η αιτιακή σχέση είναι η χρονική διαδοχή και η συντοπία μέσα στο χώρο ανάμεσα σε δυο

συμβάντα Α και Β η οποία έχει παρατηρηθεί με μεγάλη συχνότητα μέσα στην εμπειρία μας.

Τα

δυο συμβάντα είναι από μόνα τους δύο απόλυτα χωριστές και αυτοτελείς αντιληπτικές

στιγμές.

Πρόκειται για δύο παρατηρήσεις που δεν συνδέονται με τίποτε άλλο εκτός από το γεγονός

ότι

γειτνιάζουν μέσα στη μνήμη μας. Τις θυμόμαστε να ακολουθούν η μία την άλλη καθώς

ερχόμαστε σε επαφή μέσω των αισθήσεων με τον κόσμο. Η γειτνίαση αυτή (τοπική και

χρονική)

εμβάλλει λοιπόν στον νου μας μια ψυχολογική διάθεση να προσδοκούμε ότι μόλις

εμφανισθεί το Α θα ακολουθήσει και το Β. Δημιουργείται έτσι μια νοητική συνήθεια (habit

of

mind), πάνω στην οποία χτίζεται μια υποκειμενική πίστη (belief) ως προς την αιτιώδη

σύνδεση αυτών των φαινομένων (η φωτιά που καίει και το κερί που λιώνει όταν την

πλησιάζει).

Αυτό ακριβώς είναι και το θεμέλιο της αιτιακής σχέσης. Καμιά άλλη (αναγκαία,

αντικειμενική)

συνάφεια ανάμεσα στο Α και στο Β δεν μας αποκαλύπτεται ούτε είναι δυνατόν να γνωσθεί.

Μια αιτιακή σχέση ("η φωτιά είναι η αιτία που λιώνει το κερί") δεν έχει καμιά άλλη ισχύ

πέρα

Page 45: ΣΗΜΕΙΩΣΕΙΣ ΦΙΛΟΣΟΦΙΑΣ I

από το σώμα των παρατηρήσεων εκ των οποίων συνάγεται. Όσο μεγαλύτερη είναι η

συχνότητα

της συμπαρουσίας του Α και του Β, τόσο μεγαλύτερη είναι η πιθανότητα να παρατηρούμε

μιαν

αιτιώδη διαπλοκή. Η ισχύς όλων των θεωρητικών γενικεύσεων σχετικά με τον κόσμο

είναι λοιπόν καθαρά πιθανολογική.

ΔΙΑΙΣΘΗΤΙΚΗ ΑΡΧΗ ΤΗΣ ΟΜΟΙΟΜΟΡΦΙΑΣ ΤΗΣ ΦΥΣΗΣ

Η ανθρώπινη αντιληπτικότητα είναι πεπερασμένη. Τίποτε στο μέλλον δεν είναι προβλεπτό

με

βεβαιότητα. Το μέλλον βρίσκεται εξ ορισμού εκτός εμπειρίας, άρα το τι θα παρατηρηθεί

στο

μέλλον είναι πλήρως αδιάγνωστο. Όταν προβλέπουμε το μέλλον ενεργούμε συμφωνά με

τις

ψυχολογικές προσδοκίες που έχουν δημιουργηθεί μέσα μας από την εμπειρία του

παρελθόντος.

Εικάζουμε απλώς ότι η δομή των φυσικών πραγμάτων θα εξακολουθήσει να είναι η ίδια με

αυτήν που έχουμε συνηθίσει και η οποία μας ωφελεί. Αυτή είναι και η διαισθητική αρχή

της

"ομοιομορφίας της φύσης" (principle of the uniformity of nature). Πρόκειται για μια

υπόθεση που δεν μπορεί να αποδειχθεί. Πάνω σε αυτήν όμως στηρίζονται αναγκαστικά

τόσο ο

κοινός όσο και ο επιστημονικός νους, χρησιμοποιώντας την επαγωγική γενίκευση για να

προβλέψει ένα μελλοντικό συμβάν. Το πρόβλημα εδώ είναι ότι η γνώση μας είναι πάντοτε

μια

επαγωγική γενίκευση (inductive generalization) πάνω στην εμπειρία του παρελθόντος. Τη

χρησιμοποιούμε όμως για να συμπεράνουμε κάτι για το μέλλον: γνωρίζουμε πάντα

αναδρομικά,

αλλά πράττουμε προδρομικά (προλαμβάνουμε δηλαδή στο μυαλό μας τα μέλλοντα). Ένα

εμπειρικό συμπέρασμα ("όλοι οι κύκνοι είναι λευκοί") συνοψίζει την παρελθούσα εμπειρία.

Με

βάση αυτό όμως εμείς προβλέπουμε ότι ο επόμενος κύκνος που θα συναντήσουμε θα είναι

Page 46: ΣΗΜΕΙΩΣΕΙΣ ΦΙΛΟΣΟΦΙΑΣ I

ομοίως λευκός. Σήμερα όμως γνωρίζουμε πια ότι υπάρχουν και μαύροι κύκνοι (στη φύση,

και

όχι μόνο στη φαντασία ου Τσαϊκόφσκι).

ΘΕΜΕΛΙΩΔΕΣ ΓΝΩΣΙΟΛΟΓΙΚΟ ΠΡΟΒΛΗΜΑ

Η συνέχεια μεταξύ παρελθόντος και μέλλοντος την οποία υποθέτει ο ανθρώπινος νους

είναι

παντελώς αθεμελίωτη. Δεν έχει ούτε εμπειρική ούτε λογική βάση, αλλά η

αποτελεσματικότητα

των εμπειρικών μας γνώσεων ως οδηγών για τη μελλοντική μας συμπεριφορά εξαρτάται

από

εκείνη. Αυτό είναι και το θεμελιώδες γνωσιολογικό πρόβλημα της επαγωγής (problem of

induction).

Στην πράξη βέβαια ούτε αμφιβάλλουμε (ούτε ενδείκνυται να αμφιβάλλουμε) ία το αν θα

ανατείλει αύριο ο ήλιος. Ο σκεπτικισμός όμως (η θεμελιακή δηλαδή αμφιβολία σχετικά με

τη

φύση των πραγμάτων) ξεπερνιέται μόνο πρακτικά. Αν ο ζήτημα ερευνηθεί

γνωσιοθεωρητικά,

τότε αντιλαμβανόμαστε ότι δεν έχουμε το δικαίωμα καμιάς βεβαιότητας, ούτε γι' αυτό το

θέμα.

Αναφορικά με όλα ανεξαιρέτως τα δεδομένα της εμπειρίας (matters of fact) πιθανολογούμε

και

μόνο. Ο λόγος είναι πάντοτε υπηρέτης μιας πρακτικής προσταγής.

TH. KUHN

1. Τι είναι επιστήμη;

Η επιστήμη είναι ένα πλέγμα κοινωνικών θεσμών, η συστηματική οργάνωση μιας

συγκεκριμένης

πολιτιστικής δραστηριότητας με τα δικά της αξιολογικά πρότυπα, παιδευτικές

προδιαγραφές και

δεοντολογία διαπροσωπικών σχέσεων.

2. Παράδειγμα:

Η επιστημονική πρόοδος σημαδεύεται από την εναλλαγή γενικών ερμηνευτικών σχημάτων

της

Page 47: ΣΗΜΕΙΩΣΕΙΣ ΦΙΛΟΣΟΦΙΑΣ I

φυσικής πραγματικότητας, το καθένα με μιαν ιδιαίτερη προβληματική και μεθοδολογία

που

καθορίζει την ερευνητική συμπεριφορά αυτών που εργάζονται υπό την σκέψη του. Αυτά

ακριβώς είναι τα παραδείγματα γύρω από τα οποία συγκροτείται κάθε φορά η

επιστημονική

κοινότητα, οι συλλογικοί τρόποι δια των οποίων η τελευταία θεάται το σύμπαν. Το

παράδειγμα πρέπει να νοηθεί ως πηγή ερευνητικών στρατηγημάτων , υπαγορεύει δηλαδή

πειραματικούς δρόμους που στοχεύουν να διευρύνουν την εμπειρική βάση των

θεωρητικών του

προπαραδοχών , να τις φέρουν με άλλα λόγια σε μεγαλύτερη αρμογή με τις

παρατηρημένες

φυσικές διεργασίες.

3. Στόχοι του επιστήμονα:

Η δουλειά του επιστήμονα που στεγάζεται από το κυρίαρχο παράδειγμα στοχεύει στην

λύση

αποριών η αινιγμάτων που προκύπτουν ακριβώς από τη (φυσιολογική και αναμενόμενη)

αδυναμία της τρέχουσας θεωρίας να εξηγήσει ολόκληρο το εύρος των εμπειρικών

φαινομένων

στα οποία αναφέρεται. Τέτοιου είδους έρευνα είναι μια μορφή συμπληρωματικών

εργασιών με

στόχο να ενταχθούν στο θεωρητικό υπόδειγμα εκείνες οι πτυχές της φυσικής

πραγματικότητας

που εξακολουθούν να του αντιστέκονται , μια δουλειά όντως πρωτότυπη και διεισδυτική

αφού

διευρύνει το εμπειρικό περιεχόμενο και συνεπώς την ελεγξιμότητα της θεωρίας αν και

τελικά το

μεγαλύτερο επίτευγμα της είναι η ανακάλυψη βαθύτερων και πολύ πιο ριζικών

προβλημάτων

από εκείνα που κατάφερε να υπερβεί η συγκεκριμένη πειραματική της πρακτική.

4. Κανονική Επιστήμη:

Στον βαθμό που το κυρίαρχο παράδειγμα δεν έχει ακόμα κλονισθεί, η προσέγγιση των

Page 48: ΣΗΜΕΙΩΣΕΙΣ ΦΙΛΟΣΟΦΙΑΣ I

ερευνητών που δραστηριοποιούνται υπό την στέγη του είναι κατά κύριο λόγο

επαληθευτική, με

τα πειραματικά αποτελέσματα να προσθέτουν στον βαθμό πιθανότητας των θεωρητικών

του

υποθέσεων. Εξασφαλίζεται έτσι μια μακρά περίοδος θεωρητικής σταθερότητας και αυτή

αντιπροσωπεύει τη φάση εκείνη στην ανέλιξη των φυσικών μας γνώσεων που

χαρακτηρίζεται

ως Κανονική Επιστήμη, μια περίοδο δηλαδή θεωρητικού εφησυχασμού και λίγο πολύ

αβασάνιστης πίστης στις κεντρικές παραδοχές του συστήματος.

5. Ανωμαλίες:

Τα φαινόμενα που αντιβαίνουν στις προβλέψεις του παραδείγματος λέγονται

ανωμαλίες. Η λανθάνουσα αυτή αντίφαση ανάμεσα στις θεωρητικές προσδοκίες του

κυρίαρχου

μοντέλου και τα αποτελέσματα της συστηματικής έρευνας που οργανώνεται υπό την αιγίδα

του

είναι η ουσιώδης ένταση που διαπερνάει την επιστημονική δραστηριότητα και εξασφαλίζει

την

προοδευτική της δυναμική ως ενός συστήματος σκέψης που δεν μένει προσκολλημένο σε

ιερές

και αιώνιες πίστεις αλλά αναζητεί διαρκώς νέους τρόπους για τη σύλληψη της φύσης.

6. Κρίση:

Μια κρίση εξουσίας , τα δομικά χαρακτηριστικά της οποίας είναι εντελώς ανάλογα με

εκείνα

των πολιτικών κρίσεων που προηγούνται της αλλαγής πολιτικών και κοινωνικών

καθεστώτων.

7. Προϋποθέσεις Ανατροπής :

Δυο είναι οι προϋποθέσεις ανατροπής του παλαιού παραδείγματος. Πρώτον μια κρίσιμη

μάζα

αρνητικών πειραματικών δεδομένων που καθιστά αδύνατη την επιστροφή στην θεωρητική

ορθοδοξία του παρελθόντος. Δεύτερον η διατύπωση ενός εναλλακτικού παραδείγματος και

η

Page 49: ΣΗΜΕΙΩΣΕΙΣ ΦΙΛΟΣΟΦΙΑΣ I

σύμπηξη μιας ομάδας ερευνητών αφοσιωμένων σε εντελώς νέες παραδοχές για τη δομή

του

κόσμου ,που διεκδικούν πλέον την εξουσία στην επιστημονική κοινότητα ,αγωνίζονται

δηλαδή

για να κυριαρχήσουν στα ερευνητικά κέντρα και τα εκπαιδευτικά ιδρύματα που

διαμορφώνουν

την "κοινή συνείδηση" σχετικά με το τι είναι επιστήμη. Μεσολαβεί ένας πόλεμος

παραδειγμάτων που χωρίζει την επιστημονική κοινότητα έως ότου γείρει αποφασιστικά η

πλάστιγγα υπέρ του νέου παραδείγματος με τη συμβολή τόσο αποφασιστικών πειραμάτων

όσο

και της παιδευτικής δραστηριότητας των εκπροσώπων του νέου παραδείγματος που έχουν

αλώσει θέσεις κλειδιά μέσα στον μηχανισμό του παλαιού συστήματος. Ένα παράδειγμα δεν

πεθαίνει ποτέ μόνο του αλλά χάνει την ισχύ του σε σύγκριση με ένα άλλο που αγωνίζεται

να το

εκτοπίσει μέσα από μια μεταβατική περίοδο επιστημονικής έρευνας εκτάκτου ανάγκης. Οι

πρώην ανωμαλίες αρχίζουν πλέον να συγκροτούν την νέα θεωρητική κανονικότητα. Κάθε

νέο

παράδειγμα κατασκευάζει μια δική του γλώσσα ,εννοιολογία και αξιωματική χωρίς σημεία

επαφής με την παλαιότερη (θεωρητική ασυμμετρία των παραδειγμάτων).

8. Ασυμμετρία των παραδειγμάτων:

Διακρίνουμε την δυνατότητα δυο εναλλακτικών ορισμών : στην πρώτη περίπτωση με την

ευρεία έννοια του "παραδείγματος" αναφερόμαστε σε ένα σύνολο πολιτισμικών ,

αξιολογικών - ακόμα και θρησκευτικών - επιλογών που σφραγίζουν την κοινή συνείδηση

καθορίζοντας αποφασιστικά τόσο τους στόχους όσο και την μεθοδολογία της

επιστημονικής

έρευνας. Η θεωρητική διαφωνία στην επιστήμη είναι μια σύγκρουση ερμηνευτικών

στάσεων

απέναντι στο πραγματικό. Είναι προφανές ότι η εξωτερική ερμηνεία της επιστήμης

καταλήγει σε

ένα άκρατο σχετικισμό, μια ερμηνεία δηλαδή που εξηγεί την επιστημονική μέθοδο ως

δευτερογενή έκφραση κοινωνικοπολιτικών αναγκαιοτήτων εξωγενών προς αυτήν.

Page 50: ΣΗΜΕΙΩΣΕΙΣ ΦΙΛΟΣΟΦΙΑΣ I

Μια στενότερη ερμηνεία της έννοιας "παράδειγμα" θα το περιόριζε στη δεοντολογία και

στην πρακτική που χαρακτηρίζει εκάστοτε την επιστημονική κοινότητα ως αυτόνομο σώμα

που

θεσπίζει αυτοδικαίως τους κανόνες οι οποίοι κυβερνούν τη συμπεριφορά του χωρίς να

καταναγκάζεται μηχανικά και αναπότρεπτα από την αξιολογική και πολιτιστική συναίνεση

της

περιέχουσας κοινωνίας.

9. Θεωρία των παραδειγμάτων.

Η θεωρία των παραδειγμάτων σηματοδότησε μια αλλαγή παραδείγματος στη φιλοσοφία

της

επιστήμης, μιαν επαναστατική ρήξη στην κατανόηση της ως κοινωνικού φαινομένου, ρήξη

σύμφωνη με τον πολιτικό προβληματισμό μιας ταραγμένης αλλά κοσμογονικής περιόδου.

Στην

εξημερωμένη και εξευγενισμένη εκδοχή της, όμως, θα μπορούσαμε ίσως διαισθητικά να

την

ερμηνεύσουμε ως παραλλαγή της ποππεριανής διαψευσιμότητας, με την έννοια ότι στη

θέση

των πολυάριθμων μικρών ανά πάσα στιγμή επαναστάσεων (τη διαρκή διάψευση των

επιμέρους

προτάσεων και θεωριών)που οραματίζεται ο Πόππερ μπαίνει από τον Κuhn η μία και

μεγάλη

επανάσταση, που ορίζει τη μετάβαση από τη μια στην άλλη εποχή του επιστημονικού

στοχασμού. Το ιστορικό της ίσως περιεχόμενο είναι διευρυμένο, παραμένει όμως μια

θεωρία για

ειδικούς.

ΕΠΙΣΤΡΟΦΗ ΣΤΗΝ ΟΡΘΟΛΟΓΙΚΟΤΗΤΑ ΚΑΙ ΕΣΩΤΕΡΙΚΗ ΑΝΑΛΥΣΗ ΤΗΣ

ΕΠΙΣΤΗΜΟΝΙΚΗΣ ΜΕΘΟΔΟΥ. Η ΘΕΩΡΙΑ ΤΩΝ ΕΡΕΥΝΗΤΙΚΩΝ ΠΡΟΓΡΑΜΜΑΤΩΝ ΤΟΥ

LAKATOS.

1. Προϋπόθεση για την εγκυρότητα μιας θεωρίας:

Περί επιστημονικής μεθόδου είναι ο τρόπος με τον οποίο αυτή συλλαμβάνει και καθιστά

κατανοητή την ανάδειξη και τη διαδοχή των θεωριών όπως μας την παρουσιάζει η ιστορική

Page 51: ΣΗΜΕΙΩΣΕΙΣ ΦΙΛΟΣΟΦΙΑΣ I

έρευνα.. Ταυτόχρονα απορρίπτει την θεωρία των παραδειγμάτων. Ο μεθοδολογικός

ολισμός της

την καθιστά ευάλωτη σε ψυχολογικές ερμηνείες της αλλαγής των θεωρητικών προτύπων

στην

επιστήμη.

2. Ερευνητικά προγράμματα

Κάθε ερευνητικό πρόγραμμα διαμορφώνεται και λειτουργεί σε ένα ωκεανό ανωμαλιών. Θα

πρέπει κατά συνέπεια να θεωρηθεί ότι αποτελείται από δυο βασικά συστατικά . Από ένα

κατ'

αρχήν θεωρητικό πυρήνα Π τον οποίο οι επιστήμονες που συμμετέχουν στο πρόγραμμα

συμφωνούν εκ των προτέρων να θεωρήσουν δεδομένο και άθικτο, όποια κι αν είναι τα

αποτελέσματα της εμπειρικής έρευνας που θα ακολουθήσει . Ο πυρήνας αυτός

περιβάλλεται

περαιτέρω από "μια προστατευτική ζώνη" βοηθητικών υποθέσεων μέσω των οποίων

επιτυγχάνεται η κρίσιμη διασύνδεση του Π με τον τομέα της φύσης στον οποίο αναφέρεται.

Αυτό που ελέγχεται κάθε φορά από το πείραμα είναι τα φυσικά μοντέλα που προκύπτουν

από

την εκάστοτε σύζευξη των βοηθητικών υποθέσεων με τις θεμελιακές προπαραδοχές Π. Η

"προστατευτική ζώνη" αποτελείται λοιπόν από τους διάφορους τρόπους με τους οποίους ο

αρχικός θεωρητικός πυρήνας ενδέχεται να εφαρμοσθεί στη μελέτη του φυσικού πεδίου.

Κάθε

ερευνητικό πρόγραμμα είναι σαφές ότι αποτελείται από μια δέσμη τέτοιων μοντέλων. Η

κρίσιμη

συνθήκη εδώ είναι ότι οι βοηθητικές υποθέσεις και τα μοντέλα που προκύπτουν από αυτές

πρέπει να παράγονται από αυτό που ο LAKATOS ονομάζει μια "θετική ευρετική μέθοδο" .

Αυτή ακριβώς αποτελεί και την καρδιά του κάθε ερευνητικού προγράμματος. Χωρίς μια

ισχυρή

ευρετική η θεωρία παραμένει ένας αφηρημένος τρόπος να βλέπεις απλώς τον κόσμο, μια

οιονεί

μεταφυσική εποπτεία, που δεν εξαργυρώνεται με ένα απτό πειραματικό πρόγραμμα. Η ιδια

η

Page 52: ΣΗΜΕΙΩΣΕΙΣ ΦΙΛΟΣΟΦΙΑΣ I

ιστορική πρακτική της επιστημονικής κοινότητας φαίνεται να διαχωρίζει τα ερευνητικά

προγράμματα που ανά πάσα στιγμή διαγωνίζονται σε ανερχόμενα και κατερχόμενα, σε

προοδευτικά και εκφυλιζόμενα.Η αποτίμηση αυτή αναφέρεται πάντα στον τρόπο με τον

οποίο

ένα ερευνητικό πρόγραμμα διαπλέκεται με την φυσική πραγματικότητα , στις δυνατότητες

που

παρέχει η ευρετική του να κατασκευασθούν φυσικά μοντέλα που διευρύνουν τις

πειραματικές

προοπτικές και συνεισφέρουν στην ανακάλυψη καινοφανών και απρόβλεπτων εμπειρικών

δεδομένων έστω και τέτοιων που δεν είναι δυνατόν να στεγασθούν στις θεωρητικές του

παραδοχές. Το ζητούμενο με άλλα λόγια είναι εάν η ευρετική του παραμένει θετική η έχει

στερέψει.

3. Ad hoc παραδοχές.

Η διαδικασία της επεξεργασίας της προστατευτικής ζώνης του προγράμματος δια της

εισαγωγής

AD HOC βοηθητικών παραδοχών μπορεί θεωρητικά να διαρκέσει έπ' άπειρον. Δεν μπορεί

να αποφασισθεί εκ των προτέρων το ακριβές σημείο όπου η προσθήκη συμπληρωματικών

υποθέσεων, με σκοπό να απορροφηθούν τα αρνητικά πειραματικά δεδομένα, μεταπίπτει

σε

αντιεπιστημονική ιδεοληψία. Δεν είναι δυνατόν η φιλοσοφική αναπαράσταση της

επιστημονικής

μεθόδου να θέσει a priori περιορισμούς στην πρακτική των ίδιων των επιστημόνων. Δεν

είναι

δυνατόν η φιλοσοφία να θεσπίσει καθαρά λογικά κριτήρια για την αποτίμηση της

ορθολογικότητας της συμπεριφοράς των ερευνητών. Βρίσκεται στην απόλυτη διάκριση των

μετεχόντων σε ένα ερευνητικό πρόγραμμα να συνεχίσουν να το φορτώνουν με όλο και πιο

περίπλοκες υποθέσεις η να αποφασίσουν ότι εξαντλήθηκε η δυναμική του.

4. Στιγμιαία ορθολογικότητα.

Εάν το τελικό μας μέτρο είναι η επιστημονική πράξη, δηλαδή με άλλα λόγια η ίδια η ιστορία

της

επιστήμης, τότε θα πρέπει να συμπεράνουμε ότι καμιά θεωρητική επιλογή και πειραματική

Page 53: ΣΗΜΕΙΩΣΕΙΣ ΦΙΛΟΣΟΦΙΑΣ I

επιτυχία δεν επενδύει ένα ερευνητικό πρόγραμμα με "στιγμιαία ορθολογικότητα", όπως

την

αποκαλεί ο LAKATOS, αποκλείοντας διαμιάς ως αντιεπιστημονικές προλήψεις εκείνα που το

ανταγωνίζονται. Η ορθολογικότητα του προγράμματος αναπτύσσεται οργανικά και

βαθμιαία

μέσα στην ερευνητική πρακτική. Οι διάφορες εναλλακτικές θεωρίες συνυπάρχουν σε ένα

πολυσύνθετο τοπίο προωθώντας η καθεμία τη δική της πειραματική προσέγγιση, μέχρις

ότου κάποιες από αυτές να στερέψουν και να εγκαταλειφθούν. Είναι επίσης ιστορικό

γεγονός ότι

ένας επιστήμονας είναι δυνατόν να μετέχει σε περισσότερα από ένα ερευνητικά

προγράμματα.

Αυτό σημαίνει ότι υφίσταται θεωρητική επικοινωνία ανάμεσα τους, πράγμα που διαψεύδει

εκείνη

την ισχυρή έννοια της ασυμβατότητας που διαποτίζει τις ερμηνείες του Kuhn.

5. Η σκέψη του LAKATOS.

Η σκέψη του Lakatos πάντως αποτέλεσε αποφασιστικό σταθμό στη .σύγχρονη φιλοσοφία

της

επιστήμης, επειδή ερμήνευσε την επιστημονική θεωρία ως σύστημα τελικά πρακτικής

συμπεριφοράς μέσα στο ευρύτερο κοινωνικό της πλαίσιο, την ίδια στιγμή που αποτύπωνε

τα

εγγενή, "εσωτερικά", χαρακτηριστικά της επιστημονικότητας, εκείνα που τη διακρίνουν από

τις

άλλες εκφάνσεις του πολιτισμού. Ο ιστορισμός του Lakatos φιλοδοξεί να είναι μια

"εσωτερική"

αποτίμηση των μεθοδολογικών χαρακτηριστικών που ορίζουν την ιδιαιτερότητα της

επιστημονικής δραστηριότητας (μια προσέγγιση που επονομάζεται "internalism"), την ίδια

στιγμή που στοχεύει στην ακριβή αναπαράσταση της ιστορικής της πορείας. Δεν παύει

όμως να

αληθεύει ότι ο ιστορισμός περιέχει από την ίδια του τη φύση μια λανθάνουσα

αντιεπιστημονική

αιχμή, στον βαθμό που αποκαλύπτει ως ιδεολογική φενάκη τους ισχυρισμούς περί της

Page 54: ΣΗΜΕΙΩΣΕΙΣ ΦΙΛΟΣΟΦΙΑΣ I

αξιολογικής ουδετερότητας των επιστημόνων, και της απροκατάληπτης και ανιδιοτελούς

θητείας

τους στο ιδεώδες της αντικειμενικής αλήθειας πέρα από κάθε κοινωνικό καταναγκασμό ή

συμφέρον. Κάθε προσπάθεια να ενταχθεί οργανικά η επιστήμη στο συνεχές του

πολιτισμού, ως

μια από τις πολλές προσπάθειες να εμβάλει ο άνθρωπος νόημα στην πραγματικότητα,

οδηγεί

(όσους φραγμούς και αν βάλει κανείς στη ροπή αυτή, όπως ζητάει ο Lakatos) στην ιδεατού

αλληλοκαθορισμού κοινωνικής συνείδησης και επιστημονικής θεωρίας. Το έσχατο

αποτέλεσμα

είναι να αναδειχθεί ο ρόλος της επιστήμης ως κρίσιμου μηχανισμού για την ολοκλήρωση

της

κοινωνίας, ως μοχλού δηλαδή στα χέρια της κατεστημένης εξουσίας.

Η ΕΠΙΣΤΗΜΗ ΩΣ ΣΥΣΤΗΜΑ ΕΞΟΥΣΙΑΣ : Η ΡΙΖΙΚΗ ΑΠΑΞΙΩΣΗ ΤΗΣ ΟΡΘΟΛΟΓΙΚΟΤΗΤΑΣ

ΣΤΗ ΣΚΕΨΗ ΤΟΥ FEYERABEND

Στο τελευταίο τέταρτο του 20ού αιώνα η ένταξη της επιστημονικής κοινότητας στο ιστάμενο

θεσμικό σύστημα και η λειτουργία της ως εντολοδόχου της κοινωνικής εξουσίας είχε, ιδίως

στον

ανεπτυγμένο κόσμο, ολοκληρωθεί με την ποδηγέτηση της έρευνας από τις μεγάλες

εταιρείες και

τις κεντρικές κυβερνήσεις. Από τη σκοπιά αυτή τα ερευνητικά προγράμματα που

συγκεντρώνουν πιθανότητες για τη χρηματοδότηση τους και τη γενικότερη θετική

αξιολόγηση

τους είναι εκείνα που επιτυχώς υπηρετούν την κερδοσκοπική δραστηριότητα των μεγάλων

εμπορικών συγκροτημάτων και την πολεμική βιομηχανία. Η εισβολή ταυτόχρονα της

τεχνολογίας στην κοινωνική ζωή επιτρέπει αδιανόητες παλαιότερα δυνατότητες ελέγχου

και

καταναγκασμού των πολιτών. Η σωρευτική συνέπεια όλων αυτών των εξελίξεων είναι μια

ογκούμενη εναντίωση τόσο στην επιστήμη, ως οργανωμένο πλέον κρατικό θεσμό, όσο και

στην

επιστημονική νοοτροπία γενικά, ως μια διανοητική έξη που γυμνώνει τον άνθρωπο από τις

Page 55: ΣΗΜΕΙΩΣΕΙΣ ΦΙΛΟΣΟΦΙΑΣ I

διαισθητικές και ποιητικές πτυχές της προσωπικότητας του και τον ευτελίζει σε άβουλο

ενεργούμενο στα χέρια απρόσωπων εξουσιαστικών κέντρων. Η δυσαρέσκεια αυτή, που

κορυφώθηκε κατά τη δεκαετία του 1960 και συντηρείται έκτοτε, δεν άργησε να επηρεάσει

και

την καθαρή φιλοσοφία της επιστήμης και να συνενωθεί με το ρεύμα της "εξωτερικής"

(externalist) ερμηνείας της επιστημονικής μεθόδου, την οποία έτσι κι αλλιώς είχε

πυροδοτήσει ο

ιστορισμός. Με τον τρόπο αυτό εσωγενείς και εξωγενείς εντάσεις, η κρίση του θετικισμού

και η

επαναστατική έξαψη που σφράγιζε το "πνεύμα της εποχής", λειτούργησαν από κοινού για

να

απαξιωθεί η επιστήμη και το τεχνοκρατικό ήθος. Κανένας φιλόσοφος δεν εκφράζει με πιο

εκρηκτικό τρόπο την εξέγερση αυτή από τον Αυστριακό Paul Feyerabend (1924-1994).

Η σκέψη του Feyerabend διαμορφώθηκε αρχικά στο πλαίσιο της ποππεριανής κριτικής του

θετικισμού, με ιδιαίτερη όμως αιχμή την προσπάθεια να κατοχυρωθεί ο ρεαλισμός ως το

αναγκαίο υπόβαθρο της επιστημονικής διαδικασίας. Η συνάντηση του με τη φιλοσοφία του

Wittgenstein καθώς και η εντεινόμενη συναίσθηση της καταλυτικής παρουσίας της θεωρίας

μέσα στην παρατήρηση τον οδήγησαν σταδιακά σε μια γνωσιολογία που χαρακτηρίζεται ως

contextualism, την ιδέα δηλαδή ότι το νόημα, και τελικά η αλήθεια, διαμορφώνεται μέσα

στο

πλαίσιο των παραδοχών που συγκροτούν την κοινή συνείδηση μιας δεδομένης ανθρώπινης

ομάδας, μέσα σε ένα συγκεκριμένο πρότυπο ζωής (την υπαρκτική "παράδοση", όπως την

αποκαλεί). Η τελική κατάληξη της προβληματικής αυτής ήταν η κατανόηση της επιστήμης

ως

μιας ανθρώπινης δραστηριότητας δίπλα σε μια πληθώρα άλλων, μιας δραστηριότητας

που δεν διαθέτει υπέρτερο κύρος σε σχέση με τις υπόλοιπες. Τις αξιόλογες τεχνολογικές

επιτυχίες της τις εξαργυρώνει με τη συμμετοχή της στο σύστημα της εξουσίας, και

μετατρέπεται

έτσι σε κεντρικό στυλοβάτη ενός αλλοτριωτικού και καταπιεστικού κοινωνικού

σχηματισμού.

Page 56: ΣΗΜΕΙΩΣΕΙΣ ΦΙΛΟΣΟΦΙΑΣ I

Η επιστήμη δεν έχει καμία εγγενή αξία, καμία προνομιακή σχέση με την ουσία των

πραγμάτων, ούτε είναι φορέας διανοητικών και αξιολογικών προτύπων υποχρεωτικών

για όλους. Αν ο χωρισμός της εκκλησίας από το κράτος ήταν το θεμελιακό αίτημα των

μεγάλων

επαναστάσεων του 18ου αιώνα, ο χωρισμός της επιστήμης από το κράτος είναι η

προϋπόθεση για την ελευθερία στο τέλος του 20οΰ. Συνολικά λοιπόν το δυτικό

πολιτισμικό πρότυπο που στηρίζεται στην τεχνοκρατική ορθολογικότητα συνιστά μιαν

οπισθοδρόμηση του πολιτισμού. Η ελεύθερη κοινωνία είναι εκείνη που συνίσταται από

μια πλειάδα ξεχωριστών "παραδόσεων", από τις πιο νεωτερικές, ριζοσπαστικές,

ανατρεπτικές έως τις πιο οπισθοδρομικές, ανορθόλογες, συμβατικές κτλ.

Μια απροκατάληπτη μελέτη της ιστορίας της επιστήμης θα δείξει, πιστεύει ο Feyerabend,

ότι και

αυτή η σημερινή ορθολογική στάση για την οποία τόσο υπερηφανευόμαστε αναπτύχθηκε

πάνω

στη βάση μιας ριζικής μεθοδολογικής πολλαπλότητας. Καμιά μέθοδος δεν ήταν

υποχρεωτική

και καμιά δεν ήταν απαγορευμένη στην προσπάθεια για την πιο λειτουργική εμπλοκή με

τον κόσμο που χαρακτήρισε τη νεωτερική επιστήμη.

Όπως και να έχει το πράγμα, η συλλογιστική του Feyerabend κατάφερε να αγγίξει μιαν

εξόχως

προβληματική πτυχή στη σημερινή θεσμοποίηση της επιστήμης, τη μετατροπή της δηλαδή

σε

συστατικό ενός συστήματος κοινωνικού καταναγκασμού, μια πραγματικότητα που είχε

όντως

συσκοτισθεί από την επίσημη, ακαδημαϊκή θεωρία και την ιστορία. Παρά τις ακρότητες της,

λοιπόν, έπιασε τον σφυγμό μιας νέας κοινωνικής πραγματικότητας και αποτέλεσε μία από

τις

ιδεολογικές επιρροές που σφράγισαν τα σύγχρονα οικολογικά και αντιτεχνολογικά

κινήματα.

ΚΑΡΛ ΠΟΠΠΕΡ

1. Το πρόβλημα της επαγωγής όπως το είχε ορίσει ο Χιουμ είναι άλυτο.

Page 57: ΣΗΜΕΙΩΣΕΙΣ ΦΙΛΟΣΟΦΙΑΣ I

2. Οι τέσσερις κατευθύνσεις διενέργειας πειραματικού ελέγχου μιας θεωρίας.

Μπορούμε, αν θέλουμε, να ξεχωρίσουμε τέσσερις διαφορετικές κατευθύνσεις στις οποίες

θα

μπορούσε να διενεργηθεί ο πειραματικός έλεγχος μιας θεωρίας πρώτον: ή λογική σύγκριση

των

συμπερασμάτων μεταξύ τους: μ' αυτήν ελέγχεται ή εσωτερική συνέπεια του συστήματος.

Δεύτερον: ή διερεύνηση της λογικής μορφής της θεωρίας, με σκοπό τον προσδιορισμό του

αν

έχει χαρακτήρα εμπειρικής επιστημονικής θεωρίας ή αν είναι ταυτολογική. Τρίτον: ή

αντιπαραβολή με άλλες θεωρίες με κύριο σκοπό να προσδιοριστεί αν ή θεωρία, αφού

επιζήσει

μετά από τους διάφορους πειραματικούς ελέγχους, αποτελεί μια επιστημονική πρόοδο. Και

τελικά υπάρχει 6 έλεγχος της θεωρίας με βάση τις εμπειρικές εφαρμογές των

συμπερασμάτων

πού απορρέουν απ' αυτήν.

3. Το σύστημα που ονομάζεται εμπειρική επιστήμη έχει την πρόθεση να αναπαραστήσει

μόνο

ένα κόσμο τον πραγματικό η τον κόσμο της εμπειρίας μας.

4. Τρία αιτήματα ικανοποίησης του εμπειρικού θεωρητικού συστήματος:

Συνθετικό έτσι ώστε να μπορεί να αντιπροσωπεύει έναν μη αντιφατικό δυνατό κόσμο.

Κριτήριο

οροθέτησης, δηλαδή δεν πρέπει να είναι μεταφυσικό αλλά να παριστάνει ένα κόσμο

δυνατής

εμπειρίας.

Τρίτον πρέπει να είναι ένα σύστημα που να ξεχωρίζει με κάποιο τρόπο από άλλα τέτοια

συστήματα ως εκείνο που αναπαριστά το δικό μας εμπειρικό κόσμο.

5. Κατά τον Πόππερ οι θετικιστές ερμηνεύουν το πρόβλημα της οροθέτησης με

νατουραλιστικό τρόπο, το ερμηνεύουν σαν να ήταν πρόβλημα της φυσικής επιστήμης. Αντί

να

προτάσσουν μια κατάλληλη σύμβαση προσπαθούν να ανακαλύψουν μια διάφορα που

υπάρχει

Page 58: ΣΗΜΕΙΩΣΕΙΣ ΦΙΛΟΣΟΦΙΑΣ I

στη φύση των πραγμάτων ανάμεσα στην εμπειρική επιστήμη και στη μεταφυσική.

6. Ο βασικός λόγος που απορρίπτει την επαγωγική λογική είναι ακριβώς ότι δεν δίνει κατά

την γνώμη του ένα κατάλληλο διακριτικό γνώρισμα του εμπειρικού, του μη μεταφυσικού

χαρακτήρα ενός θεωρητικού συστήματος δηλαδή του κριτηρίου της οροθέτησης.

7. Ο μόνος τρόπος υποστήριξης των προτάσεων του κατά τον ίδιο είναι η ανάλυση τ6ων

λογικών συνεπειών τους: να υποδειχτεί η γονιμότητας τους - η δύναμη τους

διασαφηνίσουν τα

γνωσιολογικά προβλήματα. Πρώτο καθήκον της λογικής της γνώσης είναι η προώθηση μιας

έννοιας της εμπειρικής επιστήμης με σκοπό την όσο γίνεται μεγαλύτερη διασάφηση της

γλωσσικής χρήσης και τη χάραξη μιας καθαρής γραμμής για την οροθέτηση ανάμεσα στην

επιστήμη και στις μεταφυσικές ιδέες ακόμη και αυτές έχουν συντείνει στην πρόοδο της

επιστήμης σ' όλες τις φάσεις της ιστορίας της.

8. Η εμπειρία παρουσιάζεται σαν μια χαρακτηριστική μέθοδος με την οποία ένα θεωρητικό

σύστημα μπορεί να ξεχωριστεί από άλλα ,έτσι η εμπειρική επιστήμη φαίνεται να

καθορίζεται όχι

μόνο από τη λογική της μορφή, αλλά και από τη χαρακτηριστική της μέθοδο. Τη θεωρία της

γνώσης που έχει ως έργο την ανάλυση αυτής της μεθόδου η διαδικασίας μπορούμε να την

περιγράψουμε σα μια θεωρία της εμπειρικής μεθόδου - μια θεωρία για το τι συνήθως

ονομάζεται

εμπειρία.

9. Το σύστημα που ονομάζεται εμπειρική επιστήμη έχει την πρόθεση να αναπαραστήσει

μόνο

έναν κόσμο: το πραγματικό η τον κόσμο της εμπειρίας μας.

10. Οι θεωρίες δεν είναι ποτέ εμπειρικά επαληθεύσιμες. Εάν κανείς ενδιαφέρεται

αποκλειστικά για την εμπειρική επαλήθευση μιας θεωρίας θα βρει το τρόπο να την

επαληθεύσει.

Δεν υπάρχει θεωρία που δεν επαληθεύεται.

11. Ένα σύστημα χαρακτηρίζεται ως εμπειρικό η επιστημονικό μόνο αν έχει την δυνατότητα

να ελέγχεται από την εμπειρία. Αυτό που πρέπει να πάρουμε ως κριτήριο διαχωρισμού

(βάσει

Page 59: ΣΗΜΕΙΩΣΕΙΣ ΦΙΛΟΣΟΦΙΑΣ I

μιας ασυμμετρίας που τις διέπει)δεν είναι η επαληθευσιμότητα αλλά η διαψευσιμότητα

του και

πρέπει ένα σύστημα να μπορεί να απορριφθεί από την εμπειρία.

12. Μια υποκειμενική εμπειρία ή ένα συναίσθημα βεβαιότητας, δεν μπορεί ποτέ να

αιτιολογήσει μια επιστημονική πρόταση και ότι μέσα στην επιστήμη δεν μπορεί να παίξει

κανένα

ρόλο έκτος από το να αποτελέσει το αντικείμενο μιας εμπειρικής ερευνάς (λ.χ.,

ψυχολογικής).

"Όσο έντονο κι αν είναι αυτό το συναίσθημα της βεβαιότητας δεν μπορεί ποτέ να

αιτιολογήσει

μια πρόταση. "Έτσι, μπορεί να είμαι απόλυτα πεπεισμένος για την αλήθεια μιας πρότασης,

βέβαιος για τη μαρτυρία της αισθητηριακής μου αντίληψης, συντριμμένος από την ένταση

της

εμπειρίας μου: κάθε αμφιβολία μπορεί να μου φαίνεται παράλογη. Άλλα μπορεί αυτό να

αποτελέσει έστω και τον ελάχιστο λόγο για να δεχτεί ή επιστήμη την πρόταση μου; Μπορεί

κάποια πρόταση να αιτιολογηθεί από το γεγονός ότι ο Κ. Κ. Ρ. έχει πειστεί τελείως για την

αλήθεια της; Ή απάντηση είναι "Όχι'· οποιαδήποτε άλλη απάντηση θα ήταν ασυμβίβαστη

με την

ιδέα της επιστημονικής αντικειμενικότητας. Ακόμη και το γεγονός, πού για μένα είναι τόσο

καλά

θεμελιωμένο, ότι έχω την εμπειρία του συναισθήματος.

13. Αν επιμείνουμε στην απαίτηση μας για την αντικειμενικότητα των επιστημονικών

προτάσεων, τότε οι προτάσεις εκείνες πού ανήκουν στην εμπειρική βάση της επιστήμης

πρέπει

να είναι επίσης αντικειμενικές, δηλαδή να υπάρχει γι' αυτές δυνατότητα διϋποκειμενικού

έλεγχου. Ακόμη ο διϋποκειμενικός έλεγχος πάντα σημαίνει ότι από προτάσεις πού

πρόκειται να

ελεγχθούν πραγματικά, απορρέουν παραγωγικά άλλες προτάσεις πού μπορούν επίσης να

ελεγχθούν.

14. Τα συστήματα ή οι θεωρίες ελέγχονται πειραματικά, με το να παραγάγουμε από αυτά

Page 60: ΣΗΜΕΙΩΣΕΙΣ ΦΙΛΟΣΟΦΙΑΣ I

άλλες προτάσεις μικρότερου βαθμού γενικότητας. Αυτές οι προτάσεις, με τη σειρά τους,

μια και

προέρχονται από προτάσεις πού μπορεί να ελεγχθούν διυποκειμενικά, πρέπει μπορούν να

ελεγχθούν με τον ίδιο τρόπο· και το ίδιο μπορεί να συνεχίζεται έπ' άπειρον.

15. Ή παιδαγωγική μέθοδος έλεγχου δεν μπορεί να θεμελιώσει ή να αιτιολογήσει τις

προτάσεις πού ελέγχονται· ούτε και έχει τέτοιο σκοπό. "Έτσι δεν υπάρχει ο κίνδυνος

άπειρης

αναδρομής.

16. Το γεγονός ότι τα πειράματα δεν μπορούν να συνεχίζονται έπ' άπειρον δεν

συγκρούεται με το αίτημα μου για τη δυνατότητα ελέγχου μιας επιστημονικής πρότασης.

Γιατί

δεν απαιτείται για κάθέ επιστημονική πρόταση να πρέπει να έχει όντως ελεγχθεί πριν γίνει

αποδεκτή. Αυτό πού ζητείται μόνο είναι ότι για κάθε τέτοια πρόταση πρέπει να υπάρχει ή

δυνατότητα πειραματικού ελέγχου. Με άλλα λόγια δεν είναι δεκτή η άποψη ότι υπάρχουν

προτάσεις στην επιστήμη τις οποίες πρέπει να δεχτούμε ως αληθινές απλώς και μόνο

επειδή δεν

φαίνεται να είναι δυνατόν, από λογικής πλευράς να ελεγχθούν πειραματικά.

17. Η Ποππεριανή μεθοδολογία (η θεωρία της διαψευσιμότητας) διαποτίζεται από μια

σειρά

παράδοξα , με πρώτο εκείνο του δημιουργικού ρόλου τον οποίο αποδίδει στο ψεύδος ως

καταλύτη στην διαδικασία προαγωγής της επιστημονικής γνώσης.

Ο θεμελιακός στόχος της επιστημονικής έρευνας δεν είναι με κανένα τρόπο η

επιβεβαίωση της αλήθειας όπως την κατανοούμε σήμερα, αλλά η ανακάλυψη ριζικά

νέας και πιο ενδιαφέρουσας αλήθειας, και μάλιστα τέτοιας που ακυρώνει αυτό που

ισχύει ως αλήθεια κατά το παρόν. Κάθε θεωρία της επιστήμης λοιπόν που ζητά να

υπηρετήσει αυτή την προοδευτική και ανατρεπτική δυναμική της γνώσης οφείλει να

κρατήσει

ανοιχτό αυτόν ακριβώς τον ορίζοντα της αέναης αλλαγής.

Ο επιστήμονας λοιπόν έχει το δικαίωμα να εφευρίσκει, να επινοεί, να οραματίζεται θεωρίες

που

Page 61: ΣΗΜΕΙΩΣΕΙΣ ΦΙΛΟΣΟΦΙΑΣ I

περιγράφουν τη δομή του κόσμου με νέους και ανεπανάληπτους τρόπους. Η θεωρητική

διάνοια

είναι απόλυτα ελεύθερη να φαντάζεται και να εικοτολογεί με τον πιο ρηξικέλευθο τρόπο,

σπάζοντας αμείλικτα τις δέλτους της συμβατικής σοφίας, υπό την προϋπόθεση φυσικά ότι

τα

θεωρητικά μοντέλα που παράγονται έτσι είναι δυνατόν να υποβληθούν σε πειραματικό

έλεγχο.

Η επιστημονική θεωρία είναι συνεπώς μια σειρά πεπαιδευμένων εικασιών. Είναι

πεπαιδευμένες γιατί δεν πρόκειται για αυθαίρετες διαισθήσεις και οραματισμούς, αλλά

πηγάζουν

από τη συστηματική τριβή με το εμπειρικό υλικό καθώς και με τις άλλες θεωρίες. Δεν

παύουν

όμως να είναι "μαντεψιές" σχετικά με το πώς θα μπορούσε να είναι η πραγματικότητα αν

τολμούσαμε να τη δούμε από μια άλλη γωνία και με διαφορετικές θεωρητικές αφετηρίες,

αν

τολμούσαμε να δούμε το μέλλον όχι ως συνέχεια του παρελθόντος αλλά ως εκ θεμελίων

αναδιάρθρωση του (για να χρησιμοποιήσουμε μια ιδέα του Χιουμ). Οι επιστημονικές αυτές

εικασίες υποβάλλονται στη συνέχεια στον απαραίτητο εμπειρικό έλεγχο με στόχο την

ενδεχόμενη διάψευση τους.

18. Η παραγωγή επιστημονικής γνώσης αρχίζει πάντοτε με τη διαπίστωση ενός

προβλήματος μέσα στη φυσική ή στην κοινωνική τάξη, μιας κατάστασης δηλαδή

πραγμάτων την οποία αδυνατεί προς το παρόν να εξηγήσει ο ανθρώπινος νους.

Από την ως άνω κατανόηση της επιστημονικής μεθόδου προκύπτει και το άλλο λογικό

παράδοξο

της ποππεριανής θεωρίας, η ιδέα δηλαδή ότι η πρόοδος των φυσικών μας γνώσεων

εξαρτάται από την επιλογή κάθε φορά των θεωριών με τη λιγότερη εκ πρώτης όψεως

εμπειρική πιθανότητα. Βασικό κριτήριο για την ορθολογικότητα μιας θεωρίας είναι για τον

Πόππερ το εύρος του εμπειρικού της περιεχομένου, ο αριθμός δηλαδή και το είδος των

εμπειρικών προβλέψεων που λογικά πηγάζουν από αυτήν. Όσο πιο πολλές και όσο πιο

καινοφανείς είναι οι προβλέψεις αυτές, τόσο περισσότερες είναι και οι δυνατότητες για τον

Page 62: ΣΗΜΕΙΩΣΕΙΣ ΦΙΛΟΣΟΦΙΑΣ I

πειραματικό της έλεγχο. Από αυτό όμως έπεται ότι η θεωρία με τον υψηλότερο βαθμό

διαψευσιμότητας είναι εκείνη και με τον χαμηλότερο βαθμό πιθανότητας, ακριβώς

επειδή ο αριθμός και το είδος των προβλέψεων της μας δίνουν πολύ μεγαλύτερο

περιθώριο για

να τη διαψεύσουμε αν είναι ψευδής. Αυτό εκφράζει με άλλο τρόπο την πρωταρχική ιδέα

ότι

επιστημονική κατεξοχήν είναι εκείνη η θεωρία που από την ίδια της τη δομή μας παρέχει τα

μέσα για να θηρεύσουμε το αντιπαράδειγμα που ενδεχομένως θα την καταρρίψει.

Η μεθοδολογία της πανεπιστημιακής διδασκαλίας ήταν ο σχολαστικισμός με βάση την

γραμματική. Οι λόγιοι έπρεπε να γνωρίζουν τη σχέση ανάμεσα στη λέξη και στον κόσμο,

απαίτηση που εξηγεί και την σοβαρότητα του προβλήματος νομιναλισμού - ρεαλισμού του

Μεσαίωνα. Μέθοδος απόδειξης ήταν η Διαλεκτική, ένα σύνολο διαδικασιών μέσω των

οποίων το

αντικείμενο της γνώσης μετατρέπεται σε πρόβλημα και υπόκειται σε διάλογο

αντιτιθέμενων

πλευρών.

Οι 219 καταδικασμένες προτάσεις προέκυψαν ως αποτέλεσμα της ανάδυσης τριών

διακριτών

ιδεολογικών ρευμάτων στο Παρίσι: του Αλβέρτου και του Ακινάτη που προσπάθησαν να

εναρμονίσουν Λόγο και πίστη, των ριζοσπαστικών Αριστοτελιστών και των

ΝεοΑυγουστιανών οι

οποίοι και τελικά επέβαλλαν τις καταδίκες. Αν η φιλοσοφία κατέληγε τελικά συστηματικά

σε

συμπεράσματα αντίθετα από την πίστη , δεν μπορούσε πλέον να θεωρείται ως έμπιστη

θεραπαινίδα, αντίθετα άρχιζε να μεταβάλλεται σε εχθρική δύναμη και απειλή, η οποία

απαιτούσε

αποφασιστική δράση.

Κοϋρέ : Οι Προτάσεις δεν έχουν πρακτική αξία στις επιστήμες

Duhem: Οι καταδίκες του 1277 είναι η πράξη γένεσης της επιστήμης των νέων χρόνων,

επειδή

καθιέρωσαν τη δυνατότητα μελέτης μη Αριστοτελικών θέσεων.

Page 63: ΣΗΜΕΙΩΣΕΙΣ ΦΙΛΟΣΟΦΙΑΣ I

Τα έξι σημεία στα οποία ο αριστοτελισμός έρχεται σε αντίθεση με τη χριστιανική θεολογία

είναι :

α) η αιωνιότητα του κόσμου β) ο μονοψυχισμός γ) η άρνηση της προσωπικής αθανασίας δ)

η

αιτιοκρατία ε)η άρνηση της θείας πρόνοιας στ) η άρνηση της ελεύθερης βούλησης.

Σχετικά με ΕΝΤΑΣΕΙΣ και ΔΙΑΜΑΧΕΣ ΤΟ 1277: Οι συγκρούσεις μεταξύ αριστοτελισμού και

θεολογίας οι οποίες σχετίζονται με την φύση του Θεού και την ανθρώπινη ικανότητα

γνώσης

του, τον περιορισμό της παντοδυναμίας του και της γνώσης του, την άποψη ότι ο κόσμος

είναι

αιώνιος, όπως και στα θέματα της αναγκαιότητας και του τυχαίου, στην αβερροική ενότητα

της

διάνοιας, στους περιορισμούς της ανθρώπινης ελευθερίας που πηγάζουν από τη θεώρηση

της

ανθρώπινης θέλησης ως ενεργούμενου άλλου καθώς και στο θέμα της προτεραιότητας του

δικαιώματος στην αλήθεια ανάμεσα σε θεολόγους και φιλόσοφους. Επίσης στις

ενδοθεολογικές

συγκρούσεις ανάμεσα σε Νεοαυγουστιανούς, θωμιστές και ριζοσπάστες αριστοτελικούς.

Όποιες και αν ήταν οι κινητήριες δυνάμεις που ωθούσαν την αναθεώρηση της

αριστοτελικής

θεωρίας της κίνησης, η μεσαιωνική αποτίμηση του Αριστοτέλη είχε ζωτικές συνέπειες για

την

πορεία της φυσικής φιλοσοφίας.

Κατά την γνώμη του Κοϋρέ η Επιστημονική επανάσταση του 16ου και 17ου αιώνα δεν ήταν

εκβλάστηση η επέκταση της μεσαιωνικής επιστήμης αλλά μια πνευματική μετάλλαξη που

οδήγησε στη διάλυση της μεσαιωνικής κοσμοθεώρησης.

ΕΠΟ 22 ΙΣΤΟΡΙΑ ΤΗΣ ΦΙΛΟΣΟΦΙΑΣ ΣΤΗΝ ΕΥΡΩΠΗ

ΣΗΜΕΙΩΣΕΙΣ ΦΙΛΟΣΟΦΙΑΣ III

ΑΡΙΣΤΟΤΕΛΙΣΜΟΣ ΚΑΙ ΜΕΣΑΙΩΝΙΚΟΣ ΠΕΙΡΑΜΑΤΙΣΜΟΣ

ΑΥΓΟΥΣΤΙΝΟΣ

Παραγάγει ένα νέο δυτικό χριστιανικό πλαίσιο εντός του οποίου συσχετίζονται οι

Page 64: ΣΗΜΕΙΩΣΕΙΣ ΦΙΛΟΣΟΦΙΑΣ I

φιλοσοφικές κατηγορίες του Λόγου και της εμπειρίας με τη θεια αποκάλυψη. Αυτό

σηματοδοτεί και τη διαφοροποίηση της Επιστήμης του Μεσαίωνα από την

αρχαιοελληνική παράδοση.Δεν είναι σκεπτικιστής και πιστεύει στην αξιοπιστία των

εμπειρικών δεδομένων, αλλά θεωρεί ότι υπάρχουν πράγματα "τα οποία δεν είναι

δυνατόν να εντοπίσουμε ούτε στο Λόγο ούτε στην εμπειρία.Υποβάθμισε την επιστήμη

και τη φιλοσοφία προς όφελος της ανερχόμενης τότε θεολογίας.Στηριζόμενος στη

χριστιανική έννοια της Σωτηρίας και γνωρίζοντας την ελληνική παράδοση που κατά

τον Πλάτωνα το σώμα είναι φυλακή της ψυχής και η αποδέσμευση από τον υλικό

κόσμο οδηγεί στις ανώτερες σφαίρες της νόησης θεωρεί ότι η ύλη - σάρκα αυτή

καθαυτή είναι αγαθό. Αναβαθμίζοντας την ύλη σε σχέση με τον Πλάτωνα θεώρησε

την αιτία της αμαρτίας στη βούληση άσχετα με την φθορά της ύλης λόγω του

προπατορικού αμαρτήματος.Ο Θεός εγγυάται την επιστημολογική εγκυρότητα της

γνώσης του εξωτερικού κόσμου μέσω της "θειας φώτισης"Η φιλοσοφία αν και όχι

τόσο αξιόπιστο εργαλείο άσκησε σημαντική επίδραση και έπρεπε να είναι

θεραπαινίδα της θεολογίας και όχι να εκριζωθεί αλλά να καλλιεργηθεί, να

πειθαρχήσει και να χρησιμοποιηθείΣτο πρόβλημα της σχέσης Πίστης και Λόγου

θεωρεί ότι επειδή υπάρχει μια Λέξη του Θεού μέσω της οποίας έγιναν όλα τα

πράγματα και είναι αμετάβλητη αλήθεια και είναι ζωή, το φως των ανθρώπων, το

φως των λογικών μυαλών μέσω των οποίων οι άνθρωποι διαφέρουν από τα

ζώαΥπάρχει ρόλος για το λογικό στη γνωστική διαδικασία στο πλαίσιο όμως της

κυριαρχίας της θεϊκής δράσης.Όπως το ηλιακό φως μας επιτρέπει να βλέπουμε

αντικείμενα, το θεϊκό φως μας επιτρέπει να βλέπουμε αιώνιες αλήθειες.Οι

σπερματικές αρχές αφορούν αξιωματικές μορφές σχηματισμού της ύλης συγκρίσιμες

με τους σπόρους, οι οποίες εμφυτεύονται από τον Θεό κατά την Δημιουργία

ευνοώντας τις αναπτυξιακές ερμηνείες της κοσμικής ιστορίας του σύμπαντος.Η

Μαθηματικοποίηση του είναι ισχυρότερη της Πλατωνικής αφού η Μαθηματική

αλήθεια έρχεται πλησιέστερα στα αμετάβλητα χαρακτηριστικά του Θεού.Καταδίκασε

Page 65: ΣΗΜΕΙΩΣΕΙΣ ΦΙΛΟΣΟΦΙΑΣ I

την αστρολογία ως μορφή ειδωλολατρίας.(τάση προς την μοιρολατρία και την

αιτιοκρατία)Συζητά για την φύση του χρόνου αρχίζοντας μια αντιπαράθεση με

επικριτές της χριστιανικής αντίληψης για την Γένεση. Θεωρεί ότι δεν υπάρχει νόημα

στην παρατήρηση γιατί συσχετίζει καταστάσεις προ και μετά την Γένεση που δεν

σχετίζονται.Ο χρόνος (απομακρυνόμενος από τη Αριστοτελική άποψη) είναι σχέση

ανάμεσα σε γεγονότα και δεδομένου ότι τα πάντα έρχονται και παρέρχονται

συνέπεια της θεϊκής δράσης όλα ενέχουν την παρελθούσα και μελλοντική ύπαρξη

καταλήγοντας στον αβέβαιο ορισμό ότι ο χρόνος αποτελεί έκταση χωρίς να

προσδιορίζει τίνος.Η γνώση βασίζεται στην θεια φώτιση.

ΑΝΣΕΛΜΟΣ

Ο πιστός διαλογιζόμενος πάνω στην πίστη του μπορεί να αποδείξει λογικά ορισμένες

πλευρές της.Στο έργο του δεν υπάρχει η διάκριση (του Ακινάτη) ανάμεσα στις

φυσικές δυνατότητες του Λόγου που είναι κοινές σε όλους τους ανθρώπους πιστούς

και μη, και στις υπερφυσικές του δυνάμεις, τις οποίες κατέχουν όσοι φωτίζονται από

τον Θεό.Εφόσον υπάρχουν όντα θα πρέπει να δεχτούμε και το υπέρτατο Είναι από το

οποίο αντλούν το είναι τους όλα τα αλλά όντα. Το υπέρτατο Όν μπορεί να εννοηθεί

ως κάτι υπάρχον ακριβώς λόγω της φύσης του. Η ουσία του Θεού - και μόνο του

Θεού, όχι των άλλων όντων - συνεπάγεται την ύπαρξη του.Διακρίνει διάφορα είδη

Αλήθειας, θετικών η αρνητικών κρίσεων, αλήθεια γνώμης, αλήθεια της θέλησης,

αληθινό αυτό που επιθυμούμε, αλήθεια των πραγμάτων, αληθινά πράγματα όταν

πρέπει να είναι αληθινά σε συνάφεια με τα αρχέτυπα τους που είναι ο Θεός.

Συνεπώς μια απόλυτη αλήθεια πρέπει να είναι ουσιαστικά μια Υπέρτατη αλήθεια.Ο

Θεός είναι αυτό του οποίου δεν μπορεί να θεωρηθεί κάτι μεγαλύτερο. Αλλά αυτό του

οποίου δεν μπορεί να νοηθεί μεγαλύτερο πρέπει να υπάρχει, όχι μόνο νοητικά αλλά

και στην πραγματικότητα. Συνεπώς ο Θεός υπάρχει όχι μόνο σαν ιδέα αλλά και στην

πραγματικότητα.Λογικό σχήμα modus tollens.Γενικότερη ενίσχυση του

ορθολογισμού.

Page 66: ΣΗΜΕΙΩΣΕΙΣ ΦΙΛΟΣΟΦΙΑΣ I

ΓΚΡΟΣΣΕΤΕΣΤ

Εξομάλυνση σημείων τριβής και καθορισμός ορίων ώστε η αριστοτελική

φιλοσοφία να χρησιμοποιηθεί από ένα χριστιανικό πολιτισμό.Έντονη επιρροή

από πλατωνικά και νεοπλατωνικά στοιχεία, προβληματική αντιπαράθεση

αριστοτελικών και μη στοιχείων.Ο σκοπός της επιστημονικής αναζήτησης για τον

Αριστοτέλη ήταν να βρεθούν οι όροι ώστε κάτι ήδη γνωστό να παραχθεί η να

αποδειχθεί βάσει πρώτων αρχών. Ο μελετητής δεν επιδιώκει μόνο γνώση του

γεγονότος , "απόδειξη του ότι" αλλά βαθύτερη "απόδειξη του διότι".Οι κατώτερες

επιστήμες παρέχουν τα γεγονότα ενώ τα μαθηματικά δίνουν τους λόγους για αυτά

χωρίς όμως να προσπαθήσει να υποτάξει ολόκληρο τον φυσικό κόσμο στον

μαθηματικό.Η Γεωμετρία μπορούσε να δώσει την αιτία για το γεγονός με την έννοια

της τυπικής αιτίας και όχι της Υλίκης και δρώσας αιτίας.ήταν από τους πρώτους που

διείδαν και την αντίφαση στην αστρονομική περιγραφή βάσει των Πτολεμαίων

έκκεντρων και επίκυκλων που έσωζαν τα φαινόμενα και οι οποίοι δεν μπορούσαν να

δώσουν υλικές και δρώσες αιτίες για τις ουράνιες κινήσεις.Η μελέτη της εμπειρίας

βάσει της Αριστοτέλειας μεθόδου ανάλυσης και σύνθεσης (ως πρακτικό τρόπο για να

περιγράψει τις δυο κινήσεις της επιστήμης από την εμπειρία στη θεωρία και

αντίστροφα)προϋποθέτει την εύρεση του ονομαστικού μη αιτιακού ορισμού, ο οποίος

θα περιγράψει τους συσχετισμούς ανάμεσα στα γεγονότα προτάσεις που μας

ενδιαφέρουν.Θεμελιώδεις έννοιες ο μακρόκοσμος και ο μικρόκοσμος. Η

ανθρωπότητα αντιπροσωπεύει τον Κολοφώνα της δημιουργικής δραστηριότητας του

Θεού αντανακλώντας ταυτόχρονα τη θεια φύση και τοις δομικές αρχές του

δημιουργημένου κόσμου.Κάτι που θα μπορούσε να εννοηθεί ως η πρώτη εμφάνιση

μιας σαφούς κατανόησης των αρχών της νεώτερης πειραματικής επιστήμης

συναντούμε στα γραπτά του.

ΑΚΙΝΑΤΗΣ

Προσπάθεια συγκερασμού της χριστιανικής Πίστης και της Αριστοτελικής Φιλοσοφίας

Page 67: ΣΗΜΕΙΩΣΕΙΣ ΦΙΛΟΣΟΦΙΑΣ I

θεωρεί ότι δεν υπάρχει αντίθεση στη γνώση της πίστης και της λόγιας φιλοσοφίας,

αφού προέρχονται από τον Θεό αναβαθμίζοντας την αξία της επιστήμης.Η

αριστοτελική φιλοσοφία και χριστιανική θεολογία αν και μεθοδολογικά διακριτές

αποτελούν συμβατές οδούς προς την αλήθεια Η φιλοσοφία χρησιμοποιεί τις φυσικές

ανθρώπινες δυνάμεις της αίσθησης και του λογικού για να συλλάβει όσες αλήθειες

μπορούμε να γνωρίσουμε με αυτόν τον τρόπο. Η θεολογία προσφέρει πρόσβαση σε

αλήθειες εξ αποκαλύψεως οι οποίες υπερβαίνουν τη δυνατότητα των φυσικών

ικανοτήτων ανακάλυψης και κατανόησης.Πιστεύει στη διάκριση ανάμεσα στις

φυσικές δυνατότητες του Λόγου που είναι κοινές σε όλους τους ανθρώπους πιστούς

και μη, και στις υπερφυσικές του δυνάμεις, τις οποίες κατέχουν όσοι φωτίζονται από

τον Θεό.Η νόηση σχηματίζει ότι κατανοεί από αισθητηριακά δεδομένα και δημιουργεί

πρώτες αρχές οι οποίες γίνονται στη συνέχεια εργαλεία που δημιουργούν ολόκληρο

το οικοδόμημα .Θεωρεί το θέμα της αιωνιότητας του κόσμου ότι είναι ένα ανοιχτό

ορθολογιστικό ζήτημα αλλά το κύριο ερώτημα αφορά την αιτιακή εξάρτηση του

κόσμου από τον Δημιουργό.Σε αντίθεση με την ευρύτατα αποδεκτή Αυγουστιανή

θέση σύμφωνα με την οποία η γνώση βασίζεται στη θεια φώτιση θεωρούσε ότι ο

άνθρωπος μέσω της παρατήρησης και του λόγου μπορούσε να φτάσει σε βέβαιη και

έγκυρη γνώση. Είναι αδύνατο να βρεθούν η Πίστη και η φυσική πραγματικότητα σε

αντίθεση.Σαφής διάκριση της περιοχής του Λόγου από την περιοχή της Αποκάλυψης,

σύμφωνα με την οποία το πεδίο του πρώτου είναι σαφέστατα

περιορισμένο.(Γνωρίζουμε τον Θεό από τις πράξεις Του και όχι Αυτόν τον Ίδιο). Οι

περιοχές της αλήθειας στις οποίες έχουν πρόσβαση ο ορθολογισμός και η

παρατήρηση του εξωτερικού κόσμου είναι ρητά διαφορετικές από αυτές στις οποίες

αποκτά πρόσβαση ο πιστός με την βοήθεια του Θεού.Δεδομένου ότι όλη η γνώση

πηγάζει από τον Θεό και υπάρχει τάξη και αρμονία στον κόσμο, η Χάρη δεν

καταστρέφει την Φύση αλλά την τελειοποιεί.Θεμέλιος λίθος του Θωμισμού είναι η

διάκριση ουσίας και ύπαρξης με εξαίρεση το Θεό το μόνο αυθύπαρκτο όν. Στον Θεό

Page 68: ΣΗΜΕΙΩΣΕΙΣ ΦΙΛΟΣΟΦΙΑΣ I

οι ποιότητες της αλήθειας, της αγαθότητας του ωραίου υπάρχουν όχι απλώς δυνάμει

όπως στα αλλά όντα, αλλά στην πραγματικότητα λόγω ακριβώς αυτού του

γεγονότος. Κάθε πεπερασμένο όν γίνεται από την δράση και την δυνατότητα. Η

ύπαρξη δίνει τη δυνατότητα στην ουσία να δράσει, αλλά και η ίδια η ύπαρξη

περιορίζεται από αυτή τη δυνατότητα. Η αγαθότητα του Θεού και των ανθρώπων

,μπορεί να βεβαιωθεί όχι με ταυτοσημία ούτε με ασάφεια αλλά με την αρχή της

αναλογίας και να γίνει γνώσιμη στο επίπεδο αυτό.Το άγιο δόγμα είναι επιστήμη ,

επειδή στηρίζεται σε αξιώματα τα οποία γίνονται γνωστά μέσω μιας ανώτερης

επιστήμης(του θεού και του ευλογημένου). Συνεπώς όπως η μουσική δέχεται την

αυθεντία των αξιωμάτων της αριθμητικής έτσι και η Αγια επιστήμη δέχεται τα

αξιώματα που αποκαλύπτονται από τον Θεό.Σύνθεση λόγου και πίστης,

εμπέδωση αριστοτελικής φιλοσοφίας και διαμόρφωση λιγότερο καχύποπτης

στάσης απέναντι στους ισχυρισμούς της.Συνεχιστής του Αλβέρτου πίστευε ότι

αυτό το οποίο μας διδάσκει η πίστη δεν μπορεί να έρχεται σε αντίθεση με αυτό που

μας δίνει η φύση.

ΑΒΕΛΑΡΔΟΣ

Νομιναλιστική θέση σχετικά με το πρόβλημα των καθολικών εννοιών, θέση η οποία

διαφέρει όμως από την αντίστοιχη αντίληψη του Ύστερου Μεσαίωνα η οποία

αναπτύσσεται κάτω από το πλέγμα της μετέπειτα διάκρισης ρεαλισμού -

νομιναλισμού.Δεν υπάρχουν καθολικές οντότητες ,όλα τα υπάρχοντα είναι μοναδικά

και μόνο αυτά μελετούν οι ανθρώπινες αισθήσεις. Μόνο οι συγκεχυμένες εντυπώσεις

της φύσης οδηγούν σε υπαγωγή των αντικειμένων σε ολόκληρη κλάση και τελικά

όλες οι έννοιες είναι παράγωγες.Η γνώση αρχίζει από την μελέτη των ατομικών

όντων και οδηγείται στις έννοιες οι οποίες χρησιμοποιούνται σε λογικές προτάσεις. Η

λογική σχετίζεται μόνο με τη λογική και διασαφηνίζει έννοιες και προτάσεις τις

οποίες χρησιμοποιούμε χωρίς αναφορά στις ιδιότητες των αντικειμένων.(καθιέρωση

καθαρά τυπικής λογικής).Ο Θεός δεν μπορεί να είναι καλύτερος από τις πράξεις του

Page 69: ΣΗΜΕΙΩΣΕΙΣ ΦΙΛΟΣΟΦΙΑΣ I

γιατί τα ρήματα που δηλώνουν την καλοσύνη του Θεού είναι ταυτόσημα με αυτά που

δηλώνουν την καλοσύνη των πράξεων του.

ΜΠΕΙΚΟΝ

Εξομάλυνση σημείων τριβής και καθορισμός ορίων ώστε η αριστοτελική

φιλοσοφία να χρησιμοποιηθεί από ένα χριστιανικό πολιτισμό.Εκστρατεία υπέρ

της χρησιμότητας των νέων Αριστοτελικών γνώσεων συμβάλλει στην ερμηνεία της

γραφής και στην απόδειξη άρθρων της πίστεως.τα τέσσερα εμπόδια για την

κατανόηση της αλήθειας είναι η υποταγή σε λανθασμένη αυθεντία, η δύναμη της

συνηθείας ,η λαϊκή πρόληψη και τέλος η επικάλυψη της αγνοίας με επίδειξη

σοφίας.Τα μαθηματικά "παρέχουν αλήθειες χωρίς σφάλμα" και η εμπειρία

"βεβαιότητα χωρίς αμφιβολία".Θεωρεί τον πειραματισμό ανώτατο κριτήριο αλήθειας,

γιατί όλες οι άλλες επιστήμες πέραν του πειραματισμού "είτε χρησιμοποιούσαν

επιχειρήματα για να αποδείξουν τα συμπεράσματα τους είτε έχουν καθολικά ατελείς

εμπειρίες".Εγκωμιάζοντας τα μαθηματικά θεωρεί ότι αυτά παρέχουν την γνώση του

διότι έναντι της φυσικής που παρέχει την γνώσει του ότι. Δεν υποτάσσει τις άλλες

επιστήμες στα μαθηματικά και θα προχωρήσει αξιοποιώντας τα για την ίδια τη φύση

των πραγμάτων σασί όχι απλώς την πλατωνική αναπαράσταση τους ακόμη και για τα

θεια πράγματα.Ακόμα και στη θεολογία τα θεωρεί χρήσιμα θεωρώντας πως η γνώση

τους συνιστά στη καλύτερη κατανόηση της Βίβλου.Οι καινοτομίες του θα πρέπει να

θεωρηθούν επαναφορά της αυγουστίνειας παράδοσης αλλά η αισιοδοξία του για τον

ρόλο των επιστημών σε αντίθεση με τον πεσιμισμό του Αυγουστίνου είχε ως

αποτέλεσμα αυτή η παραδοσιακή πρακτική να συμβάλλει ώστε να επέλθουν

θεμελιακές αλλαγές του επιστημονικού λόγου.

ΟΡΕΜ

Μόνο ο Θεός μπορεί να μας δώσει εγγυήσεις αληθούς γνώσης. Έτσι προκύπτει η

έμφαση στην απεριόριστη ελευθερία βούλησης του Θεού η οποία πηγάζει από την

απόλυτη δύναμη του.Επανεξέτασε τις Αριστοτελικές έννοιες απόλυτα βαρύ και

Page 70: ΣΗΜΕΙΩΣΕΙΣ ΦΙΛΟΣΟΦΙΑΣ I

ελαφρύ των σωμάτων μετά το 1277.

ΜΠΟΥΡΙΝΤΑΝ

Θεωρεί πως ο Θεός στη φύση δρα πάντοτε με την Αριστοτέλεια αντίληψη.Είναι μη

αποδείξιμη η πρόταση ότι ένα σώμα του άλλου κόσμου (κατά τον Αριστοτέλη) θα

κινούνταν προς τη Γη λέγοντας ότι τελικά οι κινήσεις έξω από τη Γη επηρεάζονται

από τα άλλα ουράνια σώματα και τον Θεό.

ΣΙΖΕΡ (ΑΚΡΑΙΟΣ ΑΡΙΣΤΟΤΕΛΙΣΤΗΣ)

Μαζί με τον Βοήθιο της Δακίας ο κυριότερος αντιπρόσωπος του λατινικού

Αβερροiσμού η "κοσμικού Αριστοτελισμού".Οπαδός της Διπλής Αλήθειας και

Αβερροiστής ετερόδοξος η για άλλους πιστός χριστιανός και οπαδός του

ΑριστοτελισμούΣτα κείμενα του επιχειρηματολογεί υπέρ της αιωνιότητας

υποτάσσοντας την θεολογία στον Αριστοτελισμό αφήνοντας στον αναγνώστη την

τελική κρίση.Εξασκεί την φιλοσοφία χωρίς να λαμβάνει καθόλου υπ'όψιν τη

θεολογική διδασκαλία σχετικά με τα ζητήματα που έθιγε.Τα συμπεράσματα στα οποία

κατέληγε ήταν τα αναγκαία και αναπόφευκτα συμπεράσματα της φιλοσοφίας όταν

αυτή εξασκηθεί με τον σωστό τρόπο.

ΑΛΒΕΡΤΟΣ Ο ΜΕΓΑΣ

Ο πρώτος που επεξεργάστηκε μια περιεκτική ερμηνεία της φιλοσοφίας του

Αριστοτέλη στη Χριστιανική Δύση. Πάντα έτοιμος να διορθώσει η να απορρίψει

αριστοτελικές θέσεις προ όφελος συχνά πλατωνικών η νεοπλατωνικών

θέσεων.Σύνθεση λόγου και πίστης, εμπέδωση αριστοτελικής φιλοσοφίας και

διαμόρφωση λιγότερο καχύποπτης στάσης απέναντι στους ισχυρισμούς

της.Θεωρούσε την αριστοτελική φιλοσοφία αναγκαία προπαρασκευή για τις

θεολογικές σπουδές.Υποστηρικτής του χριστιανικού δόγματος της

δημιουργίας.Θεωρούσε την ιδέα του αιώνιου σύμπαντος φιλοσοφικά παράλογη και

άρα η φιλοσοφία μπορούσε να αντιμετωπίσει το ζήτημα χωρίς θεολογική

βοήθεια.Σχετικά με το πρόβλημα της φύσης της ψυχής και των δυνάμεων της: Το

Page 71: ΣΗΜΕΙΩΣΕΙΣ ΦΙΛΟΣΟΦΙΑΣ I

ζητούμενο ήταν μια περιγραφή της ως χωριστής αιώνιας ουσίας ανεξάρτητης από το

σώμα και ικανής να επιβιώσει μετά τον θάνατο του, η οποία όμως θα μπορούσε να

εξηγήσει την ένωση της ψυχής με το σώμα, φορέα της αίσθησης και της

ζωτικότητας.Προσπάθησε να διακρίνει ορθολογικά την φιλοσοφία από τη θεολογία

σε μεθοδολογική βάση και να ανακαλύψει όλα όσα η φιλοσοφία μόνη θα μπορούσε

να αποδείξει για την πραγματικότητα.θεωρούσε τον Θεό έσχατο αίτιο των πάντων

και ότι ενεργεί συνήθως μέσω της φυσικής αιτιότητας. Το καθήκον του φιλόσοφου

δεν είναι η εξέταση των αιτίων της θειας βούλησης αλλά η διερεύνηση των φυσικών

αιτίων μέσω των οποίων αυτή παράγει τα αποτελέσματα της.

ΜΠΟΝΑΒΕΝΤΟΥΡΑ

Στηρίζεται στις βασικές ιδέες του Αυγουστίνου.Πλούσια σύνθεση αριστοτελικών και

μη στοιχείων, αίσθηση κινδύνου από τη διάδοση της νέας φιλοσοφίας αυστηρός

έλεγχος της, απαισιοδοξία για την ικανότητα του ανθρώπινου λόγου να γνωρίζει την

αλήθεια χωρίς θεια φώτιση.Κομβικά σημεία η Δημιουργία εκ του μηδενός. Η σχέση

μίμησης όλων των όντων προς τον Θεό και η Θεια φώτιση.Η νόηση βρίσκει (αντίθετα

από τον Ακινάτη ) στον ίδιο τον εαυτό της ότι κατανοεί, και αυτό δεν έχει

σχηματιστεί από τα αισθητηριακά δεδομένα αλλά έχει ληφθεί από τον Ένα..θεωρείται

ο φιλόσοφος της χριστιανικής ζωής ο οποίος επιχειρεί μια σύνθεση λόγου και πίστης

με την μορφή της υποταγής του Λόγου.Εξομάλυνση σημείων τριβής και καθορισμός

ορίων ώστε η αριστοτελική φιλοσοφία να χρησιμοποιηθεί από ένα χριστιανικό

πολιτισμό.Η αιωνιότητα του κόσμου ήταν φιλοσοφικά παράλογη.

ΒΟΗΘΙΟΣ της ΔΑΚΙΑΣ(ΑΚΡΑΙΟΣ ΑΡΙΣΤΟΤΕΛΙΣΤΗΣ)

Μέλος του κύκλου του Σίζερ υποστήριζε τον αυστηρό διαχωρισμό του φιλοσοφικού

από το θεολογικό επιχείρημα. Συνέλεξε συστηματικά και κατόπιν ανασκεύασε όλα τα

φιλοσοφικά επιχειρήματα που είχαν χρησιμοποιηθεί για την υπεράσπιση του

χριστιανικού δόγματος της δημιουργίας.Ο φιλόσοφος μιλώντας ως φιλόσοφος δεν

έχει εναλλακτική λύση και πρέπει να υπερασπίσει την αιωνιότητα του

Page 72: ΣΗΜΕΙΩΣΕΙΣ ΦΙΛΟΣΟΦΙΑΣ I

κόσμου.Επέδειξε έναν έντονα "ορθολογιστικό προσανατολισμό.Αρνούμενος την

"Ανάσταση" των νεκρών και την δυνατότητα δημιουργίας του σύμπαντος.

ΑΚΙΝΑΤΗΣ

1. ΔΕΝ ΜΠΟΡΕΙ ΝΑ ΑΠΟΔΕΙΧΤΕΙ Η ΠΡΟΤΑΣΗ ΠΕΡΙ ΑΡΧΗΣ ΤΟΥ ΚΟΣΜΟΥ.

2. Η ΘΕΪΚΗ ΠΑΝΤΟΔΥΝΑΜΙΑ ΜΠΟΡΕΙ ΝΑ ΠΑΡΑΓΑΓΕΙ ΓΕΓΟΝΟΤΑ ΤΑΥΤΟΧΡΟΝΑ,

ΣΥΝΕΠΩΣ ΝΑ ΔΗΜΙΟΥΡΓΗΣΕΙ ΕΝΑΝ ΚΟΣΜΟ ΧΩΡΙΣ ΧΡΟΝΙΚΗ ΑΡΧΗ, ΕΝΑΝ

ΑΙΩΝΙΑ ΥΠΑΡΧΟΝΤΑ ΚΟΣΜΟ.

3. ΤΟ ΚΙΝΟΥΝ ΣΤΗΝ ΠΕΡΙΠΤΩΣΗ ΤΗΣ ΦΥΣΙΚΗΣ ΚΙΝΗΣΗΣ ΕΙΝΑΙ Ο ΥΠΕΥΘΥΝΟΣ

ΤΗΣ ΑΡΧΙΚΗΣ ΓΕΝΕΣΗΣ Η ΤΟΥ ΣΧΗΜΑΤΙΣΜΟΥ ΤΟΥ ΣΩΜΑΤΟΣ ΜΑΚΡΙΑ ΑΠΟ

ΤΟΝ ΦΥΣΙΚΟ ΤΟΥ ΤΟΠΟ. ΑΠΟ ΚΕΙ ΚΑΙ ΠΕΡΑ ΤΟ ΣΩΜΑ ΔΕΝ ΑΠΑΙΤΕΙ

ΚΑΠΟΙΟ ΚΙΝΟΥΝ , ΑΛΛΑ ΑΠΛΩΣ ΑΚΟΛΟΥΘΕΙ ΤΗ ΦΥΣΙΚΗ ΡΟΗ ΤΩΝ

ΠΡΑΓΜΑΤΩΝ.

ΤΙΜΑΙΟΣ

1. Η ΥΛΗ ΔΕΝ ΕΧΕΙ ΑΡΧΗ ΑΛΛΑ ΥΠΟ ΠΡΟΫΠΟΘΕΣΕΙΣ ΜΟΝΟ ΤΟ ΣΥΜΠΑΝ ΕΤΣΙ

ΟΠΩΣ ΕΙΝΑΙ ΔΟΜΗΜΕΝΟ.

2. ΕΑΝ ΑΝΑΜΕΣΑ ΣΕ ΔΥΟ ΦΥΣΙΚΑ ΕΙΔΗ ΥΠΑΡΧΕΙ ΘΕΩΡΗΤΙΚΑ ΔΥΝΑΤΟΤΗΤΑ

ΕΜΦΑΝΙΣΗΣ ΕΝΔΙΑΜΕΣΟΥ ΤΥΠΟΥ, ΤΟΤΕ ΑΥΤΟΣ ΘΑ ΠΡΕΠΕΙ ΝΑ

ΠΡΑΓΜΑΤΟΠΟΙΗΘΕΙ ΜΕ ΤΗΝ ΕΠ' ΑΠΕΙΡΟΝ ΕΜΦΑΝΙΣΗ ΕΙΔΩΝ, ΓΙΑΤΙ ΑΛΛΙΩΣ

Η ΥΠΑΡΞΗ ΚΕΝΩΝ ΣΤΗΝ ΦΥΣΗ ΘΑ ΗΤΑΝ ΕΝΔΕΙΞΗ ΜΗ ΑΓΑΘΟΤΗΤΑΣ ΤΟΥ

ΔΗΜΙΟΥΡΓΟΥ.

3. Η ΟΙΚΟΔΟΜΗΣΗ ΤΟΥ ΚΟΣΜΟΥ ΑΠΑΙΤΗΣΕ ΤΗΝ ΟΛΟΤΗΤΑ ΚΑΙ ΤΩΝ

ΤΕΣΣΑΡΩΝ ΣΤΟΙΧΕΙΩΝ. ΤΗΝ ΜΟΡΦΗ ΤΟΥ ΘΕΪΚΟΥ ΓΕΝΟΥΣ ΤΗΝ ΕΦΤΙΑΞΕ

ΚΑΤΑ ΚΥΡΙΟ ΛΟΓΟ ΑΠΟ ΦΩΤΙΑ , ΩΣΤΕ ΝΑ ΕΙΝΑΙ ΟΤΙ ΠΙΟ ΛΑΜΠΡΟ ΚΑΙ ΠΙΟ

ΩΡΑΙΟ ΜΠΟΡΕΙ ΝΑ ΔΕΙ ΚΑΝΕΙΣ. (Β. ΚΑΛΦΑΣ 1993. "Η ΣΩΜΑΤΙΚΟΤΗΤΑ ΤΩΝ

ΑΣΤΡΩΝ ΠΡΕΠΕΙ ΝΑ ΑΠΟΔΟΘΕΙ ΣΤΗΝ ΠΑΡΟΥΣΙΑ ΤΟΥ ΓΗΙΝΟΥ ΣΤΟΙΧΕΙΟΥ, Η

ΦΩΤΙΑ ΚΥΡΙΑΡΧΕΙ ΓΕΝΙΚΑ ΣΤΗ ΣΥΣΤΑΣΗ ΤΟΥΣ, ΕΝΩ ΤΑ ΑΛΛΑ ΔΥΟ

ΣΤΟΙΧΕΙΑ ΑΝΑΓΚΑΣΤΙΚΑ ΥΠΑΡΧΟΥΝ, ΛΟΓΩ ΤΗΣ ΠΛΑΤΩΝΙΚΗΣ ΑΝΤΙΛΗΨΗΣ

Page 73: ΣΗΜΕΙΩΣΕΙΣ ΦΙΛΟΣΟΦΙΑΣ I

Η ΟΠΟΙΑ ΑΠΑΙΤΕΙ ΔΥΟ ΟΝΤΟΛΟΓΙΚΕΣ ΜΕΣΟΤΗΤΕΣ , ΜΕ ΤΗ ΜΟΡΦΗ

ΓΕΩΜΕΤΡΙΚΗΣ ΑΝΑΛΟΓΙΑΣ.

ΟΚΚΑΜ

1. ΟΙ ΟΥΡΑΝΟΙ ΕΙΝΑΙ ΑΦΘΑΡΤΟΙ ΑΠΟ ΟΠΟΙΟΝΔΗΠΟΤΕ ΠΑΡΑΓΟΝΤΑ ΑΛΛΑ Ο

ΘΕΟΣ, Ο ΟΠΟΙΟΣ ΥΠΕΡΦΥΣΙΚΑ ΔΗΜΙΟΥΡΓΗΣΕ ΤΟ ΚΟΣΜΟ, ΜΠΟΡΕΙ ΕΠΙΣΗΣ

ΥΠΕΡΦΥΣΙΚΑ ΝΑ ΤΟΝ ΚΑΤΑΣΤΡΕΨΕΙ.2. ΚΙΝΗΣΗ ΕΙΝΑΙ ΑΦΗΡΗΜΕΝΟΣ ΚΑΙ

ΠΛΑΣΜΑΤΙΚΟΣ ΟΡΟΣ ,ΕΝΑ ΟΥΣΙΑΣΤΙΚΟ ΤΟ ΟΠΟΙΟ ΔΕΝ ΑΝΤΙΣΤΟΙΧΕΙ ΣΕ

ΚΑΠΟΙΑ ΥΠΑΡΧΟΥΣΑ ΟΝΤΟΤΗΤΑ. (ΔΕΝ ΕΙΝΑΙ ΠΡΑΓΜΑ)

TH. BRADWARDINE

1. ΟΥΣΙΑΣΤΙΚΑ ΚΑΙ ΜΕ ΠΑΡΟΥΣΙΑ, Ο ΘΕΟΣ ΕΙΝΑΙ ΑΝΑΓΚΑΣΤΙΚΑ ΠΑΝΤΟΥ

ΣΤΟΝ ΚΟΣΜΟ.

2. ΕΠΙΣΗΣ ΠΕΡΑ ΑΠΟ ΤΟΝ ΠΡΑΓΜΑΤΙΚΟ ΚΟΣΜΟ Η ΣΕ ΕΝΑ ΦΑΝΤΑΣΤΙΚΟ

ΑΠΕΙΡΟ ΚΕΝΟ

3. ΠΡΑΓΜΑΤΙΚΑ ΑΥΤΟΣ ΜΠΟΡΕΙ ΝΑ ΚΛΗΘΕΙ ΑΧΑΝΗΣ ΚΑΙ ΑΠΕΡΙΟΡΙΣΤΟΣ

4. ΔΙΔΕΤΑΙ Η ΑΠΑΝΤΗΣΗ ΣΤΟΥΣ ΑΠΙΣΤΟΥΣ "ΠΟΥ ΕΙΝΑΙ Ο ΘΕΟΣ ΣΑΣ" ΚΑΙ ΠΟΥ

ΗΤΑΝ Ο ΘΕΟΣ ΠΡΙΝ ΤΗΝ ΔΗΜΙΟΥΡΓΙΑ.

5. ΥΠΕΡΑΣΠΙΣΤΗΚΕ ΤΗΝ ΑΠΟΨΗ ΟΤΙ ΤΟ ΣΥΜΠΑΝ ΠΕΡΙΒΑΛΛΕΤΑΙ ΑΠΟ ΑΠΕΙΡΟ

ΚΕΝΟ ΧΩΡΟ ΔΗΜΙΟΥΡΓΗΜΕΝΟ ΑΠΟ ΤΟΝ ΘΕΟ.

6. ΣΥΝΕΠΩΣ ΦΑΙΝΕΤΑΙ ΟΤΙ ΕΙΝΑΙ ΔΥΝΑΤΟΝ ΝΑ ΥΠΑΡΞΕΙ ΚΕΝΟ ΧΩΡΙΣ ΣΩΜΑ

ΑΛΛΑ ΜΕ ΚΑΝΕΝΑ ΤΡΟΠΟ ΧΩΡΙΣ ΤΟΝ ΘΕΟ.

ΓΚΡΟΣΣΕΤΕΣΤ

1. ΚΕΝΤΡΙΚΗ ΘΕΣΗ ΚΑΤΕΧΕΙ ΤΟ ΦΩΣ : Ο ΚΟΣΜΟΣ ΑΡΧΙΣΕ ΝΑ ΥΠΑΡΧΕΙ ΑΠΟ

ΤΗΝ ΣΤΙΓΜΗ ΠΟΥ Ο ΘΕΟΣ ΔΗΜΙΟΥΡΓΗΣΕ ΕΝΑ ΑΔΙΑΣΤΑΤΟ ΣΗΜΕΙΟ ΥΛΗΣ

ΚΑΙ ΤΗ ΜΟΡΦΗ ΤΟΥ ΕΝΑ ΣΗΜΕΙΟ ΦΩΤΟΣ ΧΩΡΙΣ ΔΙΑΣΤΑΣΕΙΣ.

2. Ο ΚΟΣΜΟΣ ΕΙΝΑΙ ΟΜΟΓΕΝΗΣ ΚΑΙ ΟΙ ΟΥΡΑΝΟΙ ΑΠΟΤΕΛΟΥΝΤΑΙ ΑΠΟ ΕΝΑ

ΥΛΙΚΟ ΛΕΠΤΟΤΕΡΟ ΚΑΙ ΑΡΑΙΟΤΕΡΟ ΑΠΟ ΑΥΤΟ ΤΩΝ ΕΠΙΓΕΙΩΝ ΟΥΣΙΩΝ,

ΑΛΛΑ Η ΔΙΑΦΟΡΑ ΕΙΝΑΙ ΠΟΣΟΤΙΚΗ ΚΑΙ ΟΧΙ ΠΟΙΟΤΙΚΗ.

Page 74: ΣΗΜΕΙΩΣΕΙΣ ΦΙΛΟΣΟΦΙΑΣ I

ΝΙΚΟΛΑΟΣ ΤΗΣ ΩΤΡΕΚΟΥΡΤ

1. Η ΚΕΝΤΡΙΚΗ ΤΟΥ ΘΕΣΗ ΑΦΟΡΑ ΤΗ ΜΗ ΥΠΑΡΞΗ ΟΡΙΟΥ ΣΤΟΝ ΦΙΛΟΣΟΦΙΚΟ

ΣΚΕΠΤΙΚΙΣΜΟ ΓΙΑ ΤΗΝ ΥΠΑΡΞΗ ΑΝΤΙΚΕΙΜΕΝΩΝ ΤΑ ΟΠΙΑ ΥΠΑΡΧΟΥΝ ΕΞΩ

ΑΠΟ ΤΟΝ ΝΟΥ ΜΑΣ.ΟΙ ΑΠΟΨΕΙΣ ΤΟΥ ΔΕΝ ΗΤΑΝ ΚΥΡΙΑΡΧΕΣ ΟΥΤΕ

ΜΠΟΡΕΣΑΝ ΝΑ ΕΚΦΡΑΣΟΥΝ ΣΕ ΜΙΑ ΕΠΟΧΗ ΕΠΙΣΤΗΜΟΝΙΚΗΣ ΠΡΟΟΔΟΥ

ΤΗΝ ΚΑΤΑΡΡΕΥΣΗ ΤΟΥ ΛΟΓΟΥ. Η ΣΗΜΑΣΙΑ ΑΥΤΟΥ ΤΟΥ ΜΕΣΑΙΩΝΙΚΟΥ

ΧΙΟΥΜ ΕΓΚΕΙΤΑΙ ΣΤΟ ΓΕΓΟΝΟΣ ΟΤΙ ΚΑΤΕΔΕΙΞΕ ΜΕΣΩ ΤΗΣ ΕΣΩΤΕΡΙΚΗΣ

ΚΡΙΤΙΚΗΣ ΜΙΑ ΣΥΝΕΠΗ ΚΑΙ ΔΥΝΑΤΗ ΑΠΟΛΗΞΗ ΤΗΣ ΣΚΕΨΗΣ ΤΗΣ ΕΠΟΧΗΣ

ΤΟΥ.

ΛΟΓΙΟΙ 13ΟΥ ΚΑΙ 14ΟΥ

1. Ο ΚΟΣΜΟΣ ΠΟΥ ΥΠΑΡΧΕΙ ΣΗΜΕΡΑ ΔΗΜΙΟΥΡΓΗΘΗΚΕ ΑΛΛΑ Η ΥΛΗ ΕΙΝΑΙ

ΑΙΩΝΙΑ.

2. Ο ΧΡΟΝΟΣ ΑΡΧΙΖΕΙ ΟΤΑΝ Ο ΘΕΟΣ ΔΗΜΙΟΥΡΓΗΣΕ ΤΗΝ ΠΡΩΤΗ ΚΙΝΗΣΗ,

ΣΥΝΕΠΩΣ Η ΑΙΩΝΙΟΤΗΤΑ ΤΑΥΤΙΖΕΤΑΙ ΜΕ ΤΗΝ ΠΛΗΡΗ ΔΙΑΡΚΕΙΑ ΤΟΥ

ΧΡΟΝΟΥ.

3. ΔΕΝ ΜΠΟΡΕΙ ΝΑ ΑΠΟΔΕΙΧΤΕΙ Η ΠΡΟΤΑΣΗ ΠΕΡΙ ΑΡΧΗΣ ΤΟΥ ΚΟΣΜΟΥ.

4. ΔΕΝ ΥΠΑΡΧΕΙ ΑΠΟΛΥΤΑ ΤΕΛΕΙΟΣ ΚΟΣΜΟΣ ΑΛΛΑ ΜΟΝΟΝ ΣΧΕΤΙΚΑ.

5. Ο ΘΕΟΣ ΚΑΤΑ ΤΟΥΣ ΣΧΟΛΑΣΤΙΚΟΥΣ ΕΦΤΙΑΞΕ ΤΟΝ ΚΟΣΜΟ ΟΣΟ ΚΑΛΟ

ΘΕΩΡΗΣΕ ΟΤΙ ΧΡΕΙΑΖΕΤΑΙ.

6. ΔΕΧΟΝΤΑΝ ΣΥΜΒΙΒΑΣΤΙΚΑ ΜΕ ΤΗΝ ΠΑΝΤΟΔΥΝΑΜΙΑ ΤΟΥ ΘΕΟΥ ΤΗΝ

ΠΟΛΛΑΠΛΟΤΗΤΑ ΤΩΝ ΚΟΣΜΩΝ ΚΑΤΑ ΤΗΝ ΒΟΥΛΗΣΗ ΤΟΥ, ΛΕΓΟΝΤΑΣ ΟΤΙ Η

ΦΥΣΗ ΩΣ ΑΙΤΙΑΚΟ ΔΗΜΙΟΥΡΓΗΜΑ, ΔΕΝ ΕΙΝΑΙ ΣΕ ΘΕΣΗ ΝΑ ΔΕΧΤΕΙ ΤΟΥΣ

ΕΠΙΠΛΕΟΝ ΚΟΣΜΟΥΣ.

7. ΟΙ ΣΧΟΛΑΣΤΙΚΟΙ ΘΕΟΠΟΙΗΣΑΝ ΤΟ ΑΠΕΙΡΟ ΣΥΜΠΑΝ. ΑΠΕΔΩΣΑΝ ΣΕ ΑΥΤΟ

ΙΔΙΟΤΗΤΕΣ ΧΩΡΙΣ ΝΑ ΒΑΣΙΣΤΟΥΝ ΣΤΑ ΜΑΘΗΜΑΤΙΚΑ Η ΣΤΑ ΠΕΙΡΑΜΑΤΑ,

ΑΛΛΑ ΣΤΗΝ ΜΕΤΑΦΥΣΙΚΗ -ΘΕΟΛΟΓΙΑ ΚΑΙ ΣΕ ΝΟΗΤΙΚΑ ΠΕΙΡΑΜΑΤΑ. Η

ΣΧΕΣΗ ΔΡΩΝΤΟΣ ΘΕΟΥ ΣΤΟΝ ΑΠΕΙΡΟ ΚΟΣΜΟ ΤΟΥ ΝΕΥΤΩΝΑ ΘΑ

Page 75: ΣΗΜΕΙΩΣΕΙΣ ΦΙΛΟΣΟΦΙΑΣ I

ΑΠΟΤΕΛΕΣΕΙ ΕΝΑ ΑΠΟ ΤΑ ΚΥΡΙΑ ΧΑΡΑΚΤΗΡΙΣΤΙΚΑ ΤΗΣ ΝΕΑΣ ΕΠΙΣΤΗΜΗΣ

ΤΟΥ 17ΟΥ ΑΙΩΝΑ ΚΑΙ ΣΕ ΚΑΜΙΑ ΠΕΡΙΠΤΩΣΗ ΔΕΝ ΘΑ ΠΡΕΠΕΙ ΝΑ ΘΕΩΡΗΘΕΙ

ΑΣΧΕΤΗ ΠΡΟΣ ΤΙΣ ΜΕΣΑΙΩΝΙΚΕΣ ΑΝΑΖΗΤΗΣΕΙΣ.

8. Η ΜΕΣΑΙΩΝΙΚΗ ΑΝΤΙΛΗΨΗ ΟΤΙ ΟΙ ΟΥΡΑΝΟΙ ΣΧΕΤΙΖΟΝΤΑΙ ΜΕ ΤΑ

ΠΝΕΥΜΑΤΑ ΚΑΙ ΕΙΝΑΙ ΑΣΥΓΚΡΙΤΑ ΑΝΩΤΕΡΟΙ ΑΠΟ ΤΟ ΥΠΟΣΕΛΗΝΙΟ ΧΩΡΙΟ

ΤΟΥ ΣΥΜΠΑΝΤΟΣ ΗΤΑΝ ΕΝΑ ΑΠΟ ΤΑ ΘΕΜΕΛΙΑ ΤΗΣ ΜΕΣΑΙΩΝΙΚΗΣ

ΧΡΙΣΤΙΑΝΙΚΗΣ ΚΟΣΜΟΑΝΤΙΛΗΨΗΣ.

9. ΣΤΟ ΛΑΤΙΝΙΚΟ ΜΕΣΑΙΩΝΑ ΟΙ ΣΥΖΗΤΗΣΕΙΣ ΑΦΟΡΟΥΣΑ ΚΥΡΙΩΣ ΤΟ ΕΡΩΤΗΜΑ

ΠΟΙΟ ΚΟΣΜΟΛΟΓΙΚΟ ΣΥΣΤΗΜΑ, ΤΟ ΑΡΙΣΤΟΤΕΛΙΚΟ Η ΤΟ ΠΤΟΛΕΜΑΪΚΟ

,ΠΑΡΙΣΤΑΝΕ ΚΑΛΥΤΕΡΑ ΤΗΝ ΦΥΣΙΚΗ ΠΡΑΓΜΑΤΙΚΟΤΗΤΑ.

ΑΡΙΣΤΟΤΕΛΗΣ

1. ΥΠΑΡΧΕΙ ΚΑΤΙ ΘΕΪΚΟ ΚΑΙ ΑΙΩΝΙΟ ΚΑΤΩ ΑΠΟ ΑΥΤΟ ΠΟΥ ΠΑΡΑΤΗΡΟΥΜΕ ΚΑΙ

ΚΙΝΕΙΤΑΙ ΑΕΝΑΑ

2. ΕΙΝΑΙ ΑΔΥΝΑΤΟΝ ΝΑ ΔΗΜΙΟΥΡΓΕΙΤΑΙ Ο ΚΟΣΜΟΣ ΑΦΕΝΟΣ ΚΑΙ ΑΦΕΤΕΡΟΥ

ΝΑ ΕΙΝΑΙ ΙΔΙΟΣ. ΘΕΜΕΛΙΩΝΕΤΑΙ Η ΜΟΝΑΔΙΚΟΤΗΤΑ ΤΟΥ ΚΟΣΜΟΥ. ΕΝΑ

ΣΥΝΟΛΟ ΑΚΙΝΗΤΩΝ ΚΙΝΟΥΝΤΩΝ ΑΠΟΤΕΛΕΙ ΤΟ ΑΙΤΙΟ ΤΗΣ ΟΥΡΑΝΙΑΣ

ΚΙΝΗΣΗΣ.

3. ΟΙ ΚΙΝΗΣΕΙΣ ΕΙΝΑΙ ΠΕΠΕΡΑΣΜΕΝΕΣ. ΕΑΝ ΕΙΝΑΙ ΙΔΙΕΣ ΚΑΙ ΤΑ ΣΤΟΙΧΕΙΑ

ΤΟΤΕ ΕΙΝΑΙ ΠΑΝΤΟΥ ΙΔΙΑ. ΕΝΑΣ ΚΟΣΜΟΣ ΜΕ ΔΙΑΦΟΡΕΤΙΚΟΥ ΕΙΔΟΥΣ

ΣΩΜΑΤΑ ΘΑ ΗΤΑΝ ΤΕΛΙΚΑ ΚΟΣΜΟΣ ΜΟΝΟ ΚΑΤ' ΟΝΟΜΑ ΑΦΟΥ Η

ΚΙΝΗΜΑΤΙΚΗ ΣΥΜΠΕΡΙΦΟΡΑ ΤΩΝ ΣΩΜΑΤΩΝ ΘΑ ΕΡΧΟΤΑΝ ΣΕ ΠΛΗΡΗ

ΑΝΤΙΘΕΣΗ ΜΕ ΤΙΣ ΑΝΤΙΛΗΨΕΙΣ ΤΟΥ.

4. Η ΚΙΝΗΣΗ ΔΕΝ ΕΙΝΑΙ ΠΟΤΕ ΑΥΘΟΡΜΗΤΗ: ΔΕΝ ΥΠΑΡΧΕΙ ΚΙΝΗΣΗ ΧΩΡΙΣ

ΚΙΝΟΥΝ. Η ΔΙΑΚΡΙΣΗ ΜΕΤΑΞΥ ΔΥΟ ΤΥΠΩΝ ΚΙΝΗΣΗΣ ΤΗΝ ΚΙΝΗΣΗ ΠΡΟΣ ΤΟ

ΦΥΣΙΚΟ ΤΟΠΟ ΤΟΥ ΚΙΝΟΥΜΕΝΟΥ ΣΩΜΑΤΟΣ (ΦΥΣΙΚΗ) ΚΑΙ ΤΗΝ ΚΙΝΗΣΗ

ΠΡΟΣ ΟΠΟΙΑΔΗΠΟΤΕ ΑΛΛΗ ΚΑΤΕΥΘΥΝΣΗ (ΕΞΑΝΑΓΚΑΣΜΕΝΗ Η ΒΙΑΙΗ).

5. ΔΕΝ ΥΠΑΡΧΕΙ ΑΠΕΙΡΟ ΕΝ ΕΝΕΡΓΕΙΑ ΑΛΛΑ ΜΟΝΟ ΕΝ ΔΥΝΑΜΕΙ ΟΠΩΣ ΔΕΝ

Page 76: ΣΗΜΕΙΩΣΕΙΣ ΦΙΛΟΣΟΦΙΑΣ I

ΥΠΑΡΧΕΙ ΚΑΙ ΑΡΧΗ ΤΟΥ ΣΥΜΠΑΝΤΟΣ.

6. Η ΤΕΛΕΙΟΤΗΤΑ ΜΠΟΡΕΙ ΝΑ ΘΕΩΡΗΘΕΙ ΜΕ ΔΥΟ ΤΡΟΠΟΥΣ. ΕΙΤΕ ΑΠΟΛΥΤΑ

ΕΙΤΕ ΣΧΕΤΙΚΑ (ΑΠΟΛΥΤΑ ΤΕΛΕΙΟΣ ΘΕΟΣ, ΣΧΕΤΙΚΑ ΚΑΛΟΣ ΓΙΑΤΡΟΣ)

7. ΑΡΝΗΣΗ ΥΠΑΡΞΗΣ ΠΟΛΛΩΝ ΚΟΣΜΩΝ. ΔΕΝ ΥΠΑΡΧΕΙ ΚΑΜΙΑ ΣΩΜΑΤΙΚΗ

ΜΑΖΑ ΠΕΡΑ ΑΠΟ ΤΟΥΣ ΟΥΡΑΝΟΥΣ. ΤΟ ΚΕΝΟ ΕΙΝΑΙ ΑΔΥΝΑΤΟΝ.

8. ΑΝΤΙΠΑΡΕΘΕΣΕ ΤΗΝ ΦΥΣΙΚΗ ΕΥΘΥΓΡΑΜΜΗ ΚΙΝΗΣΗ ΤΩΝ ΤΕΣΣΑΡΩΝ

ΥΠΟΣΕΛΗΝΙΩΝ ΣΤΟΙΧΕΙΩΝ ΜΕ ΤΙΣ ΠΑΡΑΤΗΡΟΥΜΕΝΕΣ , ΟΜΟΙΟΜΟΡΦΕΣ

ΦΑΙΝΟΜΕΝΙΚΑ ΚΥΚΛΙΚΕΣ ΚΙΝΗΣΕΙΣ ΤΩΝ ΠΛΑΝΗΤΩΝ ΚΑΙ ΤΩΝ ΑΠΛΑΝΩΝ

ΚΑΙ ΚΑΤΕΛΗΞΕ ΟΤΙ ΟΙ ΚΙΝΗΣΕΙΣ ΤΩΝ ΟΥΡΑΝΙΩΝ ΣΩΜΑΤΩΝ ΑΝΑΓΚΑΣΤΙΚΑ

ΘΑ ΠΡΕΠΕΙ ΝΑ ΣΥΣΧΕΤΙΣΤΟΥΝ ΜΕ ΤΗΝ ΥΠΑΡΞΗ ΕΝΟΣ ΠΕΜΠΤΟΥ ΣΤΟΙΧΕΙΟΥ

ΤΟΥ ΑΙΘΕΡΑ, ΤΟ ΟΠΟΙΟ ΕΙΝΑΙ ΑΙΩΝΙΟ ΚΑΙ ΔΕΝ ΥΠΟΚΕΙΤΑΙ ΟΥΤΕ ΣΕ

ΑΥΞΗΣΗ ΟΥΤΕ ΣΕ ΦΘΟΡΑ , ΑΛΛΑ ΕΙΝΑΙ ΑΓΕΡΑΣΤΟ ΚΑΙ ΑΝΑΛΛΟΙΩΤΟ ΚΑΙ

ΑΝΕΠΗΡΕΑΣΤΟ ΑΠΟ ΟΤΙΔΗΠΟΤΕ.

9. ΔΕΝ ΠΑΡΑΤΗΡΗΘΗΚΑΝ ΠΟΤΕ ΑΛΛΑΓΕΣ ΣΤΟΥΣ ΟΥΡΑΝΟΥΣ.

10.ΣΧΕΤΙΚΑ ΜΕ ΤΙΣ ΤΕΧΝΙΚΕΣ ΠΛΕΥΡΕΣ ΤΟΥ ΚΟΣΜΟΛΟΓΙΚΟΥ ΤΟΥ

ΣΥΣΤΗΜΑΤΟΣ ΑΝΑΦΕΡΕΙ ΟΤΙ ΟΙ ΚΙΝΗΣΕΙΣ ΕΙΝΑΙ ΑΡΙΘΜΗΤΙΚΑ

ΠΕΡΙΣΣΟΤΕΡΕΣ ΑΠΟ ΤΑ ΚΙΝΗΤΑ, ΕΙΝΑΙ ΦΑΝΕΡΟ ΣΕ ΑΥΤΟΥΣ ΠΟΥ ΕΧΟΥΝ

ΔΩΣΕΙ ΑΡΚΕΤΗ ΠΡΟΣΟΧΗ ΣΤΟ ΘΕΜΑ, ΓΙΑΤΙ Ο ΚΑΘΕ ΠΛΑΝΗΤΗΣ ΕΧΕΙ

ΠΕΡΙΣΣΟΤΕΡΕΣ ΑΠΟ ΜΙΑ ΚΙΝΗΣΗ.. ΕΠΙΠΛΕΟΝ ΑΠΕΔΩΣΕ ΣΤΑ ΟΥΡΑΝΙΑ

ΣΩΜΑΤΑ ΤΗ ΣΥΝΕΧΗ ΟΜΑΛΗ ΚΥΚΛΙΚΗ ΚΙΝΗΣΗ.

Επιστροφή στην κορυφή

Επισκόπηση του προφίλ των χρηστών Αποστολή προσωπικού μηνύματος

marlene

Χρήστης

Χρήστης

Page 77: ΣΗΜΕΙΩΣΕΙΣ ΦΙΛΟΣΟΦΙΑΣ I

Ένταξη: Sep 22, 2005

Δημοσιεύσεις: 2463

ΔημοσίευσηΔημοσιεύθηκε: Τρι Ιούν 19, 2007 10:29 pm Θέμα δημοσίευσης: Απάντηση

με παράθεση αυτού του μηνύματος

ΑΚΙΝΑΤΗΣ

1. ΔΕΝ ΜΠΟΡΕΙ ΝΑ ΑΠΟΔΕΙΧΤΕΙ Η ΠΡΟΤΑΣΗ ΠΕΡΙ ΑΡΧΗΣ ΤΟΥ ΚΟΣΜΟΥ.

2. Η ΘΕΪΚΗ ΠΑΝΤΟΔΥΝΑΜΙΑ ΜΠΟΡΕΙ ΝΑ ΠΑΡΑΓΑΓΕΙ ΓΕΓΟΝΟΤΑ ΤΑΥΤΟΧΡΟΝΑ,

ΣΥΝΕΠΩΣ ΝΑ ΔΗΜΙΟΥΡΓΗΣΕΙ ΕΝΑΝ ΚΟΣΜΟ ΧΩΡΙΣ ΧΡΟΝΙΚΗ ΑΡΧΗ, ΕΝΑΝ

ΑΙΩΝΙΑ ΥΠΑΡΧΟΝΤΑ ΚΟΣΜΟ.

3. ΤΟ ΚΙΝΟΥΝ ΣΤΗΝ ΠΕΡΙΠΤΩΣΗ ΤΗΣ ΦΥΣΙΚΗΣ ΚΙΝΗΣΗΣ ΕΙΝΑΙ Ο ΥΠΕΥΘΥΝΟΣ

ΤΗΣ ΑΡΧΙΚΗΣ ΓΕΝΕΣΗΣ Η ΤΟΥ ΣΧΗΜΑΤΙΣΜΟΥ ΤΟΥ ΣΩΜΑΤΟΣ ΜΑΚΡΙΑ ΑΠΟ

ΤΟΝ ΦΥΣΙΚΟ ΤΟΥ ΤΟΠΟ. ΑΠΟ ΚΕΙ ΚΑΙ ΠΕΡΑ ΤΟ ΣΩΜΑ ΔΕΝ ΑΠΑΙΤΕΙ

ΚΑΠΟΙΟ ΚΙΝΟΥΝ , ΑΛΛΑ ΑΠΛΩΣ ΑΚΟΛΟΥΘΕΙ ΤΗ ΦΥΣΙΚΗ ΡΟΗ ΤΩΝ

ΠΡΑΓΜΑΤΩΝ.

ΤΙΜΑΙΟΣ

1. Η ΥΛΗ ΔΕΝ ΕΧΕΙ ΑΡΧΗ ΑΛΛΑ ΥΠΟ ΠΡΟΫΠΟΘΕΣΕΙΣ ΜΟΝΟ ΤΟ ΣΥΜΠΑΝ ΕΤΣΙ

ΟΠΩΣ ΕΙΝΑΙ ΔΟΜΗΜΕΝΟ.

2. ΕΑΝ ΑΝΑΜΕΣΑ ΣΕ ΔΥΟ ΦΥΣΙΚΑ ΕΙΔΗ ΥΠΑΡΧΕΙ ΘΕΩΡΗΤΙΚΑ ΔΥΝΑΤΟΤΗΤΑ

ΕΜΦΑΝΙΣΗΣ ΕΝΔΙΑΜΕΣΟΥ ΤΥΠΟΥ, ΤΟΤΕ ΑΥΤΟΣ ΘΑ ΠΡΕΠΕΙ ΝΑ

ΠΡΑΓΜΑΤΟΠΟΙΗΘΕΙ ΜΕ ΤΗΝ ΕΠ' ΑΠΕΙΡΟΝ ΕΜΦΑΝΙΣΗ ΕΙΔΩΝ, ΓΙΑΤΙ ΑΛΛΙΩΣ

Η ΥΠΑΡΞΗ ΚΕΝΩΝ ΣΤΗΝ ΦΥΣΗ ΘΑ ΗΤΑΝ ΕΝΔΕΙΞΗ ΜΗ ΑΓΑΘΟΤΗΤΑΣ ΤΟΥ

ΔΗΜΙΟΥΡΓΟΥ.

3. Η ΟΙΚΟΔΟΜΗΣΗ ΤΟΥ ΚΟΣΜΟΥ ΑΠΑΙΤΗΣΕ ΤΗΝ ΟΛΟΤΗΤΑ ΚΑΙ ΤΩΝ

ΤΕΣΣΑΡΩΝ ΣΤΟΙΧΕΙΩΝ. ΤΗΝ ΜΟΡΦΗ ΤΟΥ ΘΕΪΚΟΥ ΓΕΝΟΥΣ ΤΗΝ ΕΦΤΙΑΞΕ

ΚΑΤΑ ΚΥΡΙΟ ΛΟΓΟ ΑΠΟ ΦΩΤΙΑ , ΩΣΤΕ ΝΑ ΕΙΝΑΙ ΟΤΙ ΠΙΟ ΛΑΜΠΡΟ ΚΑΙ ΠΙΟ

ΩΡΑΙΟ ΜΠΟΡΕΙ ΝΑ ΔΕΙ ΚΑΝΕΙΣ. (Β. ΚΑΛΦΑΣ 1993. "Η ΣΩΜΑΤΙΚΟΤΗΤΑ ΤΩΝ

Page 78: ΣΗΜΕΙΩΣΕΙΣ ΦΙΛΟΣΟΦΙΑΣ I

ΑΣΤΡΩΝ ΠΡΕΠΕΙ ΝΑ ΑΠΟΔΟΘΕΙ ΣΤΗΝ ΠΑΡΟΥΣΙΑ ΤΟΥ ΓΗΙΝΟΥ ΣΤΟΙΧΕΙΟΥ, Η

ΦΩΤΙΑ ΚΥΡΙΑΡΧΕΙ ΓΕΝΙΚΑ ΣΤΗ ΣΥΣΤΑΣΗ ΤΟΥΣ, ΕΝΩ ΤΑ ΑΛΛΑ ΔΥΟ

ΣΤΟΙΧΕΙΑ ΑΝΑΓΚΑΣΤΙΚΑ ΥΠΑΡΧΟΥΝ, ΛΟΓΩ ΤΗΣ ΠΛΑΤΩΝΙΚΗΣ ΑΝΤΙΛΗΨΗΣ

Η ΟΠΟΙΑ ΑΠΑΙΤΕΙ ΔΥΟ ΟΝΤΟΛΟΓΙΚΕΣ ΜΕΣΟΤΗΤΕΣ , ΜΕ ΤΗ ΜΟΡΦΗ

ΓΕΩΜΕΤΡΙΚΗΣ ΑΝΑΛΟΓΙΑΣ.

ΟΚΚΑΜ

1. ΟΙ ΟΥΡΑΝΟΙ ΕΙΝΑΙ ΑΦΘΑΡΤΟΙ ΑΠΟ ΟΠΟΙΟΝΔΗΠΟΤΕ ΠΑΡΑΓΟΝΤΑ ΑΛΛΑ Ο

ΘΕΟΣ, Ο ΟΠΟΙΟΣ ΥΠΕΡΦΥΣΙΚΑ ΔΗΜΙΟΥΡΓΗΣΕ ΤΟ ΚΟΣΜΟ, ΜΠΟΡΕΙ ΕΠΙΣΗΣ

ΥΠΕΡΦΥΣΙΚΑ ΝΑ ΤΟΝ ΚΑΤΑΣΤΡΕΨΕΙ.2. ΚΙΝΗΣΗ ΕΙΝΑΙ ΑΦΗΡΗΜΕΝΟΣ ΚΑΙ

ΠΛΑΣΜΑΤΙΚΟΣ ΟΡΟΣ ,ΕΝΑ ΟΥΣΙΑΣΤΙΚΟ ΤΟ ΟΠΟΙΟ ΔΕΝ ΑΝΤΙΣΤΟΙΧΕΙ ΣΕ

ΚΑΠΟΙΑ ΥΠΑΡΧΟΥΣΑ ΟΝΤΟΤΗΤΑ. (ΔΕΝ ΕΙΝΑΙ ΠΡΑΓΜΑ)

TH. BRADWARDINE

1. ΟΥΣΙΑΣΤΙΚΑ ΚΑΙ ΜΕ ΠΑΡΟΥΣΙΑ, Ο ΘΕΟΣ ΕΙΝΑΙ ΑΝΑΓΚΑΣΤΙΚΑ ΠΑΝΤΟΥ

ΣΤΟΝ ΚΟΣΜΟ.

2. ΕΠΙΣΗΣ ΠΕΡΑ ΑΠΟ ΤΟΝ ΠΡΑΓΜΑΤΙΚΟ ΚΟΣΜΟ Η ΣΕ ΕΝΑ ΦΑΝΤΑΣΤΙΚΟ

ΑΠΕΙΡΟ ΚΕΝΟ

3. ΠΡΑΓΜΑΤΙΚΑ ΑΥΤΟΣ ΜΠΟΡΕΙ ΝΑ ΚΛΗΘΕΙ ΑΧΑΝΗΣ ΚΑΙ ΑΠΕΡΙΟΡΙΣΤΟΣ

4. ΔΙΔΕΤΑΙ Η ΑΠΑΝΤΗΣΗ ΣΤΟΥΣ ΑΠΙΣΤΟΥΣ "ΠΟΥ ΕΙΝΑΙ Ο ΘΕΟΣ ΣΑΣ" ΚΑΙ ΠΟΥ

ΗΤΑΝ Ο ΘΕΟΣ ΠΡΙΝ ΤΗΝ ΔΗΜΙΟΥΡΓΙΑ.

5. ΥΠΕΡΑΣΠΙΣΤΗΚΕ ΤΗΝ ΑΠΟΨΗ ΟΤΙ ΤΟ ΣΥΜΠΑΝ ΠΕΡΙΒΑΛΛΕΤΑΙ ΑΠΟ ΑΠΕΙΡΟ

ΚΕΝΟ ΧΩΡΟ ΔΗΜΙΟΥΡΓΗΜΕΝΟ ΑΠΟ ΤΟΝ ΘΕΟ.

6. ΣΥΝΕΠΩΣ ΦΑΙΝΕΤΑΙ ΟΤΙ ΕΙΝΑΙ ΔΥΝΑΤΟΝ ΝΑ ΥΠΑΡΞΕΙ ΚΕΝΟ ΧΩΡΙΣ ΣΩΜΑ

ΑΛΛΑ ΜΕ ΚΑΝΕΝΑ ΤΡΟΠΟ ΧΩΡΙΣ ΤΟΝ ΘΕΟ.

ΓΚΡΟΣΣΕΤΕΣΤ

1. ΚΕΝΤΡΙΚΗ ΘΕΣΗ ΚΑΤΕΧΕΙ ΤΟ ΦΩΣ : Ο ΚΟΣΜΟΣ ΑΡΧΙΣΕ ΝΑ ΥΠΑΡΧΕΙ ΑΠΟ

ΤΗΝ ΣΤΙΓΜΗ ΠΟΥ Ο ΘΕΟΣ ΔΗΜΙΟΥΡΓΗΣΕ ΕΝΑ ΑΔΙΑΣΤΑΤΟ ΣΗΜΕΙΟ ΥΛΗΣ

ΚΑΙ ΤΗ ΜΟΡΦΗ ΤΟΥ ΕΝΑ ΣΗΜΕΙΟ ΦΩΤΟΣ ΧΩΡΙΣ ΔΙΑΣΤΑΣΕΙΣ.

Page 79: ΣΗΜΕΙΩΣΕΙΣ ΦΙΛΟΣΟΦΙΑΣ I

2. Ο ΚΟΣΜΟΣ ΕΙΝΑΙ ΟΜΟΓΕΝΗΣ ΚΑΙ ΟΙ ΟΥΡΑΝΟΙ ΑΠΟΤΕΛΟΥΝΤΑΙ ΑΠΟ ΕΝΑ

ΥΛΙΚΟ ΛΕΠΤΟΤΕΡΟ ΚΑΙ ΑΡΑΙΟΤΕΡΟ ΑΠΟ ΑΥΤΟ ΤΩΝ ΕΠΙΓΕΙΩΝ ΟΥΣΙΩΝ,

ΑΛΛΑ Η ΔΙΑΦΟΡΑ ΕΙΝΑΙ ΠΟΣΟΤΙΚΗ ΚΑΙ ΟΧΙ ΠΟΙΟΤΙΚΗ.

ΝΙΚΟΛΑΟΣ ΤΗΣ ΩΤΡΕΚΟΥΡΤ

1. Η ΚΕΝΤΡΙΚΗ ΤΟΥ ΘΕΣΗ ΑΦΟΡΑ ΤΗ ΜΗ ΥΠΑΡΞΗ ΟΡΙΟΥ ΣΤΟΝ ΦΙΛΟΣΟΦΙΚΟ

ΣΚΕΠΤΙΚΙΣΜΟ ΓΙΑ ΤΗΝ ΥΠΑΡΞΗ ΑΝΤΙΚΕΙΜΕΝΩΝ ΤΑ ΟΠΙΑ ΥΠΑΡΧΟΥΝ ΕΞΩ

ΑΠΟ ΤΟΝ ΝΟΥ ΜΑΣ.ΟΙ ΑΠΟΨΕΙΣ ΤΟΥ ΔΕΝ ΗΤΑΝ ΚΥΡΙΑΡΧΕΣ ΟΥΤΕ

ΜΠΟΡΕΣΑΝ ΝΑ ΕΚΦΡΑΣΟΥΝ ΣΕ ΜΙΑ ΕΠΟΧΗ ΕΠΙΣΤΗΜΟΝΙΚΗΣ ΠΡΟΟΔΟΥ

ΤΗΝ ΚΑΤΑΡΡΕΥΣΗ ΤΟΥ ΛΟΓΟΥ. Η ΣΗΜΑΣΙΑ ΑΥΤΟΥ ΤΟΥ ΜΕΣΑΙΩΝΙΚΟΥ

ΧΙΟΥΜ ΕΓΚΕΙΤΑΙ ΣΤΟ ΓΕΓΟΝΟΣ ΟΤΙ ΚΑΤΕΔΕΙΞΕ ΜΕΣΩ ΤΗΣ ΕΣΩΤΕΡΙΚΗΣ

ΚΡΙΤΙΚΗΣ ΜΙΑ ΣΥΝΕΠΗ ΚΑΙ ΔΥΝΑΤΗ ΑΠΟΛΗΞΗ ΤΗΣ ΣΚΕΨΗΣ ΤΗΣ ΕΠΟΧΗΣ

ΤΟΥ.

ΛΟΓΙΟΙ 13ΟΥ ΚΑΙ 14ΟΥ

1. Ο ΚΟΣΜΟΣ ΠΟΥ ΥΠΑΡΧΕΙ ΣΗΜΕΡΑ ΔΗΜΙΟΥΡΓΗΘΗΚΕ ΑΛΛΑ Η ΥΛΗ ΕΙΝΑΙ

ΑΙΩΝΙΑ.

2. Ο ΧΡΟΝΟΣ ΑΡΧΙΖΕΙ ΟΤΑΝ Ο ΘΕΟΣ ΔΗΜΙΟΥΡΓΗΣΕ ΤΗΝ ΠΡΩΤΗ ΚΙΝΗΣΗ,

ΣΥΝΕΠΩΣ Η ΑΙΩΝΙΟΤΗΤΑ ΤΑΥΤΙΖΕΤΑΙ ΜΕ ΤΗΝ ΠΛΗΡΗ ΔΙΑΡΚΕΙΑ ΤΟΥ

ΧΡΟΝΟΥ.

3. ΔΕΝ ΜΠΟΡΕΙ ΝΑ ΑΠΟΔΕΙΧΤΕΙ Η ΠΡΟΤΑΣΗ ΠΕΡΙ ΑΡΧΗΣ ΤΟΥ ΚΟΣΜΟΥ.

4. ΔΕΝ ΥΠΑΡΧΕΙ ΑΠΟΛΥΤΑ ΤΕΛΕΙΟΣ ΚΟΣΜΟΣ ΑΛΛΑ ΜΟΝΟΝ ΣΧΕΤΙΚΑ.

5. Ο ΘΕΟΣ ΚΑΤΑ ΤΟΥΣ ΣΧΟΛΑΣΤΙΚΟΥΣ ΕΦΤΙΑΞΕ ΤΟΝ ΚΟΣΜΟ ΟΣΟ ΚΑΛΟ

ΘΕΩΡΗΣΕ ΟΤΙ ΧΡΕΙΑΖΕΤΑΙ.

6. ΔΕΧΟΝΤΑΝ ΣΥΜΒΙΒΑΣΤΙΚΑ ΜΕ ΤΗΝ ΠΑΝΤΟΔΥΝΑΜΙΑ ΤΟΥ ΘΕΟΥ ΤΗΝ

ΠΟΛΛΑΠΛΟΤΗΤΑ ΤΩΝ ΚΟΣΜΩΝ ΚΑΤΑ ΤΗΝ ΒΟΥΛΗΣΗ ΤΟΥ, ΛΕΓΟΝΤΑΣ ΟΤΙ Η

ΦΥΣΗ ΩΣ ΑΙΤΙΑΚΟ ΔΗΜΙΟΥΡΓΗΜΑ, ΔΕΝ ΕΙΝΑΙ ΣΕ ΘΕΣΗ ΝΑ ΔΕΧΤΕΙ ΤΟΥΣ

ΕΠΙΠΛΕΟΝ ΚΟΣΜΟΥΣ.

7. ΟΙ ΣΧΟΛΑΣΤΙΚΟΙ ΘΕΟΠΟΙΗΣΑΝ ΤΟ ΑΠΕΙΡΟ ΣΥΜΠΑΝ. ΑΠΕΔΩΣΑΝ ΣΕ ΑΥΤΟ

Page 80: ΣΗΜΕΙΩΣΕΙΣ ΦΙΛΟΣΟΦΙΑΣ I

ΙΔΙΟΤΗΤΕΣ ΧΩΡΙΣ ΝΑ ΒΑΣΙΣΤΟΥΝ ΣΤΑ ΜΑΘΗΜΑΤΙΚΑ Η ΣΤΑ ΠΕΙΡΑΜΑΤΑ,

ΑΛΛΑ ΣΤΗΝ ΜΕΤΑΦΥΣΙΚΗ -ΘΕΟΛΟΓΙΑ ΚΑΙ ΣΕ ΝΟΗΤΙΚΑ ΠΕΙΡΑΜΑΤΑ. Η

ΣΧΕΣΗ ΔΡΩΝΤΟΣ ΘΕΟΥ ΣΤΟΝ ΑΠΕΙΡΟ ΚΟΣΜΟ ΤΟΥ ΝΕΥΤΩΝΑ ΘΑ

ΑΠΟΤΕΛΕΣΕΙ ΕΝΑ ΑΠΟ ΤΑ ΚΥΡΙΑ ΧΑΡΑΚΤΗΡΙΣΤΙΚΑ ΤΗΣ ΝΕΑΣ ΕΠΙΣΤΗΜΗΣ

ΤΟΥ 17ΟΥ ΑΙΩΝΑ ΚΑΙ ΣΕ ΚΑΜΙΑ ΠΕΡΙΠΤΩΣΗ ΔΕΝ ΘΑ ΠΡΕΠΕΙ ΝΑ ΘΕΩΡΗΘΕΙ

ΑΣΧΕΤΗ ΠΡΟΣ ΤΙΣ ΜΕΣΑΙΩΝΙΚΕΣ ΑΝΑΖΗΤΗΣΕΙΣ.

8. Η ΜΕΣΑΙΩΝΙΚΗ ΑΝΤΙΛΗΨΗ ΟΤΙ ΟΙ ΟΥΡΑΝΟΙ ΣΧΕΤΙΖΟΝΤΑΙ ΜΕ ΤΑ

ΠΝΕΥΜΑΤΑ ΚΑΙ ΕΙΝΑΙ ΑΣΥΓΚΡΙΤΑ ΑΝΩΤΕΡΟΙ ΑΠΟ ΤΟ ΥΠΟΣΕΛΗΝΙΟ ΧΩΡΙΟ

ΤΟΥ ΣΥΜΠΑΝΤΟΣ ΗΤΑΝ ΕΝΑ ΑΠΟ ΤΑ ΘΕΜΕΛΙΑ ΤΗΣ ΜΕΣΑΙΩΝΙΚΗΣ

ΧΡΙΣΤΙΑΝΙΚΗΣ ΚΟΣΜΟΑΝΤΙΛΗΨΗΣ.

9. ΣΤΟ ΛΑΤΙΝΙΚΟ ΜΕΣΑΙΩΝΑ ΟΙ ΣΥΖΗΤΗΣΕΙΣ ΑΦΟΡΟΥΣΑ ΚΥΡΙΩΣ ΤΟ ΕΡΩΤΗΜΑ

ΠΟΙΟ ΚΟΣΜΟΛΟΓΙΚΟ ΣΥΣΤΗΜΑ, ΤΟ ΑΡΙΣΤΟΤΕΛΙΚΟ Η ΤΟ ΠΤΟΛΕΜΑΪΚΟ

,ΠΑΡΙΣΤΑΝΕ ΚΑΛΥΤΕΡΑ ΤΗΝ ΦΥΣΙΚΗ ΠΡΑΓΜΑΤΙΚΟΤΗΤΑ.

ΑΡΙΣΤΟΤΕΛΗΣ

1. ΥΠΑΡΧΕΙ ΚΑΤΙ ΘΕΪΚΟ ΚΑΙ ΑΙΩΝΙΟ ΚΑΤΩ ΑΠΟ ΑΥΤΟ ΠΟΥ ΠΑΡΑΤΗΡΟΥΜΕ ΚΑΙ

ΚΙΝΕΙΤΑΙ ΑΕΝΑΑ

2. ΕΙΝΑΙ ΑΔΥΝΑΤΟΝ ΝΑ ΔΗΜΙΟΥΡΓΕΙΤΑΙ Ο ΚΟΣΜΟΣ ΑΦΕΝΟΣ ΚΑΙ ΑΦΕΤΕΡΟΥ

ΝΑ ΕΙΝΑΙ ΙΔΙΟΣ. ΘΕΜΕΛΙΩΝΕΤΑΙ Η ΜΟΝΑΔΙΚΟΤΗΤΑ ΤΟΥ ΚΟΣΜΟΥ. ΕΝΑ

ΣΥΝΟΛΟ ΑΚΙΝΗΤΩΝ ΚΙΝΟΥΝΤΩΝ ΑΠΟΤΕΛΕΙ ΤΟ ΑΙΤΙΟ ΤΗΣ ΟΥΡΑΝΙΑΣ

ΚΙΝΗΣΗΣ.

3. ΟΙ ΚΙΝΗΣΕΙΣ ΕΙΝΑΙ ΠΕΠΕΡΑΣΜΕΝΕΣ. ΕΑΝ ΕΙΝΑΙ ΙΔΙΕΣ ΚΑΙ ΤΑ ΣΤΟΙΧΕΙΑ

ΤΟΤΕ ΕΙΝΑΙ ΠΑΝΤΟΥ ΙΔΙΑ. ΕΝΑΣ ΚΟΣΜΟΣ ΜΕ ΔΙΑΦΟΡΕΤΙΚΟΥ ΕΙΔΟΥΣ

ΣΩΜΑΤΑ ΘΑ ΗΤΑΝ ΤΕΛΙΚΑ ΚΟΣΜΟΣ ΜΟΝΟ ΚΑΤ' ΟΝΟΜΑ ΑΦΟΥ Η

ΚΙΝΗΜΑΤΙΚΗ ΣΥΜΠΕΡΙΦΟΡΑ ΤΩΝ ΣΩΜΑΤΩΝ ΘΑ ΕΡΧΟΤΑΝ ΣΕ ΠΛΗΡΗ

ΑΝΤΙΘΕΣΗ ΜΕ ΤΙΣ ΑΝΤΙΛΗΨΕΙΣ ΤΟΥ.

4. Η ΚΙΝΗΣΗ ΔΕΝ ΕΙΝΑΙ ΠΟΤΕ ΑΥΘΟΡΜΗΤΗ: ΔΕΝ ΥΠΑΡΧΕΙ ΚΙΝΗΣΗ ΧΩΡΙΣ

ΚΙΝΟΥΝ. Η ΔΙΑΚΡΙΣΗ ΜΕΤΑΞΥ ΔΥΟ ΤΥΠΩΝ ΚΙΝΗΣΗΣ ΤΗΝ ΚΙΝΗΣΗ ΠΡΟΣ ΤΟ

Page 81: ΣΗΜΕΙΩΣΕΙΣ ΦΙΛΟΣΟΦΙΑΣ I

ΦΥΣΙΚΟ ΤΟΠΟ ΤΟΥ ΚΙΝΟΥΜΕΝΟΥ ΣΩΜΑΤΟΣ (ΦΥΣΙΚΗ) ΚΑΙ ΤΗΝ ΚΙΝΗΣΗ

ΠΡΟΣ ΟΠΟΙΑΔΗΠΟΤΕ ΑΛΛΗ ΚΑΤΕΥΘΥΝΣΗ (ΕΞΑΝΑΓΚΑΣΜΕΝΗ Η ΒΙΑΙΗ).

5. ΔΕΝ ΥΠΑΡΧΕΙ ΑΠΕΙΡΟ ΕΝ ΕΝΕΡΓΕΙΑ ΑΛΛΑ ΜΟΝΟ ΕΝ ΔΥΝΑΜΕΙ ΟΠΩΣ ΔΕΝ

ΥΠΑΡΧΕΙ ΚΑΙ ΑΡΧΗ ΤΟΥ ΣΥΜΠΑΝΤΟΣ.

6. Η ΤΕΛΕΙΟΤΗΤΑ ΜΠΟΡΕΙ ΝΑ ΘΕΩΡΗΘΕΙ ΜΕ ΔΥΟ ΤΡΟΠΟΥΣ. ΕΙΤΕ ΑΠΟΛΥΤΑ

ΕΙΤΕ ΣΧΕΤΙΚΑ (ΑΠΟΛΥΤΑ ΤΕΛΕΙΟΣ ΘΕΟΣ, ΣΧΕΤΙΚΑ ΚΑΛΟΣ ΓΙΑΤΡΟΣ)

7. ΑΡΝΗΣΗ ΥΠΑΡΞΗΣ ΠΟΛΛΩΝ ΚΟΣΜΩΝ. ΔΕΝ ΥΠΑΡΧΕΙ ΚΑΜΙΑ ΣΩΜΑΤΙΚΗ

ΜΑΖΑ ΠΕΡΑ ΑΠΟ ΤΟΥΣ ΟΥΡΑΝΟΥΣ. ΤΟ ΚΕΝΟ ΕΙΝΑΙ ΑΔΥΝΑΤΟΝ.

8. ΑΝΤΙΠΑΡΕΘΕΣΕ ΤΗΝ ΦΥΣΙΚΗ ΕΥΘΥΓΡΑΜΜΗ ΚΙΝΗΣΗ ΤΩΝ ΤΕΣΣΑΡΩΝ

ΥΠΟΣΕΛΗΝΙΩΝ ΣΤΟΙΧΕΙΩΝ ΜΕ ΤΙΣ ΠΑΡΑΤΗΡΟΥΜΕΝΕΣ , ΟΜΟΙΟΜΟΡΦΕΣ

ΦΑΙΝΟΜΕΝΙΚΑ ΚΥΚΛΙΚΕΣ ΚΙΝΗΣΕΙΣ ΤΩΝ ΠΛΑΝΗΤΩΝ ΚΑΙ ΤΩΝ ΑΠΛΑΝΩΝ

ΚΑΙ ΚΑΤΕΛΗΞΕ ΟΤΙ ΟΙ ΚΙΝΗΣΕΙΣ ΤΩΝ ΟΥΡΑΝΙΩΝ ΣΩΜΑΤΩΝ ΑΝΑΓΚΑΣΤΙΚΑ

ΘΑ ΠΡΕΠΕΙ ΝΑ ΣΥΣΧΕΤΙΣΤΟΥΝ ΜΕ ΤΗΝ ΥΠΑΡΞΗ ΕΝΟΣ ΠΕΜΠΤΟΥ ΣΤΟΙΧΕΙΟΥ

ΤΟΥ ΑΙΘΕΡΑ, ΤΟ ΟΠΟΙΟ ΕΙΝΑΙ ΑΙΩΝΙΟ ΚΑΙ ΔΕΝ ΥΠΟΚΕΙΤΑΙ ΟΥΤΕ ΣΕ

ΑΥΞΗΣΗ ΟΥΤΕ ΣΕ ΦΘΟΡΑ , ΑΛΛΑ ΕΙΝΑΙ ΑΓΕΡΑΣΤΟ ΚΑΙ ΑΝΑΛΛΟΙΩΤΟ ΚΑΙ

ΑΝΕΠΗΡΕΑΣΤΟ ΑΠΟ ΟΤΙΔΗΠΟΤΕ.

9. ΔΕΝ ΠΑΡΑΤΗΡΗΘΗΚΑΝ ΠΟΤΕ ΑΛΛΑΓΕΣ ΣΤΟΥΣ ΟΥΡΑΝΟΥΣ.

10.ΣΧΕΤΙΚΑ ΜΕ ΤΙΣ ΤΕΧΝΙΚΕΣ ΠΛΕΥΡΕΣ ΤΟΥ ΚΟΣΜΟΛΟΓΙΚΟΥ ΤΟΥ

ΣΥΣΤΗΜΑΤΟΣ ΑΝΑΦΕΡΕΙ ΟΤΙ ΟΙ ΚΙΝΗΣΕΙΣ ΕΙΝΑΙ ΑΡΙΘΜΗΤΙΚΑ

ΠΕΡΙΣΣΟΤΕΡΕΣ ΑΠΟ ΤΑ ΚΙΝΗΤΑ, ΕΙΝΑΙ ΦΑΝΕΡΟ ΣΕ ΑΥΤΟΥΣ ΠΟΥ ΕΧΟΥΝ

ΔΩΣΕΙ ΑΡΚΕΤΗ ΠΡΟΣΟΧΗ ΣΤΟ ΘΕΜΑ, ΓΙΑΤΙ Ο ΚΑΘΕ ΠΛΑΝΗΤΗΣ ΕΧΕΙ

ΠΕΡΙΣΣΟΤΕΡΕΣ ΑΠΟ ΜΙΑ ΚΙΝΗΣΗ.. ΕΠΙΠΛΕΟΝ ΑΠΕΔΩΣΕ ΣΤΑ ΟΥΡΑΝΙΑ

ΣΩΜΑΤΑ ΤΗ ΣΥΝΕΧΗ ΟΜΑΛΗ ΚΥΚΛΙΚΗ ΚΙΝΗΣΗ.

11.ΣΧΕΤΙΚΑ ΜΕ ΤΗΝ ΚΙΝΗΣΗ ΤΩΝ ΠΛΑΝΗΤΩΝ ΔΙΑΚΡΙΝΕΙ ΔΥΟ ΕΙΔΩΝ ΑΥΛΕΣ

ΟΝΤΟΤΗΤΕΣ ΟΙ ΟΠΟΙΕΣ ΕΙΝΑΙ ΣΕ ΘΕΣΗ ΝΑ ΚΙΝΗΣΟΥΝ ΤΙΣ ΤΡΟΧΙΕΣ ΤΟΥΣ,

ΤΗΝ ΨΥΧΗ ΚΑΙ ΤΗΝ ΔΙΑΝΟΙΑ. ΑΝΤΙΣΤΟΙΧΟΥΝ ΣΤΙΣ ΔΥΟ ΑΝΤΙΛΗΨΕΙΣ ΓΙΑ

ΤΟΥΣ ΚΙΝΗΤΗΡΕΣ ΤΩΝ ΠΛΑΝΗΤΩΝ, ΤΟΥΣ ΕΣΩΤΕΡΙΚΟΥΣ ΚΑΙ ΤΟΥΣ

Page 82: ΣΗΜΕΙΩΣΕΙΣ ΦΙΛΟΣΟΦΙΑΣ I

ΕΞΩΤΕΡΙΚΟΥΣ.

12.ΣΧΕΤΙΚΑ ΜΕ ΤΟ "ΠΟΙΟΣ ΚΙΝΕΙ ΤΟΥΣ ΟΥΡΑΝΟΥΣ" ΘΑ ΑΙΤΙΟΛΟΓΗΣΕΙ ΤΗΝ

ΦΥΣΙΚΗ ΑΕΝΑΗ ΚΙΝΗΣΗ ΤΟΥ ΑΙΘΕΡΑ ΛΕΓΟΝΤΑΣ "ΟΤΙ ΔΕΝ ΥΠΟΚΕΙΤΑΙ ΣΕ

ΚΟΠΩΣΗ, ΕΠΕΙΔΗ ΔΕΝ ΕΧΕΙ ΧΡΕΙΑ ΑΠΟ ΚΑΜΙΑ ΒΙΑΙΗ ΑΝΑΓΚΗ ΠΟΥ ΝΑ ΤΟΝ

ΚΑΤΕΧΕΙ ΚΑΙ ΝΑ ΤΟΝ ΕΜΠΟΔΙΖΕΙ ΝΑ ΣΥΜΠΕΡΙΦΕΡΕΤΑΙ ΔΙΑΦΟΡΕΤΙΚΑ ΑΠΟ

ΟΤΙ ΕΙΝΑΙ ΣΤΗ ΦΥΣΗ ΤΟΥ ΦΤΙΑΓΜΕΝΟΣ ΝΑ ΕΝΕΡΓΕΙ. ΕΝΩ ΣΤΑ

"ΜΕΤΑΦΥΣΙΚΑ" ΜΕ ΜΙΑ ΑΝΤΙΤΙΘΕΜΕΝΗ ΑΠΟΨΗ ΣΥΖΗΤΑ ΓΙΑ ΤΟ ΠΡΩΤΟ

ΑΚΙΝΗΤΟ ΚΙΝΟΥΝ, ΤΟ ΟΠΙΟ ΕΙΝΑΙ Η ΑΙΤΙΑ ΟΛΩΝ ΤΩΝ ΚΙΝΗΣΕΩΝ ΚΑΙ

ΠΑΡΑΓΕΙ ΤΗΝ ΚΙΝΗΣΗ "ΜΕΣΩ ΤΗΣ ΑΓΑΠΗΣ" ΠΟΥ ΤΟΥ ΕΠΙΔΕΙΚΝΥΟΥΝ ΤΑ

ΑΛΛΑ ΚΙΝΗΤΑ ΣΩΜΑΤΑ. ΕΤΣΙ ΤΟ ΑΡΙΣΤΟΤΕΛΙΚΟ ΔΟΓΜΑ "ΟΤΙ ΚΙΝΕΙΤΑΙ

ΠΡΕΠΕΙ ΝΑ ΚΙΝΕΙΤΑΙ ΑΠΟ ΚΑΤΙ" ΔΕΝ ΟΔΗΓΕΙ ΣΕ ΑΠΕΙΡΗ ΑΝΑΔΡΟΜΗ ,

ΕΡΧΕΤΑΙ ΣΕ ΕΝΑ ΤΕΛΟΣ ΜΕ ΤΗΝ ΑΓΑΠΗ ΠΡΟΣ ΤΟ ΠΡΩΤΟ ΚΙΝΟΥΝ.

13.Η ΓΗ ΗΡΕΜΕΙ ΣΤΟ ΚΕΝΤΡΟ ΤΟΥ ΣΥΜΠΑΝΤΟΣ ΚΑΙ ΕΙΝΑΙ ΣΦΑΙΡΙΚΗ.

14.Ο ΧΩΡΟΣ ΑΝΤΙ ΝΑ ΕΙΝΑΙ ΕΝΑ ΟΥΔΕΤΕΡΟ, ΟΜΟΓΕΝΕΣ ΠΛΑΙΣΙΟ(ΣΕ

ΑΝΑΛΟΓΙΑ ΜΕ ΤΗΝ ΣΥΓΧΡΟΝΗ ΑΝΤΙΛΗΨΗ ΤΟΥ ΓΕΩΜΕΤΡΙΚΟΥ ΧΩΡΟΥ -

ΚΟΣΜΟΣ ΧΩΡΟΥ) ΜΕΣΑ ΣΤΟ ΟΠΟΙΟ ΣΥΜΒΑΙΝΟΥΝ ΤΑ ΓΕΓΟΝΟΤΑ , ΑΠΟΚΤΑ

ΙΔΙΟΤΗΤΕΣ (ΚΟΣΜΟΣ ΤΟΠΟΥ).

15.Η ΦΥΣΗ ΤΗΣ ΟΥΡΑΝΙΑΣ ΠΕΡΙΟΧΗΣ ΕΙΝΑΙ ΑΝΩΤΕΡΗ ΚΑΙ ΣΧΕΔΟΝ ΘΕΙΑ. Η

ΥΠΟΣΕΛΗΝΙΑ ΠΕΡΙΟΧΗ ΕΙΝΑΙ Η ΣΚΗΝΗ ΤΗΣ ΓΕΝΕΣΗΣ ΤΗΣ ΦΘΟΡΑΣ ΚΑΙ

ΤΗΣ ΑΣΤΑΘΕΙΑΣ.

ΟΡΕΜ

1. Ο ΚΟΣΜΟΣ ΔΗΜΙΟΥΡΓΕΙΤΑΙ ΒΑΣΕΙ ΤΗΣ ΠΙΣΤΗΣ, ΑΛΛΑ Η ΛΟΓΙΚΗ ΑΝΑΛΥΣΗ

ΤΟΥ ΥΠΕΡΣΕΛΗΝΙΟΥ ΚΟΣΜΟΥ ΟΔΗΓΕΙ ΣΤΟ ΣΥΜΠΕΡΑΣΜΑ ΟΤΙ Ο ΚΟΣΜΟΣ

ΔΕΝ ΜΠΟΡΕΙ ΝΑ ΕΧΕΙ ΔΗΜΙΟΥΡΓΗΘΕΙ Η ΝΑ ΚΑΤΑΣΤΡΑΦΕΙ ΜΕ ΦΥΣΙΚΑ

ΜΕΣΑ, ΜΕ ΤΡΟΠΟ ΛΟΓΙΚΑ ΚΑΤΑΝΟΗΤΟ. ΔΕΝ ΜΠΟΡΕΙ ΝΑ ΕΙΝΑΙ ΑΙΩΝΙΟΣ

ΕΠΕΙΔΗ ΕΧΕΙ ΔΗΜΙΟΥΡΓΗΘΕΙ ΜΕ ΤΗ ΘΕΛΗΣΗ ΤΟΥ ΘΕΟΥ ΕΚ ΤΟΥ ΜΗΔΕΝΟΣ.

2. ΜΕΤΑ ΤΟ 1277 ΜΕ ΝΟΗΤΙΚΑ ΠΕΙΡΑΜΑΤΑ ΥΠΟΝΟΜΕΥΕΙ ΤΗΝ ΑΡΙΣΤΟΤΕΛΙΚΗ

Page 83: ΣΗΜΕΙΩΣΕΙΣ ΦΙΛΟΣΟΦΙΑΣ I

ΦΥΣΙΚΗ ΔΙΔΑΣΚΑΛΙΑ. ΑΦΟΥ ΚΑΤ' ΑΥΤΟΝ ΔΕΝ ΥΠΑΡΧΕΙ ΚΙΝΗΣΗ ΣΤΟ ΚΕΝΟ

ΤΟΤΕ Η ΚΙΝΗΣΗ ΠΕΡΙΟΡΙΖΕΤΑΙ ΑΡΙΣΤΟΤΕΛΙΚΑ ΑΝΑΜΕΣΑ ΣΤΟ ΑΝΩΤΕΡΟ ΚΑΙ

ΚΑΤΩΤΕΡΟ ΣΗΜΕΙΟ ΤΗΣ ΓΙΑ ΤΟΝ ΚΟΣΜΟ ΜΑΣ, ΤΑ ΟΠΟΙΑ ΕΙΝΑΙ ΤΟ

ΕΣΩΤΕΡΙΚΟ ΚΕΛΥΦΟΣ ΤΗΣ ΣΕΛΗΝΙΑΣ ΣΦΑΙΡΑΣ ΓΙΑ ΤΗΝ ΦΩΤΙΑ ΚΑΙ ΤΟ

ΚΕΝΤΡΟ ΤΗΣ ΓΗΣ ΓΙΑ ΤΑ ΓΗΙΝΑ ΣΤΟΙΧΕΙΑ. ΕΑΝ Ο ΘΕΟΣ ΚΑΤΑΣΚΕΥΑΣΕΙ

ΑΛΛΟΝ ΚΟΣΜΟ, Η ΓΗ ΔΕΝ ΘΑ ΕΛΚΕΤΑΙ ΠΡΟΣ ΤΟ ΚΕΝΤΡΟ ΑΥΤΟΥ ΤΟΥ

ΕΞΩΤΕΡΙΚΟΥ ΑΛΛΟΥ ΚΟΣΜΟΥ ΑΠΛΩΣ ΛΟΓΩ ΤΗΣ ΑΔΥΝΑΜΙΑΣ ΚΙΝΗΣΗΣ ΣΤΟ

ΚΕΝΟ, ΕΤΣΙ Ο ΘΕΟΣ ΘΑ; ΜΠΟΡΟΥΣΕ ΝΑ ΚΑΤΑΣΚΕΥΑΣΕΙ ΕΝΑ ΝΕΟ ΚΟΣΜΟ.

3. ΥΠΕΡΑΣΠΙΣΤΗΚΕ ΤΗΝ ΑΠΟΨΗ ΟΤΙ ΤΟ ΣΥΜΠΑΝ ΠΕΡΙΒΑΛΛΕΤΑΙ ΑΠΟ ΑΠΕΙΡΟ

ΚΕΝΟ ΧΩΡΟ ΔΗΜΙΟΥΡΓΗΜΕΝΟ ΑΠΟ ΤΟΝ ΘΕΟ

4. ΔΕΧΕΤΑΙ ΟΛΑ ΤΑ ΕΠΙΧΕΙΡΗΜΑΤΑ ΤΟΥ ΜΠΟΥΡΙΝΤΑΝ ΠΕΡΙ ΤΟΥ ΕΥΛΟΓΟΥ ΤΗΣ

ΚΙΝΗΣΗΣ ΤΗΣ ΓΗΣ ΠΛΗΝ ΠΡΟΦΑΝΩΣ ΤΟΥ ΚΑΤΑΛΥΤΙΚΟΥ ΓΙΑ ΤΟΝ

ΜΠΟΥΡΙΝΤΑΝ ΕΠΙΧΕΙΡΗΜΑΤΟΣ - ΤΟ ΟΠΟΙΟ ΕΙΝΑΙ ΟΜΩΣ ΑΠΟΤΕΛΕΣΜΑ ΤΗΣ

ΔΙΚΗΣ ΤΟΥ ΚΑΙ ΜΟΝΟ ΘΕΩΡΙΑΣ ΠΕΡΙ IMPETUS - ΚΑΙ ΚΑΤΑΛΗΓΕΙ ΡΗΤΑ ΟΤΙ

ΕΙΝΑΙ ΛΟΓΙΚΑ ΑΔΥΝΑΤΟΝ ΝΑ ΑΠΟΔΕΙΞΟΥΜΕ ΤΗΝ ΚΙΝΗΣΗ Η ΤΗΝ ΑΚΙΝΗΣΙΑ

ΤΗΣ ΓΗΣ ΚΑΤΙ ΔΗΛΑΔΗ ΑΝΑΛΟΓΟ ΜΕ ΤΗ ΡΗΤΟΡΙΚΗ ΤΟΥ ΓΑΛΙΛΑΙΟΥ.

5. ΘΕΩΡΗΣΕ ΡΗΤΑ ΟΤΙ Η ΘΡΗΣΚΕΥΤΙΚΗ ΠΙΣΤΗ ΕΙΝΑΙ ΙΣΧΥΡΟΤΕΡΗ ΤΟΥ ΟΡΘΟΥ

ΛΟΓΟΥ ΣΥΝΕΠΩΣ Η ΓΗ ΕΙΝΑΙ ΑΚΙΝΗΤΗ.

6. ΘΕΩΡΗΣΕ ΜΙΚΡΗ ΤΗΝ ΠΙΘΑΝΟΤΗΤΑ ΟΤΙ ΤΑ ΠΛΑΝΗΤΙΚΑ ΣΩΜΑΤΑ ΘΑ

ΕΠΙΣΤΡΕΨΟΥΝ ΣΤΗΝ ΑΡΧΙΚΗ ΤΟΥΣ ΘΕΣΗ ΜΕΤΑ ΑΠΟ 36000 ΧΡΟΝΙΑ.

7. ΘΕΩΡΗΣΕ ΑΚΙΝΗΤΗ ΤΗΝ ΓΗ ΟΠΩΣ Ο ΜΠΟΥΡΙΝΤΑΝ ΜΕ ΒΑΣΗ ΤΑ ΒΙΒΛΙΚΑ

ΚΕΙΜΕΝΑ ΑΛΛΑ ΠΑΛΙΝΩΔΗΣΕ ΘΕΤΟΝΤΑΣ ΕΝΑ ΣΥΝΟΛΟ ΕΠΙΧΕΙΡΗΜΑΤΩΝ ΓΙΑ

ΤΗΝ ΚΑΤΑΔΕΙΞΗ ΤΗΣ ΟΙΚΟΝΟΜΙΑΣ ΠΟΥ ΕΠΙΤΥΓΧΑΝΟΥΜΕ ΑΝ ΚΙΝΕΙΤΑΙ Η ΓΗ

ΚΑΙ ΟΧΙ ΟΙ ΟΥΡΑΝΟΙ ΙΣΩΣ ΦΟΒΟΥΜΕΝΟΣ ΤΗΝ ΕΚΚΛΗΣΙΑΣΤΙΚΗ

ΑΝΤΙΔΡΑΣΗ.

ΜΠΟΥΡΙΝΤΑΝ

1. Ο ΚΟΣΜΟΣ ΔΗΜΙΟΥΡΓΕΙΤΑΙ ΒΑΣΕΙ ΤΗΣ ΠΙΣΤΗΣ, ΑΛΛΑ Η ΛΟΓΙΚΗ ΑΝΑΛΥΣΗ

Page 84: ΣΗΜΕΙΩΣΕΙΣ ΦΙΛΟΣΟΦΙΑΣ I

ΤΟΥ ΥΠΕΡΣΕΛΗΝΙΟΥ ΚΟΣΜΟΥ ΟΔΗΓΕΙ ΣΤΟ ΣΥΜΠΕΡΑΣΜΑ ΟΤΙ Ο ΚΟΣΜΟΣ

ΔΕΝ ΜΠΟΡΕΙ ΝΑ ΕΧΕΙ ΔΗΜΙΟΥΡΓΗΘΕΙ Η ΝΑ ΚΑΤΑΣΤΡΑΦΕΙ ΜΕ ΦΥΣΙΚΑ

ΜΕΣΑ, ΜΕ ΤΡΟΠΟ ΛΟΓΙΚΑ ΚΑΤΑΝΟΗΤΟ. ΔΕΝ ΜΠΟΡΕΙ ΝΑ ΕΙΝΑΙ ΑΙΩΝΙΟΣ

ΕΠΕΙΔΗ ΕΧΕΙ ΔΗΜΙΟΥΡΓΗΘΕΙ ΜΕ ΤΗ ΘΕΛΗΣΗ ΤΟΥ ΘΕΟΥ ΕΚ ΤΟΥ ΜΗΔΕΝΟΣ.

2. Η ΠΡΩΤΗ ΕΝΝΟΙΑ ΤΟΥ ΚΟΣΜΟΥ - ΣΥΜΠΑΝΤΟΣ ΔΕΝ ΑΠΟΤΕΛΕΙ ΟΛΟΝ, ΕΙΝΑΙ

ΣΥΝΑΘΡΟΙΣΗ ΕΤΕΡΟΓΕΝΗΣ. Η ΟΠΟΙΑ ΠΕΡΙΛΑΜΒΑΝΕΙ ΚΑΙ ΤΟΝ ΘΕΟ, Ο

ΟΠΟΙΟΣ ΕΙΝΑΙ ΤΟ ΑΠΟΛΥΤΟ ΤΕΛΕΙΟ, ΣΥΝΕΠΩΣ Η ΣΥΝΑΘΡΟΙΣΗ ΔΕΝ

ΜΠΟΡΕΙ ΝΑ ΕΧΕΙ ΤΟΝ ΙΔΙΟ ΒΑΘΜΟ ΤΕΛΕΙΟΤΗΤΑΣ.

3. ΤΟ ΕΤΕΡΟΓΕΝΕΣ ΣΥΝΟΛΟ ΤΩΝ ΥΛΙΚΩΝ ΣΩΜΑΤΩΝ ΤΟΥ ΚΟΣΜΟΥ ΔΕΝ

ΜΠΟΡΕΙ ΝΑ ΕΙΝΑΙ ΤΕΛΕΙΟ ΓΙΑΤΙ ΜΟΝΟΝ Ο ΘΕΟΣ ΕΙΝΑΙ ΤΕΛΕΙΟΣ.

4. Ο ΘΕΟΣ ΓΕΝΙΚΑ ΔΕΝ ΚΑΤΑΣΚΕΥΑΖΕΙ ΑΠΕΙΡΟ ΕΝ ΕΝΕΡΓΕΙΑ, ΣΥΝΕΠΩΣ ΚΑΙ

ΑΠΟΛΥΤΑ ΤΕΛΕΙΟ, ΓΙΑΤΙ ΑΥΤΟ ΘΑ ΠΕΡΙΟΡΙΖΕ ΤΗΝ ΔΥΝΑΜΗ ΤΟΥ, ΑΦΟΥ

ΜΕΤΑ ΔΕΝ ΘΑ ΜΠΟΡΟΥΣΕ ΝΑ ΚΑΤΑΣΚΕΥΑΣΕΙ ΚΑΤΙ ΤΕΛΕΙΟΤΕΡΟ.

5. ΔΕΧΕΤΑΙ ΤΗΝ ΥΠΕΡΦΥΣΙΚΗ ΔΥΝΑΤΟΤΗΤΑ ΥΠΑΡΞΗΣ ΕΞΩΚΟΣΜΙΚΟΥ ΚΕΝΟΥ.

Ο ΘΕΟΣ ΘΑ ΜΠΟΡΟΥΣΕ ΝΑ ΤΟ ΦΤΙΑΞΕΙ ΑΛΛΑ ΣΤΗΝ ΠΡΑΓΜΑΤΙΚΟΤΗΤΑ ΔΕΝ

ΘΑ ΠΡΕΠΕΙ ΝΑ ΤΟ ΥΠΟΘΕΣΟΥΜΕ ΓΙΑΤΙ ΟΦΕΙΛΟΥΜΕ ΝΑ ΥΠΟΘΕΤΟΥΜΕ

ΠΡΑΓΜΑΤΑ ΤΑ ΟΠΟΙΑ ΔΕΝ ΕΙΝΑΙ ΑΝΤΙΛΗΠΤΑ ΑΠΟ ΤΙΣ ΑΙΣΘΗΣΕΙΣ ΟΥΤΕ

ΑΠΟ ΤΗΝ ΕΜΠΕΙΡΙΑ , ΟΥΤΕ ΑΠΟ ΤΟΝ ΦΥΣΙΚΟ ΛΟΓΟ, ΟΥΤΕ ΑΠΟ ΤΗΝ ΒΙΒΛΟ.

6. ΟΙ ΟΥΡΑΝΟΙ ΕΧΟΥΝ ΔΥΝΗΤΙΚΑ ΤΙΣ ΠΟΙΟΤΗΤΕΣ ΤΟΥ ΘΕΡΜΟΥ ΚΑΙ ΤΟΥ

ΨΥΧΡΟΥ, ΤΟΥ ΥΓΡΟΥ ΚΑΙ ΞΗΡΟΥ, ΟΧΙ ΟΜΩΣ ΣΤΗ ΠΡΑΓΜΑΤΙΚΟΤΗΤΑ ,ΜΕ

ΑΥΤΟΝ ΤΟΝ ΤΡΟΠΟ ΕΠΙΔΡΟΥΝ ΣΕ ΑΛΛΑΓΕΣ ΘΕΡΜΟΤΗΤΑΣ ΣΤΟ

ΥΠΟΣΕΛΗΝΙΟ ΧΩΡΙΟ ,ΧΩΡΙΣ ΝΑ ΕΧΟΥΜΕ ΑΛΛΑΓΕΣ ΣΤΟ ΥΠΕΡΣΕΛΗΝΙΟ.

7. ΑΝΑΦΕΡΕΙ ΟΠΩΣ ΚΑΙ Ο ΑΡΙΣΤΟΤΕΛΗΣ ΟΤΙ ΔΕΝ ΠΑΡΑΤΗΡΗΘΗΚΑΝ ΠΟΤΕ

ΑΛΛΑΓΕΣ ΣΤΟΥΣ ΟΥΡΑΝΟΥΣ.

8. ΤΟ ΧΑΡΑΚΤΗΡΙΣΤΙΚΟ ΤΗΣ ΥΛΗΣ ΕΙΝΑΙ Η ΜΕΤΑΒΛΗΤΟΤΗΤΑ ΤΗΣ, Η ΟΠΟΙΑ

ΔΕΝ ΠΑΡΑΤΗΡΕΙΤΑΙ ΣΤΟΥΣ ΟΥΡΑΝΟΥΣ, ΣΥΝΕΠΩΣ "ΕΙΝΑΙ ΑΠΟΛΥΤΑ ΜΑΤΑΙΟ

ΚΑΙ ΧΩΡΙΣ ΠΕΙΣΤΙΚΟ ΛΟΓΟ ΝΑ ΒΑΛΟΥΜΕ ΤΗΝ ΥΛΗ ΣΤΟΥΣ ΟΥΡΑΝΟΥΣ..

Page 85: ΣΗΜΕΙΩΣΕΙΣ ΦΙΛΟΣΟΦΙΑΣ I

ΤΑΥΤΟΧΡΟΝΑ "ΣΩΖΟΝΤΑΣ ΤΑ ΦΑΙΝΟΜΕΝΑ" ΑΝΑΦΕΡΕΙ ΟΤΙ ΑΥΤΑ ΣΩΖΟΝΤΑΙ

ΜΕ ΤΗΝ ΥΠΟΘΕΣΗ ΜΙΑΣ ΑΠΛΗΣ ΟΥΡΑΝΙΑΣ ΟΥΣΙΑΣ Η ΟΠΟΙΑ ΕΠΕΙΔΗ

ΑΚΡΙΒΩΣ ΛΕΙΤΟΥΡΓΕΙ ΩΣ ΦΟΡΕΑΣ ΕΚΤΕΤΑΜΕΝΩΝ ΜΕΓΕΘΩΝ, ΘΑ ΠΡΕΠΕΙ ΝΑ

ΕΧΕΙ ΑΚΡΙΒΩΣ ΤΗΝ ΙΔΙΟΤΗΤΑ ΤΗΣ ΕΚΤΑΣΗΣ. Η ΑΜΕΤΑΒΛΗΤΟΤΗΤΑ ΤΗΣ

ΟΥΡΑΝΙΑΣ ΥΛΗΣ ΕΙΝΑΙ ΣΧΕΤΙΚΗ ΚΑΙ ΕΞΑΡΤΩΜΕΝΗ ΑΠΟ ΤΗΝ

ΠΑΝΤΟΔΥΝΑΜΙΑ ΤΟΥ ΘΕΟΥ.

9. Η ΟΡΜΗ (IMPETUS) ΕΙΝΑΙ ΚΑΤΙ ΤΟ ΟΠΟΙΟ ΕΧΕΙ ΣΤΑΘΕΡΗ ΦΥΣΗ ΔΙΑΚΡΙΤΗ

ΚΑΙ ΕΙΝΑΙ ΜΙΑ ΠΟΙΟΤΗΤΑ Η ΟΠΟΙΑ ΥΠΑΡΧΕΙ ΑΠΟ ΤΗΝ ΦΥΣΗ ΚΑΙ ΕΙΝΑΙ

ΠΡΟΟΡΙΣΜΕΝΗ ΑΠΟ ΑΥΤΗ ΝΑ ΚΙΝΕΙ ΤΟ ΣΩΜΑ ΠΑΝΩ ΣΤΟ ΟΠΟΙΟ ΕΙΝΑΙ

ΑΠΟΤΥΠΩΜΕΝΗ.

10.ΜΕΣΩ ΤΗΣ ΘΕΩΡΙΑΣ ΤΟΥ ΠΕΡΙ "IMPETUS" (ΕΠΙΒΑΛΛΟΜΕΝΗ ΔΥΝΑΜΗ) ΚΑΙ

ΤΟ ΠΑΡΑΔΕΙΓΜΑ ΤΟΥ ΠΛΟΙΟΥ ΚΑΙ ΤΟΥ ΒΕΛΟΥΣ ΘΕΩΡΕΙ ΟΤΙ ΑΥΤΟ ΟΤΑΝ

ΔΙΔΕΤΑΙ ΣΤΟ ΒΕΛΟΣ ΕΙΝΑΙ ΣΕ ΘΕΣΗ ΝΑ ΥΠΕΡΝΙΚΗΣΕΙ ΑΥΤΗ ΤΗ ΜΕΤΑΦΟΡΑ

ΤΟΥ ΠΕΡΙΒΑΛΛΟΝΤΟΣ ΑΕΡΑ ΚΑΙ ΕΠΕΙΔΗ ΔΕΝ ΠΑΡΑΤΗΡΗΘΗΚΕ ΠΤΩΣΗ ΤΟΥ

ΒΕΛΟΥΣ ΣΕ ΔΥΤΙΚΟΤΕΡΟ ΣΗΜΕΙΟ ΚΑΤΑΛΗΓΕΙ ΟΤΙ Η ΓΗ ΕΙΝΑΙ ΑΚΙΝΗΤΗ.

11.ΟΙ ΑΣΤΡΟΝΟΜΟΙ ΠΑΡΑΤΗΡΟΥΝ ΣΧΕΤΙΚΕΣ ΚΑΙ ΟΧΙ ΑΠΟΛΥΤΕΣ ΤΑΧΥΤΗΤΕΣ,

ΚΑΙ ΑΡΑ Η ΑΠΟΔΟΣΗ ΠΕΡΙΣΤΡΟΦΙΚΗΣ ΚΙΝΗΣΗΣ ΣΤΗ ΓΗ ΔΕΝ ΘΑ ΕΙΧΕ

ΚΑΜΙΑ ΣΥΝΕΠΕΙΑ ΣΤΟΥΣ ΑΣΤΡΟΝΟΜΙΚΟΥΣ ΥΠΟΛΟΓΙΣΜΟΥΣ.

12.ΕΙΝΑΙ ΜΗ ΑΠΟΔΕΙΞΙΜΗ Η ΠΡΟΤΑΣΗ ΟΤΙ ΕΝΑ ΣΩΜΑ ΤΟΥ ΑΛΛΟΥ ΚΟΣΜΟΥ

(ΚΑΤΑ ΤΟΝ ΑΡΙΣΤΟΤΕΛΗ) ΘΑ ΚΙΝΟΥΝΤΑΝ ΠΡΟΣ ΤΗ ΓΗ ΛΕΓΟΝΤΑΣ ΟΤΙ

ΤΕΛΙΚΑ ΟΙ ΚΙΝΗΣΕΙΣ ΕΞΩ ΑΠΟ ΤΗ ΓΗ ΕΠΗΡΕΑΖΟΝΤΑΙ ΑΠΟ ΤΑ ΑΛΛΑ

ΟΥΡΑΝΙΑ ΣΩΜΑΤΑ ΚΑΙ ΤΟΝ ΘΕΟ

13.ΑΠΟΤΕΛΕΙ Η ΤΟΠΙΚΗ ΚΙΝΗΣΗ ΧΩΡΙΣΤΗ ΟΝΤΟΤΗΤΑ ΑΠΟ ΤΟ ΚΙΝΟΥΜΕΝΟ

ΣΩΜΑ ΚΑΙ ΤΟΥΣ ΤΟΠΟΥΣ ΠΟΥ ΑΥΤΟ ΚΑΤΑΛΑΜΒΑΝΕΙ ΔΙΑΔΟΧΙΚΑ;

ΑΝΑΦΕΡΟΜΕΝΟΣ ΣΤΟ ΘΕΟΛΟΓΙΚΟ ΔΟΓΜΑ ΘΕΩΡΕΙ ΟΤΙ Ο ΘΕΟΣ ΕΙΝΑΙ

ΙΚΑΝΟΣ ΝΑ ΚΑΝΕΙ ΟΤΙΔΗΠΟΤΕ ΔΕΝ ΟΔΗΓΕΙ ΣΕ ΛΟΓΙΚΗ ΑΝΤΙΦΑΣΗ ΚΑΙ

ΛΟΓΩ ΤΗΣ ΠΑΝΤΟΔΥΝΑΜΙΑΣ ΤΟΥ ΘΑ ΜΠΟΡΟΥΣΕ ΝΑ ΕΠΙΒΑΛΛΕΙ ΣΤΟ

Page 86: ΣΗΜΕΙΩΣΕΙΣ ΦΙΛΟΣΟΦΙΑΣ I

ΣΥΜΠΑΝ ΣΥΝΟΛΙΚΑ ΜΙΑ ΠΕΡΙΣΤΡΟΦΙΚΗ ΚΙΝΗΣΗ.

1. ΟΙ ΤΡΕΙΣ ΤΡΟΠΟΙ ΠΡΟΣΛΗΨΗΣ ΤΟΥ ΑΡΙΣΤΟΤΕΛΙΣΜΟΥ:

Ο αριστοτελισμός προσλαμβάνεται από τη χριστιανική Ευρώπη με τρεις τρόπους: πρώτον,

μέσω

του έργου του Αγίου Θωμά, το οποίο αποτέλεσε την κορυφαία έκφραση της προσπάθειας

συγκερασμού Λόγου και πίστης- δεύτερον, με το. έργο του Σίζερ και άλλων, με το οποίο ο

Λόγος αναβαθμίζεται σε βαθμό που να προκαλέσει αντίδραση η οποία διήρκεσε σχεδόν

όλο τον

13ο αιώνα, με αποκορύφωση την 7η Μαρτίου του 1277· τρίτον, μέσω της αντιπαράθεσης

με τον

αριστοτελισμό, με κύριο εκπρόσωπο τον Άγιο Μποναβεντούρα, ο οποίος εξέφρασε

νεοαυγουστινιανές θέσεις.

2. ΣΙΖΕΡ Η ΔΙΠΛΗ ΑΛΗΘΕΙΑ - ΓΕΝΙΚΗ ΠΑΡΟΥΣΙΑΣΗ

Η διπλή αλήθεια, με την κυριολεκτική έννοια του όρου, την περίοδο που εξετάζουμε είναι

το

κριτήριο της θρησκευτικότητας. Τρεις είναι οι τρόποι της γενικής παρουσίασης του

προβληματισμού γύρω από το γεγονός ότι η φιλοσοφική και η χριστιανική διδασκαλία

μπορεί να

αντιφάσκουν:

Μπορεί κανείς να δεχτεί ταυτόχρονα ότι οι δυο αλήθειες αντιφάσκουν, όπως π.χ. για το

θέμα

της αιωνιότητας ή της δημιουργίας του κόσμου. Οι λόγιοι λέγουν ότι αναφέρουν απλώς τις

θεωρίες του Αριστοτέλη χωρίς να εκφράζουν τις δικές τους απόψεις: ο Θωμάς μπορεί να

υπαχθεί σε αυτή την κατηγορία. Η αντίφαση ερμηνεύεται λέγοντας ότι τα συμπεράσματα

της

φιλοσοφίας είναι

ανακάλυψη του ανθρώπινου νου, ενώ οι αλήθειες της πίστης Αποκαλυπτικές, συνεπώς σε

περίπτωση αντίφασης πρέπει να γίνουν δεκτές οι τελευταίες.

ΤΑ ΠΑΝΕΠΙΣΤΗΜΙΑ ΣΧΟΛΑΣΤΙΚΙΣΜΟΣ

Τα πανεπιστήμια που εξετάζουμε είχαν σαφέστατους κανόνες λειτουργίας. Όπως λέχθηκε:

Page 87: ΣΗΜΕΙΩΣΕΙΣ ΦΙΛΟΣΟΦΙΑΣ I

σκέψη είναι ένα επάγγελμα του οποίου οι κανόνες είναι λεπτομερώς ορισμένοι" Η

μεθοδολογία

της .διδασκαλίας ήταν ο σχολαστικισμός. Βάση του σχολαστικισμού ήταν η γραμματική

δεδομένου ότι οι λόγιοι έπρεπε να γνωρίζουν τη σχέση ανάμεσα στη λέξη και στον κόσμο,

απαίτηση που εξηγεί και τη σοβαρότητα του προβλήματος νομιναλισμού - ρεαλισμού του

Μεσαίωνα Η Μέθοδος απόδειξης ήταν η διαλεκτική, ένα σύνολο διαδικασιών μέσω των

οποίων

το αντικείμενο της γνώσης μετατρέπεται σε πρόβλημα και υπόκειται σε διάλογο

αντιτιθέμενων

πλευρών. Είχε γίνει όμως από τότε σαφές ότι η μέθοδος κινδύνευε να καταλήξει σε

απεραντολογία αν τα επιχειρήματα δεν γίνουν αντιληπτά κάπου αλλού (εκτός του

διαλόγου)

(ό.π. σ. 87).

ΤΑ ΕΞΙ ΣΗΜΕΙΑ ΤΗΣ ΑΝΤΙΘΕΣΗΣ ΑΡΙΣΤΟΤΕΛΙΣΜΟΥ ΚΑΙ ΦΙΛΟΣΟΦΙΑΣ

Τα έξι σημεία στα οποία ο αριστοτελισμός έρχεται σε αντίθεση με τη χριστιανική θεολογία

είναι:

α) η αιωνιότητα του κόσμου β) ο μονοψυχισμός γ) η άρνηση της προσωπικής αθανασίας δ)

η

αιτιοκρατία ε)η άρνηση της θείας πρόνοιας στ) η άρνηση της ελεύθερης βούλησης.

ΤΑ 4 ΕΜΠΟΔΙΑ ΚΑΤΑΝΟΗΣΗΣ ΤΗΣ ΑΛΗΘΕΙΑΣ ΤΟΥ ΜΠΕΙΚΟΝ.

Τα τέσσερα εμπόδια για την κατανόηση της αλήθειας είναι η υποταγή σε λανθασμένη

αυθεντία,

η δύναμη της συνηθείας ,η λαϊκή πρόληψη και τέλος η επικάλυψη της αγνοίας με επίδειξη

σοφίας.

ΕΝΤΑΣΕΙΣ ΚΑΙ ΔΙΑΜΑΧΕΣ ΤΟ 1277

Οι συγκρούσεις μεταξύ αριστοτελισμού και θεολογίας οι οποίες σχετίζονται με την φύση

του

Θεού και την ανθρώπινη ικανότητα γνώσης του, τον περιορισμό της παντοδυναμίας του και

της

γνώσης του, την άποψη ότι ο κόσμος είναι αιώνιος, όπως και στα θέματα της

αναγκαιότητας και

Page 88: ΣΗΜΕΙΩΣΕΙΣ ΦΙΛΟΣΟΦΙΑΣ I

του τυχαίου, στην αβερροική ενότητα της διάνοιας, στους περιορισμούς της ανθρώπινης

ελευθερίας που πηγάζουν από τη θεώρηση της ανθρώπινης θέλησης ως ενεργούμενου

άλλου

καθώς και στο θέμα της προτεραιότητας του δικαιώματος στην αλήθεια ανάμεσα σε

θεολόγους

και φιλόσοφους. Επίσης στις ενδοθεολογικές συγκρούσεις ανάμεσα σε Νεοαυγουστιανούς,

θωμιστές και ριζοσπάστες αριστοτελικούς.

7. Οι διαφορές της νια Moderna από τη Via Antiqua του 14ου αιώνα μπορούν να

ταξινομηθούν

στις ακόλουθες περιοχές.

Η πρώτη αφορά τη φύση του γνώσιμου και της πηγής της γνώσης τα οποία ο Όκκαμ ταύτισε

με

ατομικά αντικείμενα που υπάρχουν έξω από τον νου και αποτελούν τη μοναδική

οντολογική

πραγματικότητα, οδηγώντας σε μια ασυμμετρία ανάμεσα στο Εννοιολογικό και στο

Πραγματικό.

Γενική με τον Όκκαμ, αλλά ήταν εφικτή μόνο μέσα από τη γνώση των ατομικών και τελικά

αφορούσε αυτά Η αντιστροφή δηλαδή ήταν, αντί να γίνεται προσπάθεια ερμηνείας των

ατομικών όντων σ έναν κόσμο καθολικών εννοιών να μελετούνται τα καθολικά σε έναν

απόλυτα

ατομικό κόσμο. Η μεταφυσική τάξη της προηγούμενης αντίληψης θα πρέπει να

αντικατασταθεί

με τη λογική τάξη, και οι καθολικές έννοιες θα γίνονται αντικείμενο μελέτης στον βαθμό

που το

νόημα τους εξαρτάται από γραμματικούς τύπους και από το λογικό περιεχόμενο των

σχετικών

όρων. Η δεύτερη περιοχή διαφορών ανάμεσα στις δυο θεωρήσεις αφορά θέματα πίστης

και

γνώσης

Ανεξάρτητα της αποδοχής ή μη των αντιλήψεων του Όκκαμ και, του 14ου αιώνα, οι

χριστιανοί

πίστευαν στην εγγενή αβεβαιότητα και στο τυχαίο, όλων των όντων πλην του Θεού. Αυτή η

Page 89: ΣΗΜΕΙΩΣΕΙΣ ΦΙΛΟΣΟΦΙΑΣ I

αντίληψη δεν συγκαλύπτει τις διαφορές που εκφράζονταν αυτή την περίοδο του

εκλεκτικισμού,

π.χ. από τον Νικόλαο της Ωτρεκούρτ, ο οποίος προχωρώντας πέρα από τον Όκκαμ έθεσε

απόλυτα όρια στη βεβαιότητα των γνώσεων μας ακόμη και για τα ατομικά αντικείμενα. Στη

χριστιανική παράδοση υπήρχε πάντοτε η παντοδυναμία του Θεού, η δυνατότητα Του να

κάνει

τον κόσμο όπως θέλει, το τυχαίο δηλαδή της Δημιουργίας. Οι νόμοι που επέλεξε ο Θεός για

το

δημιούργημα Του, τον κόσμο, ήταν συμβατικά μόνο αναγκαίοι, λόγω της θέλησης Του και

όχι

λόγω απόλυτης αναγκαιότητας τους.

8. ΤΟ ΑΡΙΣΤΟΤΕΛΙΚΟ ΔΙΑΝΟΗΤΙΚΟ ΠΛΑΙΣΙΟ

Η αριστοτέλεια αντίληψη για τη μελέτη της φύσης απαιτεί τη χρήση των συγκεκριμένων

κατηγοριών του Αριστοτέλη: της ουσίας της ποσότητας, της ποιότητας, της σχέσης, του

τόπου,

του χρόνου, της θέσης, της κατοχής, της ενέργειας και του πάθους. Η ταξινόμηση, π.χ., των

μεταβολών στον Σταγειρίτη είναι η εξής: η μεταβολή της ίδιας της ουσίας, η γένεσις και η

φθορά, η μεταβολή η οποία δρα σε υπάρχουσες ουσίες και είναι η κίνηση. Η κίνηση

ταξινομείται

στον Αριστοτέλη ως εξής: εάν η μεταβολή αφορά την ποιότητα της ουσίας έχουμε

αλλοίωση,

εάν αφορά την ποσότητα της, έχουμε αύξηση ή φθίση και, τέλος, εάν αφορά τον τόπο της

έχουμε φορά. Η φορά συμπίπτει με τη δική μας έννοια της κίνησης. Αυτές οι τρεις κινήσεις -

μεταβολές - εμπεριέχουν τους όρους "κινούν" και "κινούμενο" ως θεμελιακά στοιχεία τους,

και ο

Αριστοτέλης δεν επιχειρεί να αναγάγει το ένα είδος της μεταβολής στο άλλο. Ουσιαστικά

όπως

αναφέρει και, ο Leff (1976) πρόκειται για μια μέθοδο ταξινόμησης των φυσικών

φαινομένων η

οποία ενώ πηγάζει από εμπειρικά δεδομένα. στη συνέχεια παγιώνεται σε αφαιρέσεις, δεν

σχετίζεται με την παρατήρηση και το πείραμα, και το κυριότερο, βρίσκεται πλησιέστερα

στην

Page 90: ΣΗΜΕΙΩΣΕΙΣ ΦΙΛΟΣΟΦΙΑΣ I

όλη μεταφυσική του Αριστοτέλη. Αυτή η κυρίαρχη επί δύο χιλιάδες χρόνια άποψη δεν

καταρρίπτεται τον 14ο αιώνα, αφού ούτε σε αυτό τον αιώνα υπήρξε ουσιαστική

πειραματική

επιβεβαίωση των νέων θεωριών, απλώς ο 14ος αιώνας ανασχημάτισε το αριστοτέλειο

πλαίσιο με

βάση τις έννοιες της ύλης και της ποσότητας και επέλυσε σχετικά προβλήματα με χρήση

υπολογισμών, αντί μέσω των ταξινομήσεων και της χρήσης ορισμών. Τα θέματα της

ταξινόμησης των επιστημών θα μας απασχολήσουν σε επόμενο κεφάλαιο.

Η μεθοδολογία της πανεπιστημιακής διδασκαλίας ήταν ο σχολαστικισμός με βάση την

γραμματική. Οι λόγιοι έπρεπε να γνωρίζουν τη σχέση ανάμεσα στη λέξη και στον κόσμο,

απαίτηση που εξηγεί και την σοβαρότητα του προβλήματος νομιναλισμού - ρεαλισμού του

Μεσαίωνα. Μέθοδος απόδειξης ήταν η Διαλεκτική, ένα σύνολο διαδικασιών μέσω των

οποίων το

αντικείμενο της γνώσης μετατρέπεται σε πρόβλημα και υπόκειται σε διάλογο

αντιτιθέμενων

πλευρών.

Οι 219 καταδικασμένες προτάσεις προέκυψαν ως αποτέλεσμα της ανάδυσης τριών

διακριτών

ιδεολογικών ρευμάτων στο Παρίσι: του Αλβέρτου και του Ακινάτη που προσπάθησαν να

εναρμονίσουν Λόγο και πίστη, των ριζοσπαστικών Αριστοτελιστών και των

ΝεοΑυγουστιανών οι

οποίοι και τελικά επέβαλλαν τις καταδίκες. Αν η φιλοσοφία κατέληγε τελικά συστηματικά

σε

συμπεράσματα αντίθετα από την πίστη , δεν μπορούσε πλέον να θεωρείται ως έμπιστη

θεραπαινίδα, αντίθετα άρχιζε να μεταβάλλεται σε εχθρική δύναμη και απειλή, η οποία

απαιτούσε

αποφασιστική δράση.

Κοϋρέ : Οι Προτάσεις δεν έχουν πρακτική αξία στις επιστήμες

Duhem: Οι καταδίκες του 1277 είναι η πράξη γένεσης της επιστήμης των νέων χρόνων,

επειδή

καθιέρωσαν τη δυνατότητα μελέτης μη Αριστοτελικών θέσεων.

Page 91: ΣΗΜΕΙΩΣΕΙΣ ΦΙΛΟΣΟΦΙΑΣ I

Τα έξι σημεία στα οποία ο αριστοτελισμός έρχεται σε αντίθεση με τη χριστιανική θεολογία

είναι :

α) η αιωνιότητα του κόσμου β) ο μονοψυχισμός γ) η άρνηση της προσωπικής αθανασίας δ)

η

αιτιοκρατία ε)η άρνηση της θείας πρόνοιας στ) η άρνηση της ελεύθερης βούλησης.

Σχετικά με ΕΝΤΑΣΕΙΣ και ΔΙΑΜΑΧΕΣ ΤΟ 1277: Οι συγκρούσεις μεταξύ αριστοτελισμού και

θεολογίας οι οποίες σχετίζονται με την φύση του Θεού και την ανθρώπινη ικανότητα

γνώσης

του, τον περιορισμό της παντοδυναμίας του και της γνώσης του, την άποψη ότι ο κόσμος

είναι

αιώνιος, όπως και στα θέματα της αναγκαιότητας και του τυχαίου, στην αβερροική ενότητα

της

διάνοιας, στους περιορισμούς της ανθρώπινης ελευθερίας που πηγάζουν από τη θεώρηση

της

ανθρώπινης θέλησης ως ενεργούμενου άλλου καθώς και στο θέμα της προτεραιότητας του

δικαιώματος στην αλήθεια ανάμεσα σε θεολόγους και φιλόσοφους. Επίσης στις

ενδοθεολογικές

συγκρούσεις ανάμεσα σε Νεοαυγουστιανούς, θωμιστές και ριζοσπάστες αριστοτελικούς.

Όποιες και αν ήταν οι κινητήριες δυνάμεις που ωθούσαν την αναθεώρηση της

αριστοτελικής

θεωρίας της κίνησης, η μεσαιωνική αποτίμηση του Αριστοτέλη είχε ζωτικές συνέπειες για

την

πορεία της φυσικής φιλοσοφίας.

Κατά την γνώμη του Κοϋρέ η Επιστημονική επανάσταση του 16ου και 17ου αιώνα δεν ήταν

εκβλάστηση η επέκταση της μεσαιωνικής επιστήμης αλλά μια πνευματική μετάλλαξη που

οδήγησε στη διάλυση της.